SCS (Copied)

¡Supera tus tareas y exámenes ahora con Quizwiz!

What is the external rotation limitations for anterior shoulder instability surgery?

External rotation range of motion is limited to 30° (0° abduction) for the first 4 postoperative weeks.

What are the 6 phases of throwing?

"1) Wind-up 2) Stride 3) Arm Cocking 4) Arm Acceleration 5) Arm Deceleration

What is the follow up medial humeral fracture?

Late follow-up should be considered to screen for growth disturbance after injury to the epiphysis.

What are the signs and symptoms of SAI?

Pain weakness, loss of movement in the affected shoulder.. pain worsened with overhead movement... pain may be chronic or acute... limited ROM

What is the pathology and MOI of posterior instability?

Posterior shoulder instability is generally uncommon and rarely occurs alon. Contributing existing/co-existing factors include microtrauma or macrotrauma injuries to the shoulder. previous dislocations, soft tissue or boney structural abnormalities, and scapulothoracic dysfunction. PSI can occur through volitional means or from positional or muscular instabilities.

What is TACS procure?

Reinold et al43 reported that in a series of 130 overhead athletes with symptomatic hyper laxity undergoing thermal-assisted capsular shrinkage of the glenohumeral joint (TACS), 69% exhibited superior labral degeneration, while 35% had type II SLAP lesions.

What is the most functional ER infraspinatus strengthening therex.

standing ER

When do you start bicep strengthening for SLAP surgery?

ight biceps resistance is usually not initiated until 2 weeks following surgery in an at- tempt to prevent debridement site irritation.

What therex produces the greatness EMG activity for the infraspinatus?

prone ER at 90

Which therex produces high EMG for the serratus anterior?

push-up with a plus

What ligaments is responsible for preventing anterior humeral translation in the 90/90 position?

...

What specific ligament is most often injured in the pitchers elbow?

...

What specific nerve injuries could lead to scapular dyskinesia, and why are they prevalent in pitchers?

...

What causes SICK scapula, and what problems stem from scapular dyskinesia?

...

What is the pathology and MOI of a humeral fracture

1) A humeral fracture is caused by a direct blow or force to the humerus. 2) An indirect stress due to falling on an outstretched hand or violent muscle contraction. 3) Rarely, though possible, throwing hand enough to produce a twisting force can fracture the humerus.

What is the pathophysiology for clavicle fracture?

Because of its subcutaneous position, the clavicle may be fractured easily, and the fracture is often an isolated injury. However, clavicle fractures are also common in the context of high-energy injury or multiple traumatic injuries. In these situations, it is important to examine the patient for other associated injuries, such as rib fractures, scapula fractures, other fractures about the shoulder girdle, pulmonary contusion, pneumothorax, hemothorax, and closed head injuries.

What are the laboratory studies for median neuropathy?

Blood tests to screen for underlying rheumatologic or inflammatory disease or other treatable systemic conditions known to be associated with CTS, if suspected: Fasting blood glucose and hemoglobin A1c Thyroid function (thyroid stimulating hormone [TSH]) Vitamin B-12 level Erythrocyte sedimentation rate (ESR) Rheumatoid factor

What films do you order for the stress fracture of the olcranon?

Bone scan or CT scan, or both, can be helpful for detect- ing stress fractures of the olecranon. A CT scan may be preferable for identifying the facture line and the typical sclerotic bone signifying stress reaction, although a bone scan will generally show increased uptake at the stress fracture site.

What are the other medial neuropathy test?

Electrodiagnosis: The clinical bedside examination, including diagnostic provocative tests, have low validity, and patients with CTS symptoms should be referred directly for neurophysiologic examination. Electrodiagnostic studies remain the criterion standard for diagnosis of CTS, but should always be interpreted in combination of the clinical symptoms and signs. Nerve conduction study (NCS): NCS measures the sensory and motor nerve conduction velocity (latency) and amplitudes across the wrist. Any focal median nerve conduction delay implies a demyelinative lesion of the median nerve. In mild or early CTS, there is usually conduction delay of sensory fibers only, without prolongation of distal motor latency. In more severe CTS, focal conduction block or secondary axon loss results in decreased median nerve sensory and motor amplitudes. Routine NCS may miss the diagnosis of CTS in up to 25% of cases. The sensitivity is greatly improved by measuring the median nerve latency within a shorter segment across the wrist in comparison to an adjacent nerve for the same distance. The clinical diagnosis of CTS may thus be confirmed with a high degree of sensitivity (>85%) and specificity (>95%).[34] Multiple techniques are used to diagnosis CTS. A typical electrodiagnostic protocol may include the following: Antidromic sensory NCS are recorded from a median innervated digit, typically by placing ring electrodes on the index finger, with electrical stimulation at the wrist at a distance of 14 cm. For internal median nerve comparison, the sensory potential from the index finger is also recorded by stimulation in the palm of the hand, at a midpoint between the wrist stimulation site and the recording ring electrode on the index (7 cm to each). The upper normal limit for the peak latency of the distal segment is 1.9 ms. The upper limit for the calculated peak latency difference between the wrist and palm stimulations is 1.6 ms. Antidromic sensory NCS of the ulnar nerve is recorded with ring electrodes on digit 5 and stimulation at the wrist (on the ulnar aspect). The median motor response is recorded with surface electrodes over the abductor pollicis brevis muscle (APB) and stimulation at the wrist and elbow. If the above routine median NCS is not diagnostic, the median nerve latency across the transcarpal segment is measured in comparison to an adjacent nerve (ulnar or radial).[35, 36] This strategy may be particularly helpful in mild CTS cases and in patients with underlying polyneuropathy to detect superimposed focal median conduction delay. Sensitivity and specificity of these internal comparison studies depends greatly on the upper normal limit (cutoff) values. In patients with ulnar neuropathy, the comparison should be to the radial nerve or by internal median nerve comparison of the wrist segment versus the finger segment. Median versus ulnar palmar mixed nerves studies (orthodromic) with stimulation at the palm and recording from the wrist at a distance of 8 cm. A palmdiff of 0.4 ms or greater is abnormal. Median versus ulnar nerve distal sensory latencies to the ring finger, with stimulation at the wrist, at a distance of 14 cm (antidromic). A ringdiff of 0.5 ms or greater is abnormal. Median versus radial nerve distal sensory latencies to the thumb with stimulation at the wrist, at a distance of 10 cm (antidromic). A thumbdiff of 0.5-0.7 ms or greater is abnormal. Needle electromyography (EMG) testing is optional for the diagnosis of CTS. It may be needed to differentiate CTS from cervical radiculopathy. In cases where surgery is being considered, it may document severity of CTS by documenting denervation to the APB muscle. Robinson et al[35] recommended the use of the combined sensory index (CSI) defined as the sum of the 3 latency differences listed above under 5) with higher sensitivity and reliability than the individual tests. Sensitivity for the tests was palmdiff 69.7%, ringdiff 74.2%, thumbdiff 75.8%, and CSI 83.1%. Specificity was 95.4-96.9%. Requiring 1, 2, or 3 tests to be abnormal yielded sensitivities of 84.8%, 74.2%, or 56.1%, respectively, but specificities of 92.3%, 98.5%, and 100%, respectively. In a follow-up retrospective report on a larger patient group (300 hands), the same authors determined endpoints for individual tests that confidently predicted the results of the CSI; for ranges between these endpoints, further testing was required. These ranges were palmdiff 0-0.3 ms, ringdiff 0.1-0.4 ms, and thumbdiff 0.2-0.7 ms.[37] A smaller prospective study of the same technique documented the overall superiority of the SCI versus individual tests for diagnostic accuracy, but when individual tests were markedly abnormal, it was not necessary to perform all 3 nerve conduction studies.[38] Electrodiagnostic studies in carpal tunnel syndrome A report of the American Association of Electrodiagnostic Medicine, American Academy of Neurology, and the American Academy of Physical Medicine and Rehabilitation published in 2002 recommended the following electrodiagnostic studies in patients with suspected CTS (see list below for sensitivity and specificity of Techniques A-K):[34] Perform a median sensory NCS across the wrist with a conduction distance of 13-14 cm (Technique G). If the result is abnormal, compare the result of the median sensory NCS to the result of a sensory NCS of 1 other adjacent sensory nerve in the symptomatic limb (Standard). If the initial median sensory NCS across the wrist has a conduction distance greater than 8 cm and the result is normal, 1 of the following additional studies is recommended: Comparison of median sensory or mixed nerve conduction across the wrist over a short (7-8 cm) conduction distance (Technique C) with ulnar sensory nerve conduction across the wrist over the same short (7-8 cm) conduction distance (Technique D) (Standard) Comparison of median sensory conduction across the wrist with radial or ulnar sensory conduction across the wrist in the same limb (Techniques B and F) (Standard) Comparison of median sensory or mixed nerve conduction through the carpal tunnel to sensory or mixed NCSs of proximal (forearm) or distal (digit) segments of the median nerve in the same limb (Technique A) (Standard) Perform a motor conduction study of the median nerve recording from the thenar muscle (Technique H) and of 1 other nerve in the symptomatic limb to include measurement of distal latency (Guideline). Supplementary NCS: Comparison of the median motor nerve distal latency (second lumbrical) to the ulnar motor nerve distal latency (second interossei) (Technique J), median motor terminal latency index (Technique I), median motor nerve conduction between wrist and palm (Technique E), median motor nerve compound muscle action potential (CMAP) wrist to palm amplitude ratio to detect conduction block, median sensory nerve action potential (SNAP) wrist to palm amplitude ratio to detect conduction block, short segment (1 cm) incremental median sensory nerve conduction across the carpal tunnel (Option). Perform needle electromyography of a sample of muscles innervated by the C5-T1 spinal roots, including a thenar muscle innervated by the median nerve of the symptomatic limb (Option). Comparison of pooled sensitivities and specificities of electrodiagnostic techniques to diagnose CTS [34] For each electrodiagnostic technique to summarize results across studies, sensitivities were pooled from individual studies by calculating a weighted average. In calculating the weighted average, studies enrolling more patients received more weight than studies enrolling fewer patients. Specificities were similarly pooled by calculating the weighted average. Technique A. Median sensory and mixed nerve conduction: wrist and palm segment compared with forearm or digit segment: sensitivity 0.85; specificity 0.98 Technique B. Comparison of median and ulnar sensory conduction between wrist and ring finger: sensitivity 0.85; specificity 0.97 Technique C. Median sensory and mixed nerve conduction between wrist and palm: sensitivity 0.74; specificity 0.97 Technique D. Comparison of median and ulnar mixed nerve conduction between wrist and palm: sensitivity 0.71; specificity 0.97 Technique E. Median motor nerve conduction between wrist and palm: sensitivity 0.69; specificity 0.98 Technique F. Comparison of median and radial sensory conduction between wrist and thumb: sensitivity 0.65; specificity 0.99 Technique G. Median sensory nerve conduction between wrist and digit: sensitivity 0.65; specificity 0.98 Technique H. Median motor nerve distal latency: sensitivity 0.63; specificity 0.98 Technique I. Median motor nerve terminal latency index: sensitivity 0.62; specificity 0.94 Technique J. Comparison of median motor nerve distal latency (second lumbrical) to the ulnar motor nerve distal latency (second interossei): sensitivity 0.56; specificity 0.98 Technique K. Sympathetic skin response: sensitivity 0.04; specificity 0.5.

What is the presentation of cubital tunnel syndrome?

Patients who are affected with cubital tunnel syndrome often experience numbness and tingling along the little finger and ulnar half of the ring finger, usually accompanied by weakness of grip. This frequently occurs when the patient rests upon or flexes the elbow. Patients may experience pain and tenderness at the level of the cubital tunnel, which may radiate proximally or distally. Symptoms vary from a vague discomfort to hypersensitivity at the elbow, and they may be intermittent at first and then become more constant. Nocturnal symptoms, especially emedicine.medscape.com/article/1231663‐overview#showall 2/12 9/3/12 Cubital Tunnel Syndrome with elbow flexion, may be quite disturbing. Patients with chronic ulnar neuropathy may complain of loss of grip and pinch strength and loss of fine dexterity. Rarely, patients with severe prolonged compression present with intrinsic muscle wasting and clawing or abduction of the little finger. The physical examination should include the following steps: Check elbow ROM and examine the carrying angle; examine for areas of tenderness or ulnar nerve subluxation. A positive Tinel sign finding is typically present in cubital tunnel syndrome; however, up to 24% of the asymptomatic population present with a positive Tinel sign finding. The elbow flexion test is the best diagnostic test for cubital tunnel syndrome.[27, 28] The test involves the patient flexing the elbow past 90°, supinating the forearm, and extending the wrist. Results are positive if discomfort is reproduced or paresthesia occurs within 60 seconds. The addition of shoulder abduction may enhance the diagnostic capacity of this test. Recently, a small study reported on 25 patients with cubital tunnel syndrome examined before and after surgery with 10 seconds each of the elbow flexion test and the shoulder internal rotation test. The results suggest that the 10second shoulder internal rotation test appears specific to cubital tunnel syndrome and may be more sensitive for testing cubital tunnel syndrome than the 10second elbow flexion test. In this test, a patient's upper extremity was kept at 90° of shoulder abduction, maximal internal rotation, and 10° of flexion, with the elbow flexed 90°, the wrist in neutral, and the fingers extended. A test was considered positive if any symptom attributed to cubital tunnel syndrome appeared within 10 seconds.[29] Palpate the cubital tunnel region to exclude mass lesions. Examine for intrinsic muscle weakness. Examine for clawing or abduction of the small finger with extension (Wartenberg sign). Assess ability to cross the index and middle fingers. Check for a Froment sign with key pinch. Check grip and pinch strength. Check vibratory perception and light touch with SemmesWeinstein monofilaments. This is more important than static and moving 2point discrimination tests, which reflect innervation density, as the initial changes in nerve compression affect threshold. Check 2point discrimination. Evaluate sensation, especially the area on the ulnar dorsum of the hand supplied by the dorsal ulnar sensory nerve; hypesthesia in this area suggests a lesion proximal to the Guyon canal. Exclude other causes of dysesthesias and weakness along the C8T1 distribution, such as cervical disk disease or arthritis; thoracic outlet syndrome; or ulnar nerve impingement at the Guyon canal. Differential diagnoses include the following: Systemic Diabetes, renal disease, multiple myeloma, amyloidosis, chronic alcoholism, malnutrition, leprosy, others Compression Extrinsic - Postoperative; tourniquet; occupational or recreational activities requiring repetitive flexion or prolonged use of vibrating tools; recurrent trauma; others Intrinsic Supracondylar process, ligament of Struthers, anconeus epitrochlears,[30] medial head of the triceps, arcuate ligament, Osborne ligament, nerve subluxation Valgus ligament instability Elbow injury and deformities Fractures and dislocations; cubitus valgus or varus; trochlear hypoplasia Space occupying lesions Ganglia, tumors, osteophytes, bursae Perineural adhesions Burns and heterotopic bone Arthritic conditions Osteophytes, synovitis Conditions that mimic cubital tunnel Syringomyelia, cervical disc disease, thoracic outlet syndrome, Pancoast tumor, double crush, entrapment of the nerve at the Guyon canal

What is the presentation of MDI?

The diagnosis of multidirectional instability (MDI) is highly clinical. Suggestive history and physical examination findings are the basis of a diagnosis of MDI. Imaging studies, including plain radiographs, magnetic resonance imaging (MRI), and MRI-arthrography, may be of marginal help. Examination under anesthesia (EUA) and arthroscopic findings are highly supportive. History The patient with MDI most often presents with complaints of a generalized painful or sore shoulder, which is usually worse with activity or with certain arm positions. Instability symptoms perceived by the patient, such as dislocation, subluxation, or functional symptoms (eg, catching, locking), are less commonly reported than pain.[11] In fact, some patients may not appreciate or describe any actual sense of instability. Symptoms may follow a roller coaster pattern and may be aggravated by overhead activity, carrying objects at the side, overuse, or injury. These symptoms are relieved by rest and support of the arm. Nocturnal pain is variable. The patient usually denies a history of frank traumatic dislocation but may describe subluxation or looseness, even with activities of daily living (ADL). This history should provoke suspicion of and search for a multidirectional pattern of laxity, particularly if bilateral or posterior. The combination of posterior and inferior laxity is classic, according to Neer and Foster.[12] An athletic history may be contributory.[13] Patients with a predisposition to MDI who are engaged in sports that are stressful to the shoulder girdle, such as swimming, throwing, or racquet sports, may have a difficult time with consistent high activity levels. Perhaps one of the most confusing presentations is that of concomitant impingement. Not uncommonly, a patient with MDI may complain chiefly of pain with overhead use, especially if there is involvement with overhead athletics, such as throwing, volleyball, swimming, or racquet sports. Pain, in this case, may be minimal with the arm at the side. Tibone et al[14] have shown that therapeutic management directed at the diagnosis of impingement and rotator cuff pathology in patients participating in overhead activities may be unsuccessful. Underlying instability always must be considered in those who report a painful shoulder, especially in the younger patient who is involved in vigorous activities above the shoulder. Impingement symptoms (ie, pain with the arm at 90° or more) may be secondary to glenohumeral hypermobility and superior humeral head translation, regardless of acromial arch architecture. Physical examination A notable highlight of MDI on examination is the bilaterality of physical findings. Although active ROM (AROM) may be guarded, there are no passive limits. A good stability examination yields underlying glenohumeral hyperlaxity if adequate relaxation can be achieved. The pathognomonic feature of MDI is demonstration of the sulcus sign—the hallmark of the inferior component of the capsular laxity. Again, with adequate relaxation, a patient examiner demonstrates laxity beyond the normal limits with anterior and posterior testing. Grade may be variable, and anterior and posterior components need not be symmetrical. If the patient is unable to relax, an EUA may be required to demonstrate increased glenohumeral anterior and posterior translation, as well as inferior translation (ie, sulcus sign). More often than not, these findings are symmetrical. Examination of the labrum (eg, labral grind test, superior labrum anterior and posterior lesion [SLAP] test) also may reveal positive findings, with or without true labral anatomic abnormalities. Furthermore, apprehension testing also may be positive, usually in the direction of the chief component of instability. For example, anterior apprehension findings in the external rotation and abducted position may suggest a predominant anterior-inferior MDI pattern, with or without positive relocation, crank, or fulcrum tests.

What is the anatomy medial humeral fracture?

The humerus is a bone in the arm. The distal humeral physis, also called the growth plate, is located between the humeral metaphysis proximally and epiphysis distally. The distal humeral epiphysis is bordered proximally by the physeal growth plate and distally by its articular surface with the ulna and radius. The humeral metaphysis is the growing portion of the humerus that lies between the epiphysis and diaphysis (the shaft or central part of a long bone).[26, 27] The medial condyle of the humerus is the medial column of the distal expansion of the humerus that includes the trochlea, the coronoid fossa, the olecranon fossa, the medial epicondyle, the medial supracondyle ridge, the medial metaphysis, and the groove for the ulnar nerve. Trochlea means pulley. The trochlea is the distal medial articulating end of the humerus, which acts as a pulley for the ulnar trochlear notch to rotate around as the elbow is flexed. The coronoid fossa is the depression on the anterior surface of the medial condyle proximal to the trochlea that accommodates the coronoid process of the ulna. The olecranon fossa is the depression on the posterior surface of the medial condyle proximal to the trochlea, which accommodates the olecranon of the ulna. The medial epicondyle is a prominent palpable process that projects medially from the trochlea and is the point of origin of the pronator teres and common flexor tendon, which includes the flexor carpi radialis, palmaris longus, flexor carpi ulnaris, and flexor digitorum superficialis.[28] The medial supracondyle ridge is a bony ridge that runs proximally on the medial humerus from the medial epicondyle. The capitellum, a rounded ball of bone adjoining the trochlea laterally, is the distal lateral articulating end of the humerus that articulates with the radial head. The lateral epicondyle is a prominent palpable process that projects laterally from the capitellum and is the point of origin of the common extensor tendon. The lateral supracondyle ridge is a bony ridge that runs proximally on the lateral humerus from the lateral epicondyle. The ulnar nerve, which runs in close proximity to the medial epicondyle, often may be palpated as a rounded cord just posterior to this bony prominence. The elbow joint is a hinge-type synovial joint formed where the distal end of the humerus articulates with the proximal ends of the radius and ulna. This is a uniaxial joint with movements of flexion and extension. Normal range of motion is from 0° (full extension) to 135° (full flexion). Most functions of the elbow require motion of 30-130°. Consequently, a 30° extension lag has little functional significance. The normal physiologic carrying angle of the elbow in the anatomic position (full supination and extension) is approximately 165-170° (10-15° of valgus angulation). Flexion is produced by the brachialis (main flexor muscle) and the brachioradialis muscles. In supination, the biceps brachii muscle also flexes this joint; in pronation, the pronator teres assists in flexion. Flexion is limited by the apposition of the anterior surfaces of the forearm and arm, by tension of the posterior arm muscles, and by the collateral ligaments. Extension is produced by the triceps brachii muscle and is assisted by gravity and the anconeus muscle. Extension is limited by impingement of the olecranon of the ulna on the olecranon fossa of the humerus and by tension of the anterior muscles and collateral ligaments. At birth, a single cartilaginous cap covers the distal end of the humerus. During development, 4 separate ossification centers form at different times: The capitellum is first and begins to ossify when the infant is aged 6-12 months. The medial epicondyle is second to form when the child is aged 5-7 years. The third center to ossify is the trochlea when the child is aged 7-12 years. The last center to ossify is the lateral epicondyle when the child is aged 12-14 years. Further complicating this pattern of development, the trochlear ossification center is frequently composed of multiple irregularly sized foci that eventually coalesce into a single structure. The trochlear and capitellar ossification centers eventually fuse. The pattern of ossification at these multiple sites is highly variable.

What is the relevant anatomy for anterior instability?

The shoulder joint is a simple structure that provides complex function. It is the most mobile joint of the body, and it is also the joint that is most frequently dislocated. The gross anatomy consists of 3 main components: musculature, capsule/ligaments, and bone. Capsuloligamentous structures provide the primary stabilization for the joint. The capsule of the shoulder joint extends from the periphery of the glenoid around the articular surface of the proximal humerus. Within this capsule are 3 distinct thickenings that constitute the superior glenohumeral ligament (SGHL), middle glenohumeral ligament (MGHL), and IGHL. The SGHL and MGHL attach proximally at the anterosuperior portion of the glenoid labrum. The proximal attachment of the SGHL has 2 origins, including the one at the apex of the labrum that is joined with the long head of the biceps brachii and a second origin at the base of the coracoid process. Distally, the SGHL attaches just superior to the lesser tuberosity at the edge of the articular surface. The MGHL inserts just medial to the lesser tuberosity. The IGHL is the key stabilizer of the shoulder, preventing anterior glenohumeral instability. The IGHL attaches proximally to the anterior, inferior, and posterior margins of the glenoid labrum. Distally, it attaches to the inferior margin of the anatomic neck of the humerus. Together, the glenohumeral ligaments function to limit lateral rotation of the shoulder. Each of the 3 ligaments is relied upon for stability, depending on the position of the arm. In 1910, Delorme found that the MGHL tightens as the arm is externally rotated or dorsally flexed.[8] If the arm is then abducted, the IGHL becomes the primary stabilizer, with the upper fibers tight at slight abduction and the whole ligament tightening at about 45° of abduction. The secondary stabilizers of the shoulder joint are the surrounding musculature. This consists of the rotator cuff muscles. The supraspinatus, infraspinatus, teres minor, and subscapularis are intimately associated with the capsule. These muscles provide dynamic secondary stabilization. Conservative therapy focuses on strengthening the rotator cuff muscles to prevent recurrent dislocation. The subscapularis is the most important contributor of the rotator cuff muscles to anterior shoulder stability. With the arm adducted, it tightens with external rotation. Cutting it results in 15-20° of increased external rotation. At 45° of abduction, the subscapularis becomes taught over the anterior joint surface and ascends so its inferior margin lies at the inferior margin of the glenoid. External rotation raises it even further and makes it more taught. Finally, at 90° of abduction, the inferior portion of the subscapularis no longer covers the inferior humeral head. It continues to provide anterior stabilization by remaining taught. The glenoid fossa provides a shallow socket in which the humeral head articulates. It is composed of the bony glenoid and the glenoid labrum. The labrum is comparable to the menisci of the knee. It is a fibrocartilaginous structure surrounding the periphery of the glenoid. Like the menisci of the knee, it is flexible but constant; when torn, it generally does not heal. The stability of the glenohumeral joint is greatly increased by the labrum, which provides a 50% increase in the depth of the concavity. The bony concavity measures approximately 2.5 mm and has been measured at 5.0 mm with an intact labrum. In 1923, Bankart claimed that anterior inferior detachment of the labrum from the glenoid was the essential lesion in anterior glenohumeral instability. It has since been proven that this is true in most cases of instability but is not required for instability. The labrum is closely related to the joint capsule and the glenohumeral ligaments. It is anchored to the bony rim of the glenoid and can be attached to the capsular structures as well. Finally, understand the supporting musculature of the scapula that allows such a wide range of motion at the shoulder. A total of 16 muscles move and stabilize the scapula.

What ligament is a is affect with multidirectional instability

a Loose inferior ligament.

What are the signs and symptoms of subacrominal bursitis?

acute burning pain.... loss of motion.... local tenderness on the anterior and superior aspect of the shoulder

What is the presentation for anterior shoulder dislocation?

anterior dislocations usually present with the arm in slight abduction and externally rotated. The humeral head can often be palpated in the front of the shoulder. Internal rotation and adduction are limited. Movement is usually very painful as a result of muscle spasms

What therex produces the Ideal UL/LT ratio?

bilateral ER in neutral.. W

What are the postoperative detail

he elbow is immobilized at 90° of flexion in a well-padded posterior splint. The neurovascular status of the upper limb is monitored closely for the fist 24 hours. At the earliest sign of neurovascular dysfunction, encircling dressing and bandages should be loosened and compartment pressures should be measured.

What is the surgical care for medial neuropathy?

he decision to proceed to carpal tunnel release (CTR) surgery should be driven by the preference of the patient.[42] Surgery is indicated in most patients with moderate-to-severe CTS. According to a Cochrane review in 2008, surgical treatment of carpal tunnel syndrome relieved symptoms significantly better than splinting. A significant proportion of people treated medically eventually required surgery, and the risk of reoperation in surgically treated patients was low. Complications were more common in the surgical arm (RR 1.38, 95% CI, 1.08-1.76).[43] In a 2005 comparison study of open carpal tunnel release with steroid injection, surgery resulted in better symptomatic and neurophysiologic outcome but not grip strength in patients with idiopathic CTS over 20 weeks.[44] In a 2009 randomized multicenter study of patients with CTS without denervation, surgical treatment led to modestly better outcome than multimodality, nonsurgical treatment (including hand therapy and ultrasonography).[45] The American Academy of Orthopaedic Surgeons provides treatment guidelines, including surgical recommendations. Regardless of the specific technique used, surgical treatment of carpal tunnel syndrome should involve complete division of the flexor retinaculum.[39] Indications for surgical decompression Acute CTS due to local trauma (fracture, hematoma, infection) (requires surgical decompression as soon as possible) Mass lesion (eg, nerve tumor, ganglion cyst) Failure to respond to conservative therapy (recommended time ranges from 2-7 weeks[39] to 1 year[42] ) Severe CTS on clinical examination Significant weakness or atrophy of the thenar muscles Persistent numbness and paresthesias in the median sensory territory Severe electrodiagnostic abnormalities with documented axonal loss Decreased compound motor action potential on NCS Denervation in distal median innervated muscles on electromyography Surgery techniques Surgery includes complete resection of the transcarpal ligament by open or endoscopic techniques. Classic open CTR surgery requires a longitudinal incision from the distal wrist crease to the palm, about 5-6 cm in length. Modifications with limited open release surgeries have been described. Endoscopic surgery is done by either single or dual portal techniques, with overall similar success rate than open surgery. According to a Cochrane review in 2004, no strong evidence supports open or endoscopic surgery for CTR, and the decision seems to be guided by surgeon and patient preferences.[46] The overall success rate with endoscopic CTR surgery is reported as 96.5%, with a complication rate of 2.7% and a failure rate of 2.6%.[47] The endoscopic technique has a slightly higher risk of injury to the median nerve. In some reports, patients with endoscopic surgery experience less pain and have earlier return to work and daily activities.[47] Complications Transient paresthesias of the ulnar and median nerves are common. Tenderness of the surgical scar is greater after open surgery and may persist for up to 1 year. Superficial palmar arch injuries, reflex sympathetic dystrophy, and flexor tendon lacerations can occur. Causes of incomplete relief from surgery include incomplete section of flexor retinaculum, multifactorial hand symptoms, and an incorrect preoperative diagnosis.

What is the goal with phase 4 of elbow rehab?

Phase IV Return to activity phase Progressively return to full competition using an interval return-to-throwing program

What supraspinatus therex has the highest EMG activity?

Full can

What is the special test for medial epicondylitis?

Golfer's Elbow Test

What special tests should you perform with a traction injury?

Speed test dynamic speed test active compression test

What are the special tests/imaging for subacrominal bursitis?

Speed's test.... Neer's test.... X-ray.... MRI

What are the special tests for bicep rupture?

Speeds. yergason tests. dynamic speeds tests..

What is the pathophysiology of medial neuropathy?

The median nerve is formed by C5-C7 fibers from the lateral cord and C8-T1 fibers from the medial cord of the brachial plexus. Muscular branches of the median nerve innervate most of the forearm flexor muscles and include the anterior interosseus nerve. The palmar cutaneous branch of the median nerve leaves the main trunk proximal to the wrist crease and provides sensation over the thenar eminence. Within the hand, the median nerve carries C8-T1 motor fibers to the abductor pollicis brevis, opponens pollicis, and superficial head of the flexor pollicis brevis muscles (thenar or recurrent motor branch) and the first and second lumbrical muscles. It supplies sensory innervation to the palmar surface of the thumb, and digits 2, 3, and the lateral half of digit 4 (via the common palmar digits nerves 1-3). The median nerve crosses from the distal forearm to the hand through the carpal tunnel. The carpal tunnel is located at the base of the palm, just distal to the distal wrist crease. The floor of the carpal tunnel is formed by the carpal bones that create an arch. The fibrous flexor retinaculum, or transverse carpal ligament (TCL), is the roof of the carpal tunnel on the palmar side. The carpal tunnel is the narrowest at the level of the distal carpal row, at the level of the hook of the hamate bone. Within the carpal tunnel, the median nerve is physiologically flattened in configuration, and this flattening is maximal about 2-2.5 cm distal to the proximal edge of TCL. Along with the median nerve, 9 flexor digitorum tendons (8 tendons of the superficial and deep finger flexors and 1 of the flexor pollicis longus) pass through the carpal tunnel. The TCL is under tension, helps to maintain the carpal arch, and provides a retinacular pulley to the flexor tendons. CTS is caused by increased pressure in the carpal tunnel and on the median nerve. Compression of a peripheral nerve induces marked changes in intraneural microcirculation and nerve fiber structure, impairment of axonal transport, and alterations in vascular permeability, with edema formation and deterioration of nerve function.[1] Ischemia is a more significant factor of nerve fiber damage in acute median nerve compression, whereas in chronic entrapment, mechanical distortion plays a greater role. The pathology of idiopathic CTS is a noninflammatory fibrosis of the subsynovial connective tissue surrounding the flexor tendons. Biochemical studies of surgical specimens suggest that a variety of regulatory molecules may be inducing fibrous and vascular proliferation and that this may be a response to mechanical stresses.[2] In a study of patients with CTS, when the wrist was in neutral position, the mean pressure in the carpal canal was 32 mm Hg versus 2.5 mm Hg in healthy patients.[3] The pressure increased to 94 mm Hg during wrist flexion (healthy patients 32 mm Hg) and 110 mm Hg during wrist extension (healthy patients 30 mm Hg). Carpal tunnel release brought about an immediate and sustained reduction in pressure. In animal experiments, acute and severe compression caused persistent impairment of intraneural microcirculation due to mechanical injury to blood vessels.[4] In rabbits undergoing a graded compression of the tibial nerve, interference with venular flow was observed at a pressure of 20-30 mm Hg, while arteriolar and intrafascicular capillary flow was impaired at about 40-50 mm Hg. At 60-80 mm Hg, no blood flow ceased completely.[4] In early or mild CTS, the median nerve has no morphological changes, and neurologic symptoms are intermittent. Prolonged increased pressure on the nerve results in segmental demyelination. The focal demyelination causes short segment conduction delay or conduction block across the site of entrapment. In more severe cases, wallerian degeneration and denervation of the thenar muscles develops. The peripheral nerves of patients with underlying generalized neuropathies are more susceptible to compression injury, and the condition is associated in up to one third of cases with systemic medical conditions. Most cases of CTS are considered idiopathic. Some patients have an inherited increased susceptibility of the nerve to pressure, and on rare occasions CTS may be familial. The concept of double crush syndrome was introduced in 1973 by Upton and McComas.[5] They proposed that focal compression of the nerve proximally predisposes it to injury at a more distal site along its course through impaired axoplasmic flow. The hypothesis remains of uncertain validity; there is no clear association between the frequency and severity of CTS and level of cervical radiculopathy.[6]

What is the pathophysiology of SLAP lesion?

To accurately classify superior labral (labrum) lesions, one must be aware of normal anatomy, including the many normal variants that are observed. The biceps tendon origin is divided roughly in half between the supraglenoid tubercle and the superior labrum. Vangsness et al demonstrated that, 55% of the time, the labral insertion is entirely or mostly into the posterior labrum.[9] Only a small percentage (approximately 8%) have a predominant anterior insertion. The remainder, approximately 37%, have equal insertions to the anterior and posterior labrum. Cooper et al in their anatomic study noted that the superior portion of the labrum had a distinctly different morphology than the inferior labrum.[10] The superior and anterosuperior portions were found to be loosely attached to the glenoid rim through thin connective tissue that easily stretched. This is similar to that of the meniscus of the knee. In contrast, the inferior labrum has a firm attachment through thick inelastic fibers and appears as a firm immobile extension of the glenoid articular cartilage. The 12-o'clock position was the only location on the glenoid rim where the hyaline articular cartilage extended over the rim of the glenoid. The biceps tendon inserts into the supraglenoid tubercle, which is 5 mm medial to the glenoid rim. This, along with the often meniscoid attachment of the superior labrum, results in a sublabral recess. This should not be mistaken for a type II SLAP (superior labrum, anterior and posterior) lesion. As reported by DePalma, a sublabral recess may be present in up to 50% of individuals older than 20 years.[11] This incidence increases with patient age. More than 95% of the specimens in DePalma's study obtained from patients in the seventh and eighth decades of life were found to have a sublabral recess. The middle glenohumeral ligament can sometimes insert directly into the superior labrum as a large thick cordlike structure (ie, the so-called Buford complex). In these cases, the anterosuperior labrum is absent. The Buford complex is not pathologic and should not be stabilized because to do so would markedly restrict external rotation. A sublabral foramen can be observed where the anterosuperior labrum, from approximately the 1- to 3-o'clock position in a right shoulder, is loosely attached or not attached at all to the glenoid rim. Again, this is not pathologic and should not be stabilized. To do so would result in a significant loss of external rotation. Histologically, the superior labrum is composed of fibrocartilage. This is composed of type II cartilage in a relatively acellular matrix with occasional interspersed elastin fibrils. This is in contrast to the hyaline cartilage of the glenoid and the dense fibrous glenohumeral capsule. Branches of the suprascapular, circumflex scapular, and posterior humeral circumflex supply the labrum. Periosteal and capsular vessels supply the labrum throughout its periphery. No vessels enter the labrum from the underlying bone. In general, the superior and anterosuperior labrum have less vascularity than other portions of the labrum. On a biomechanical level, incompetence of the superior labrum and biceps anchor has been shown to have a deleterious effect on anterior glenohumeral stability. Rodosky et al compared anterior glenohumeral stability in specimens with an intact superior labrum with those with a SLAP lesion in a cadaver study.[12] They demonstrated that the presence of a SLAP lesion decreased the torsional resistance by 11-19%, as compared with the intact shoulder, as it was placed in the abducted and externally rotated position. The inferior glenohumeral ligament was subject to significantly increased strain (increase by >100%) in the presence of a SLAP lesion. Pagnani and Deng et al in another cadaver study demonstrated that a SLAP lesion results in significant increases in both anterior-posterior and superior-inferior translations.[13] At 45° of elevation, a 6-mm increase was noted in anterior translation with the arm in neutral rotation and a 6.3-mm increase in translation in internal rotation occurred. Several other studies have examined the strain changes in the superior labrum and biceps anchor with different positions of the throwing motion. Pradhan et al found that a significant increase in strain in the anterior and posterior portions of the superior labrum only occurs when the arm is in maximum external rotation as found in the late cocking phase. Furthermore, the strain in the posterior portion of the superior labrum was significantly higher than that of the anterior portion.[14] Kuhn et al supported these findings with their study of failure patterns of the biceps superior labral complex.[15] They found that failure was significantly more likely in the late cocking position as compared to the early acceleration position. In the late cocking position, 9 of 10 specimens demonstrated failure of the biceps superior labral complex. In contrast, of the 10 paired specimens that were tested in the early acceleration position, only 2 had failure of the biceps superior labral complex. The load to failure was found to be significantly less in the late cocking position than in the early acceleration position. Of the 5 patients that developed type II SLAP lesions, 4 of these occurred in the late cocking positions. These studies emphasize the important role the biceps superior labral complex likely plays in anterior shoulder stability. An unstable SLAP lesion found in the course of a Bankart repair should be stabilized. The important role of the posterior portion of the superior labrum likely reflects the fact that the biceps tendon attachment is usually posterior-dominant as demonstrated in a study by Vangsness et al.[9] In repairing SLAP lesions, particular attention should be given to ensuring the posterior aspect is well stabilized.

Which of the following study designs falls highest on the Oxford Level of Evidence scale

a. Individual RCT with a narrow confidence interval

Injury to what nerve is most closely associated with medial scapular winging?

a. Long Thoracic nerve Correct: The long thoracic nerve innervates the serratus anterior muscle, vital to keeping the scapula in correct position in relation to the rib cage.

Which of the following thermometers is recommended for use with potential heat stroke?

b. Rectal Correct: The rectal thermometer is truly the gold standard. The only other method that is accurate are the ingestible thermometers, which have other problems associated with them. See Casa et al (2007) for further info.

What structures are being stressed during the wind up

the posterior/middle deltoid

Mechanism of injury for AC sprains, acute and chronic labral tears, as well as dislocations

...

Rehab progression after a SLAP repair and anterior stabilization procedure

...

Special testing for the instability and labral teats. What tests are best at ruling in diagnoses, and which are best at ruling out

...

Typical operative procedure for a UCL reconstruction. Post-op rehab progression after UCL surgery.

...

What is the kissing lesion of the elbow?

"kissing lesion" (articular damage on the posteromedial trochlea caused by the olecranon osteo- phyte) of chondromalacia on the posteromedial trochlea (olecranon fossa)

What are the mechanics of the apprehension test

Described by Rowe and Zarins15 in 1981, "This test can be performed when the patient is either in a standing or a supine position. As the shoulder is moved passively into maximum external rotation in abduction and forward pressure is applied to the posterior aspect of the humeral head, the patient suddenly becomes apprehensive and complains of pain in the shoulder." This test is depicted in Figure 1. In Rowe's series, all 60 patients had a positive apprehension test.

What is the background for Monteggia Fracture?

The eponym Monteggia fracture is most precisely used to refer to a dislocation of the proximal radioulnar joint in association with a forearm fracture. These injuries are relatively uncommon, accounting for less than 5% of all forearm fractures. The ulna fracture is usually clinically and radiographically apparent. Findings associated with the concomitant radial head dislocation are often subtle and can be overlooked. The keys to successful diagnosis of a Monteggia fracture are clinical suspicion and radiographs of the entire forearm and elbow. Properly assessing the nature of this injury in a timely fashion is imperative in order to prevent permanent disability or limb dysfunction. For excellent patient education resources, visit eMedicine's Breaks, Fractures, and Dislocations Center. Also, see eMedicine's patient education articles Broken Arm, Broken Elbow, and Elbow Dislocation. History of the injury In 1814, Giovanni Battista Monteggia of Milan first described this injury as a fracture to the proximal third of the ulna with associated anterior dislocation of the radial head.[1] Interestingly, he described this injury pattern in the pre-Roentgen era based solely on the history of injury and on physical examination findings. However, this particular fracture pattern only accounts for about 60% of these types of injuries. More than 150 years later, in 1967, Bado coined the term Monteggia lesion and classified the injury into the following 4 types (see the images below)[2] : Type I - Fracture of the proximal or middle third of the ulna with anterior dislocation of the radial head Type II - Fracture of the proximal or middle third of the ulna with posterior dislocation of the radial head Type III - Fracture of the ulnar metaphysis with lateral dislocation of the radial head Type IV - Fracture of the proximal or middle third of the ulna and radius with anterior dislocation of the radial head The Bado classification is based on the recognition that the apex of the fracture is in the same direction as the radial head dislocation. Guitton et al performed a retrospective study on the functional and radiologic long-term outcome of open reduction and internal fixation in 11 skeletally mature patients with Bado type 1 Monteggia fractures. Two patients had subsequent surgery for nonunion, and 3 elbows had radiographic signs of arthrosis. The mean arc of elbow flexion increased from 110º (range, 35º-140º) at early follow-up to 120º (range, 40º-150º) at late follow-up. The mean arc of forearm rotation increased from 145º (range, 90º-180º) to 149º (range, 90º-180º). The mean Broberg and Morrey score increased from 89 points (range, 62-100 points) to 94 points (range, 76-100 points), and the median DASH score was 7 points (range, 0-34 points) at long-term follow-up.[3] Nakamura et al evaluated the long-term clinical and radiographic outcomes after open reduction for the treatment of missed Monteggia fracture-dislocations in 22 children (14 boys, 8 girls; age range, 4 y to 15y 11 mo). The postoperative Mayo Elbow Performance Index at follow-up ranged from 65 to 100, with 19 excellent results, 2 good results, 1 fair result, and zero poor results. In 17 patients, the radial head remained in a completely reduced position, and it was subluxated in 5 patients. Osteoarthritic changes were seen at the radiohumeral joint in 4 patients. Radiographically, there were 15 good results, 7 fair results, and zero poor results. A good radiographic result was seen in all patients who underwent open reduction within 3 years after injury or before reaching 12 years of age.[4]

What structures are being involved Deceleration?

"> Eccentric workload of external rotators, elbow extension deceleration by LH of the bicep. > Can lead to injury of the bicep(LH) and stresses posterior GH joint"

What are the signs and symptoms of a anterior shoulder dislocation?

1) There is a sudden onset of severe pain, and often a feeling of the shoulder popping out.. 2) The shoulder will often look obviously different to the other side. usually loosing the smooth, rounded contour...3) Pins and needles, numbness or discoloration.

What exercises should be avoid with a peel-back SLAP injury?

Furthermore, patients with peel-back lesions, such as overhead athletes, should avoid excessive amounts of shoulder ER while the SLAP lesion is healing

What are the signs and symptoms of posteiror instability

Complaints of pain and poorly localized aching... clicking in the posterior aspect of their involved shoulder more then often than actual instabilty symptoms. May carry their arm in an internally rotated position. pain or sense of instability if often elicited when their arm is in the position of forward flexion.adduction. and internal rotation

What is the normal movement of valgus gaping at the elbow?

Less than 1 mm of medial opening should be noted in the normal elbow. Any increased opening or reproduc- tion of the patient's pain with valgus stress is abnormal and may indicate medial ligament injury.

What is the clinicial anatomy of medial epicondylitis?

Osteology: Medial epicondyle of the Humerus Musculature: Wrist Flexor Group - M. pronator teres - M. flexor carpi radialis - M. palmaris longus - M. flexor digitorum superficialis - M. flexor carpi ulnaris All these muscles have the same origin: the medial epicondyle.

What is the relevant anatomy for the proximal humeral fracture?

Osteology The anatomy of the proximal humerus is quite varied. Multiple cadaveric studies have been performed to compare anatomic relationships that are constant among individuals. Unfortunately, few exist. The critical anatomical relationships of the proximal humerus are those of the articular segment to the shaft and the tuberosities. These include retroversion, inclination angle, and translation of the head relative to the shaft, and the relationship of the head to the greater tuberosity. On average, the articular segment is retroverted 30° relative to the forearm. The range is quite large (0-70°) and can vary from one side to the other. Inclination of the articular segment also can vary (from 120-140°). The head segment can lie directly over the medullary canal but often is translated either posteriorly or medially. Therefore, if a prosthetic replacement is placed in the intramedullary canal, a resultant shift in position of the articular segment can occur unless some design feature of the prosthesis allows for a simultaneous shift in the prosthetic head position. Finally, the proper anatomic relationships of the prosthetic head must be reconstructed meticulously to avoid overreducing the tuberosity to the head height.[8] The articular head always lies above the greater tuberosity, but the difference can range from 3-20 mm. The biceps groove at the level of the articular surface has a constant relationship to the version of a prosthetic articular surface in relation to the fins of the prosthetic body. If the anterior fin is placed at the biceps groove, the articular segment will be in 30° of retroversion. If the posterior fin is placed 8 mm posterior to the biceps groove, the same degree of retroversion will be recreated. Injury to the blood supply of the proximal humerus has been implicated in the development of avascular necrosis.[9] The ascending branch of the anterior circumflex humeral artery (artery of Liang) has been demonstrated by Gerber to provide most of the blood flow to the articular segment. If the medial calcar of the humerus is spared by the fracture, the vessel will be spared. Rotator cuff The rotator cuff is the critical structure that must be reconstructed following proximal humerus fracture. The initial fracture pattern, displacement of the fracture fragments, reduction maneuvers, and fixation techniques used to oppose the displacement forces are dependent on the rotator cuff forces that produced the fracture.[10] The supraspinatus attaches to the greater tuberosity at the superior facet and the superior half of the middle facet. Avulsion-type forces from this muscle produce a short transverse fracture of the greater tuberosity that displaces primarily superiorly. Straight abduction helps reduce the fragment, and tension band fixation neutralizes initial displacement forces. If the infraspinatus, which attaches to the entire middle facet of the greater tuberosity, also is involved, the fracture fragment is larger, and the fragment is displaced posterosuperiorly. In addition to a vertical tension band to neutralize displacement forces, horizontal fixation helps neutralize rotational forces from the infraspinatus. The subscapularis inserts onto the lesser tuberosity. These fractures avulse the lesser tuberosity anteromedially. Horizontal fixation best neutralizes these fractures. In 4-part fractures, the tuberosities are displaced, and the supportive structures of the articular segment are removed. Therefore, this fragment tilts superiorly and subsides. If the forces then axially load the shaft against this head segment, it can extrude laterally, disrupting the medial calcar and its blood supply. Neurovascular supply Twenty-one to 36% of proximal humerus fractures are associated with neurovascular injuries. Eight percent result in permanent motor loss. The axillary nerve is the nerve most commonly injured. The fracture pattern most commonly associated with axillary nerve injury is an anterior fracture dislocation with a displaced greater tuberosity. Loss of sensation over the lateral deltoid should alert the examiner to possible axillary nerve injury. Isometric contraction of the deltoid should also be tested. The suprascapular, radial, and musculocutaneous nerves also are at risk. Vascular injuries occur rarely, but 27% of axillary artery injuries may have palpable pulses due to scapular collateral circulation. Associated paresthesias and an enlarging mass must be viewed with caution. Most vascular injuries (84%) occur in patients older than 50 years. Fifty-three percent are associated with brachial plexus injuries.

What is the images for olceranon fracture?

Standard anteroposterior and lateral radiographs of the elbow are sufficient for evaluation of isolated olecranon fractures. Direct supervision of the x-ray process may be necessary to ensure that true anteroposterior and lateral radiographs are obtained. The radiocapitellar view may be helpful for delineation of the radial head and capitellar fractures.

What is follow up for Monteggia fracture?

The patient should be evaluated with the dressing and splint changed at 5-7 days. A posterior long arm cast in 90° flexion should be placed at that time. Follow-up radiography at 2, 4, and 6 weeks is recommended to monitor healing and union of the fracture. After this point, if the patient is reliable and stability is present, the above-described therapy can be continued. If the hardware is causing significant problems, the provider may consider removing it. However, unless infection is present, waiting at least 1 year is recommended.

What are the common complication of tommy john surgery?

the authors noted a high incidence of complications related to the ulnar nerve, including ulnar neuropathy in two patients who then required secondary operations.49

What are the special tests or imaging for a posterior impingement

Posterior impingement sign... relocation test... Test for GIRD.

What is the limited joint volume mechanism

"The limited joint volume mechanism reflects the fact that the normal glenohumeral joint is really a potential space, contains minimal fluid, and has an inherent negative pressure. A sealed joint ensures an increase in this negative pressure with attempted distraction, thus increasing the joint reactive force independent of other muscular forces. Joint puncture by any means, increase in joint fluid secondary to trauma or inflammation, and laxity of the capsule (increasing joint volume) all contribute to the loss of this stabilizing mechanism. "

What is the examination for medial epicondylitis?

- Tenderness to palpation (usually over m. pronator teres and m. flexor carpi radialis) - Local swelling and warmth - Range of motion in the beginning of the disease can be full, but later on there is a possibility of a decreased range of motion[8]

Obrien's Active Compression Test is used to distinguish between which of the following diagnoses? Choose one answer.

. b. Superior labral abnormalities and acromioclavicular abnormalities Correct: The Active Obrien's test is used to distinguish between superior labral abnormalities and acromioclavicular abnormalities. This was described in your weekly readings. See Tennett et al. (2003).

Describe the best special tests for identifying a UCL injury in a pitcher.

...

What are the signs and symptoms of a humeral fracture.

1) Serve arm pain at the time of injury... 2) tenderness swelling and bruising of the arm./..3) later swelling and bruising in the elbow and hand... 4) Visable deformity if the fracture is complete....5) numbness, coldness, paralysis below the fracture involving the forarm or hand pressure on or stretching of blood vessels or nerves.

How long to you immobilize a shoulder instability?

6-8 weeks

What structures are being involved with Follow Through?

> Tension on posterior GH capsule > ER cuff muscle( minimal risk here)

What region of the shoulder capsule prevents inferior subluxations with adducted shoulder

A biomechanical study using a cadaveric shoulder model has confirmed the impor- tance of the anterosuperior capsule in preventing inferior subluxation of the adducted shoulder.

What are the other test for little league elbow syndrome?

Electrophysiologic studies are often performed to localize the area of nerve entrapment in cases of radial tunnel syndrome and pronator syndrome.[3, 10, 15] The main disadvantage to these studies is a high false- negative rate. Needle electromyography seems to be more useful than nerve conduction studies in localizing the lesions.

What is the presentation for lateral humeral condyle fracture?

Children usually present with a history of a fall onto an extended arm. Patients present with pain and associated elbow swelling. Physical examination demonstrates a swollen elbow, pain greatest over the lateral condyle, and refusal of the patient to actively move the elbow. Occasionally, crepitus is present in an unstable fracture pattern. Significant deformity may indicate an elbow dislocation.

What are the contraindications for clavicle fracture?

Clavicle fractures are most commonly the result of a fall directly onto the shoulder. Generally, nondisplaced or minimally displaced clavicle fractures heal with nonoperative management. Many displaced midshaft clavicle fractures can be successfully treated nonoperatively as well. These fractures tend to heal with varying degrees of cosmetic deformity from the fracture ends, typically causing a bump under the skin at the fracture site. Operative repair of these injuries is not indicated if the goal of surgery is cosmesis instead of fracture healing. Often, the surgical scar is equally, if not more, displeasing cosmetically for the patient. Another contraindication to surgical fixation of clavicle fractures is the presence of infection, either localized in the region of the fracture or systemic. Unless the injury is open, fixing these fractures on an emergent or urgent basis is not necessary. Thus, surgery should be performed at an optimal time based on surgical risk.

What is the differential diagnoses?

Elbow and Forearm Overuse Injuries Little League Elbow Syndrome Medial Condylar Fracture of the Elbow Medial Epicondylitis

What is the postoperative detail for lateral humeral condyle fracture?

Elevate the operated extremity above the heart level, and apply ice to the splint at the fracture site. A sling may be applied for support and comfort.

What supraspinatus therex places the most stress on the tendon?

Empty can

What are the intraoperative olecranon fracture?

Excision of fragment and triceps advancement Excision of the fracture fragment and reattachment of the triceps tendon may be indicated in a select group of elderly patients with osteoporotic bone in whom the olecranon fracture involves less than 50% of the joint surface or when the fragment is too small or comminuted for successful internal fixation.[8, 12, 14, 15] Integrity of the collateral ligaments, intraosseous membrane, and distal radioulnar joint must be established before considering excision;; otherwise, instability can result. The triceps is reattached with nonabsorbable sutures that are passed through drill holes in the proximal ulna. The drill holes are placed such that the triceps will insert just off the articular margin of the olecranon articular surface, in essence extending the articular margin. Weakening of the extensor mechanism is a drawback of excision and triceps advancement. However, comparison of the isometric strength of patients treated by excision with those who had internal fixation showed no differences. Excision and triceps advancement may be followed by immediate motion if the suture repair of the triceps is secure. Tension band wiring A tension band wire is the most common fixation technique for simple fractures. The goal is to convert the extensor force of the triceps to a dynamic compression force along the articular surface. In this technique, a direct straight posterior incision is used with the patient supine with the arm across the chest or, occasionally, in the prone position.[12, 16, 17] Two smooth K-wires are placed through the triceps tendon into the olecranon with great care made to bury the wires beneath the tendon and firmly impact them into the bone. Otherwise, the wires certainly will migrate posteriorly and can become an irritant or possibly a source of infection. The key is to place the tension band wire as dorsally as possible on the surface of the olecranon, as shown in the image below. K-wires help to hold the tension band wire in place as it loops around the tip of the olecranon. A drill hole through emedicine.medscape.com/article/1231557-‐‑treatment#showall 2/7 9/3/12 Olecranon Fractures Treatment & Management the ulna at 1-2 cm distal to the articular surface provides the distal fixation point. Placing 2 knots in the 18-gauge wire results in more rigid fixation than a single knot and provides symmetrical tension at the fracture site. Plate fixation Plate fixation is most commonly recommended for comminuted fractures for which tension band wire fixation is not feasible. It also is indicated for fractures that involve the coronoid process and for those associated with Monteggia fracture dislocations of the elbow. Some authors have used one-third tubular, dynamic compression, or pelvic reconstruction plates for comminuted fractures. The proximal end of the one-third tubular plate can be modified to make a hook-plate that provides additional fixation for small fragments. The subcutaneous location of the hardware raises concerns about prominence necessitating subsequent removal of fixation.[18, 5] Do not narrow the olecranon-to-coronoid distance. Restore it to within a few millimeters of the correct anatomic distance. Bone grafting sometimes is necessary. The last hole in the plate where it has been bent to make a hook provides a good location for an intramedullary screw. Intramedullary screw fixation Use of a single large-diameter cancellous screw for repair of olecranon fractures has been advocated for a long time. The Rush brothers wrote that intramedullary insertion of a Steinmann pin was the beginning of the Rush pin technique of fracture fixation in 1936. They claimed this to be the first American case of intramedullary pinning. They found that Steinmann pins were difficult to use and designed their own pins. When 6.5-mm AO-ASIF screws became available, they were used more commonly.[11] In the frontal plane, 4° of valgus angulation exists between the ulnar shaft with respect to the sigmoid notch, as is shown in the image below. When intramedullary screws are used, take care to properly place the screw along the intramedullary shaft axis to avoid displacement of the fracture. With the advent of cannulated screws, it is much easier to correctly place the screw in the medullary canal of the ulna simultaneously, accommodating for the bow in the ulna and achieving anatomic reduction. The 7.3-mm cannulated AO screws are the most secure. Also, screw fixation is probably the easiest technique.Biomechanically, screw fixation does not provide as secure a fixation as tension-band wiring. However, by adding a tension band around the screw, excellent fixation can be obtained. The most secure technique is placement of a large-diameter cannulated screw with a braided cable, as shown in the images below. A 1.6-mm cable is adequate and much stronger than an 18-gauge wire.

What are the postoperative for Monteggia fracture?

Follow-up appointments are scheduled for wound checks and suture removal based on the nature of the soft-tissue injury and operative treatment. If rigid fixation is achieved and the radiocapitellar joint remains stable, the patient is referred to begin range-of-motion exercises under the close supervision of a qualified physical therapist. Patients with stable injuries may be placed in range-of-motion braces for 6-8 weeks. Unstable injuries should remain in cast immobilization until stability is achieved at 4-6 weeks. Significant concern exists regarding loss of elbow motion with prolonged immobilization. Chronic radiocapitellar instability is unusual and may be addressed surgically

What is the history of medial humeral fracture?

Fracture of the medial condyle of the humerus is a rare injury. Isolated case reports appear in the literature. Although the medial condyle fracture has been described in the literature since the early 1800s, some controversy exists as to whether these were descriptions of true medial condyle fractures or whether these were really descriptions of more common medial epicondyle fractures. Studies have reported greater numbers of medial condyle fractures in the literature; however, the overall incidence of these fractures remains quite low. Of all elbow fractures in children, medial condyle fractures are reported to account for less than 1%.[1, 2, 3, 4, 5, 6, 7, 8, 9, 13, 14] In 1964, Milch proposed the first classification system for unicondylar humerus fractures.[15] The Milch system is based on the location of the fracture line in the distal humeral epiphysis. Milch first described an avulsion fracture due to a transverse valgus force. He then described a classification system for 2 types of fracture caused by longitudinal forces. A Milch type I fracture splits the trochlear groove, leaving the lateral trochlear ridge intact. A Milch type II fracture splits the capitotrochlear sulcus such that the lateral trochlear ridge is part of the fracture fragment (see image below for a depiction of the two types). A type II fracture is inherently unstable and is called a fracture dislocation.[4] The avulsion and type I fractures can be treated open or closed; however, more complex type II fractures should be treated only with open reduction and internal fixation (ORIF) In 1965, Kilfoyle combined his own experience with 5 colleagues to collect a total of 11 examples of medial condyle fracture and separated them into 3 types of injury, as depicted in the image below.[16] Type I involves a greenstick fracture or crush of the medial condyle metaphysis down to but not including the physis. He also stated that these may actually be incomplete supracondyle or intracondyle fractures. Type 2 involves a fracture through the physeal plate and epiphysis without displacement or rotation. Type 3 is similar to type 2 but with moderate-to-severe displacement and rotation of the fracture fragment. In 1818, Granger reported the first unequivocal description of a medial epicondyle fracture. Granger described a fracture that resolved rapidly and left little functional deficit. In the early 1900s, several authors recognized that the fracture was often associated with elbow dislocation and that the avulsed fragment could become entrapped within the joint. In 1950, Smith dispelled many of the complications previously attributed to medial epicondyle fractures. Smith refuted that medial epicondyle fractures were associated with growth disturbance, pain and disability, weak elbow flexion, or ulnar nerve dysfunction and went on to prove his theories in his classic study. He concluded that fractures involving the medial epicondyle were relatively benign and were not associated with significant functional deficit. Farsetti et al confirmed Smith's conclusions.[17] Even in 42 patients with isolated fractures of the medial epicondyle with displacement of 5-15 mm, no significant difference was found between those treated with ORIF and those treated nonsurgically. No universally accepted system exists for classification of medial epicondyle fractures.

What happen to the elbow carrying angle when you flex are elbow?

It should be noted that, as the elbow is flexed, the carrying angle progresses into more of a varus alignment, making eval- uation of the carrying angle difficult in the setting of elbow contracture.62, 98

What is the management of posterior impingement.

Non surgical intervention for internal impingement that are recommended in the literature is rest. ice, and NSAIDS. However, physical therapy is the cornerstone of non-surgical interventiong and should always be extensively tried before considering surgical interventions. Rehab should be focus on restoring a total 180 arc of motion, restoring normal GH and Scapulothoracic mechanics. as well as education with pitch counts and form.

What are the management of Olecranon Bursitis?

PRICE, and identification and removal of the cause

What is the goal with phase 3 of elbow rehab?

Phase III Advanced strengthening phase Gradually increase strength, power, endurance, and neuromuscular control. Prepare for gradual return to sport

What is the MOI for radial head fracture?

Radial head fractures are most often caused by a fall on an outstretched hand (FOOSH). 20% of the acute elbow injuries that occur are radial head fractures, and are seen more frequently in women than men between the ages of 30 and 40 years old. Approximately 10% of elbow dislocations result in a chipped or fractured radial head that occurs as the dislocation of the humerus and radius are reduced. [1] The radial head is intrarticular and acts primarily as a spacer in the elbow. Due to the lack of blood supply to that area, often fractures result in avascular necrosis or potential nonunion.

What therex would you use for subscapularis?

SL IR

What is the epidemiology of shoulder dislocation?

Shoulder dislocations account for almost 50% of all joint dislocations.

What are the images of ulnar neuropathy?

Ultrasonographic examination of peripheral nerves may be used to support the clinical and electrophysiologic diagnosis in a compressive neuropathy. It may also be helpful in identifying specific compressive etiologies of nerve injury (tumors, cysts, etc) and in visualizing structural nerve changes. Advantages of ultrasonography include the following: Unlike CT or MRI, ultrasonography provides real time evaluation of nerve displacement/compression during movements of adjacent joints. Ultrasonography is noninvasive, cheap, portable, and well-tolerated by patients. Ultrasonography is readily available (however, technicians who are experienced in peripheral nerve ultrasonography may not be readily available). The peripheral nerve can be followed for much of its course in an extremity.[44, 45, 46, 47, 48] The ultrasonography finding that seems to be the most useful is a change in the diameter of a nerve at the site of compression. Just proximal to the site of compression, swelling of the nerve can often be seen. One small study hypothesized that using a ratio of the cross-sectional nerve area at the site of maximal enlargement and at an uninvolved site could improve diagnostic accuracy. Using this ratio did not add any diagnostic accuracy to simply looking for the point of maximal swelling; the ratio did, however, help distinguish compressive neuropathies from other systemic processes associated with diffuse nerve enlargement (eg, diabetes, polyneuropathy).[49] One specific area in which ultrasonography may be useful is the evaluation of traumatic peripheral nerve injuries. In one interesting study, 20 fresh cadaver arms were disarticulated, and the median, ulnar, or radial nerves were randomly transected in 0-2 locations per arm. Sham incisions were performed throughout the extremity. The peripheral nerves were then systematically scanned by ultrasonographers who were blinded to the sites of transection. High-resolution ultrasonography was able to identify transected nerves with 89% sensitivity and 95% specificity. The diagnostic accuracy improved throughout the study; with the first 10 arms, the ultrasonographer correctly identified the transection in 77% of cases. With the final 10 arms, the accuracy was 100%. This suggests that the experience of the ultrasonographer plays a vital role in the use of ultrasonography in peripheral nerve injury. The study suggests that ultrasonography may be useful both in prognostication of nerve injury when an experienced ultrasonographer can assess for a partial versus complete nerve injury, and/or in localizing a nerve transection for possible surgical repair.[50, 51] Magnetic resonance imaging MRI is being increasingly used in the evaluation of peripheral neuropathies, including ulnar neuropathy. In most patients, history, physical examination, and electrophysiologic testing are sufficient to make the diagnosis of ulnar neuropathy. However, there may be a subgroup of patients with inconclusive findings on the standard evaluation in whom MRI may be beneficial. On MRI, normal nerves appear as smooth, round, or ovoid structures that are isointense to surrounding muscles on T1-weighted sequences. There is often a rim of hyperintense signal on T1. On T2-weighted images, the nerve is normally isointense to slightly hyperintense with respect to surrounding muscle. Normal nerves do not enhance after administration of gadolinium. Possible changes that could be seen in neuropathies include increased signal intensity within the nerve on T2-weighted sequences. Neurogenic muscle edema can be seen as early as 24-48 hours after denervation, and STIR sequences are particularly sensitive for that. This is to be contrasted with electrophysiologic testing, in which changes after denervation are not seen for 1-3 weeks. After months of denervation, fatty muscle atrophy is seen. Changes in the surrounding structures that may be related to the neuropathy in question, such as osteoarthritis, synovitis, or tumors, can be seen with MRI as well.[52] Several small studies exist that attempt to address the use of MRI in the diagnostic evaluation of ulnar neuropathy. In one study by Vucic et al, 52 patients were identified who met clinical criteria for ulnar neuropathy, based on either sensory symptoms or motor weakness in the distribution of the ulnar nerve. All of these patients underwent electrophysiologic testing. In 63%, the electrophysiologic studies were diagnostic of an ulnar neuropathy at a specific location, commonly at the elbow. In 37%, the EP studies were nonlocalizing based on the criteria of the American Association of Electrodiagnostic Medicine. All patients subsequently underwent MRI scanning as well, which revealed abnormalities in 90% of patients. In the subgroup of patients who had diagnostic EP studies, 94% had an abnormal MRI; in those who had nondiagnostic EP studies, 84% had an abnormal MRI. The authors' conclusion was that MRI was "more sensitive" than neurophysiologic testing, and that the sensitivity of MRI does not change, regardless of the EP results.[53] Another study by Andreisek et al looked at 51 patients with clinically evident neuropathies in either the radial, median, or ulnar nerves who were referred to their center for MRI scans of an upper extremity. This study was designed to assess the impact of the MRI results on clinical decision making and patient management. In summary, this study found only a weak/moderate correlation between MRI results and clinical findings, which the authors felt was not surprising given that clinical findings imply physiologic dysfunction of the nerves, whereas MRI findings can evaluate nerve morphology alone. The authors reported that the greatest use of MRI in this study seemed to be in the patients in whom the etiology of their symptoms was unclear; in these cases, the MRI scan was said to have found the etiology of the symptoms in 93% of cases. This resulted in a moderate to major impact on treatment in 84% of patients in this subgroup.[54] Despite the seemingly positive results of these 2 studies, some caveats should be applied. Firstly, imaging criteria for diagnosing neuropathy on MRI scans are not well-defined. Furthermore, the clinical significance of certain MRI findings has been called into question. A study by Husarik et al took 60 asymptomatic patients and performed MRI scans of their elbows. In these healthy volunteers, 60% had increased signal intensity of their ulnar nerves without accompanying changes in the medial or radial nerves. This study suggests that an increase in signal intensity should not be used as the only criterion when evaluating for possible ulnar neuropathy.[55] The role of MRI in the evaluation of ulnar and other peripheral neuropathies continues to evolve. At this point, it seems safe to conclude that MRI may be a useful adjunct in select cases, either when a specific compressive lesion such as a mass is suspected, or when a patient with the clinical syndrome of ulnar neuropathy has nondiagnostic electrophysiologic tests. To improve diagnostic accuracy, further research is required to develop standardized criteria to make the diagnosis of ulnar neuropathy on MRI.

What is the Management of anterior instability.

Treatment consists of a period of shoulder immobilization. Subsequent physical therapy reestablishes range of motion and strengthenings the rotator cuff.

What are the preoperative olecranon fracture?

When determining the appropriate surgical approach, consider patient age, health, bone quality, fracture pattern, and ligamentous stability.

A 50 year old recreational tennis player presents to you complaining of an insidious onset of knee pain over 2-3 months. He reports that he may have "tweaked" his knee 3-4 months ago, but isn't sure. He brings in his MRI (see below). What is the most likely cause of his pain

b. Baker's Cyst Correct: The Baker's cyst is evident posteriorly. For more about diagnostic imaging of the knee, review Sanders & Miller (2005)

Which ligament is the primary restraint to inferior translation of the humerus at 90 degrees of gleno-humeral abduction?

b. Inferior glenohumeral ligament Correct! The posterior band of the Inferior glenohumeral ligament is the primary restraint when the arm is in external rotation, the anterior band when the arm is in internal rotation. When the arm is at zero degrees of elevation, the superior glenohumeral, and coracohumeral ligaments are the primary restraints to inferior humeral translation

During the first 4-6 weeks of training in previously untrained individuals, increases in strength are primarily due to

b. Neural adapations Correct: improved neural control accounts for initial improvement in strength

While covering a wrestling match, you witness an athlete catch an elbow in the jaw. The athletes comes to the medical bench with his tooth in hand. You elect to send the individual to a dentist. What will optimize his chances for saving his tooth?

b. Rinse the tooth and attempt re-implantation. If unsuccessful, place in saline. Incorrect: You never scrub a tooth

What are the prognosis with lateral humeral condyle fracture?

With prompt and appropriate treatment and a satisfactory reduction, good results may be expected with full elbow ROM.

What is the common biomechanics that lead to elbow injury?

"Valgus extension overload syndrome," the mechanism behind many of the above mentioned injuries, is a phenomenon presenting with the combination of high valgus and extension tensile forces to the medial stabilizers in addition to compressive forces laterally.

What are the various surgeries performed for shoulder instability, and what factors contribute to deciding on the best one?

...

What is the pathology and MOI of anterior instability?

Anterior shoulder instability in an injury to the shoulder so that the humerus is displaced from its normal position in the center of the socket and the joint surface no longer touch each other. It often occurs due to a direct blow to the shoulder or backward force on an extended arm or elbow, a repetitive throwing or swimming motion or a congenital abnormailty. It could also be the end result of a severe shoulder sprain of from powerful muscle twisting in a violent muscle contraction.

what is the pathology/treatment/surgery for Medial Epicondyle Apophysitis injury?

Description Repetitive submaximal injury and apophyseal fragmentation or acute avulsion of medial epicondyle apophysis (Saltar-Harris type I fracture) Treatment options Non-operative rehabilitation, open reduction with internal fixation Author's preference Non operative rehab unless avulsion with greater than 5 mm displacement, then ORIF

what is the pathology/treatment/surgery for Loose Bodies

Description Loose bone fragments in and around joint space Treatment options Open surgical removal, arthroscopic excision Author's preference Arthroscopic excision

What are the imaging studies for MDI?

Most often, plain radiographs are negative. Findings of an osseous glenoid rim fracture or a Hill-Sachs humeral head impression defect are usually not seen unless concomitant traumatic instability exists. Results of noncontrast MRI are the same as described for plain radiographs—that is, benign and negative, unless MRI is performed with contrast (gadolinium).[15, 16] Typically, MRI-arthrography may demonstrate blunting of the labrum, diffuse capsular laxity, and increased capsular volume. Labral and capsular tears, such as those seen with traumatic instability, are unusual in classic multidirectional instability (MDI), and rotator cuff tears and SLAP lesions are only rarely seen in association with MDI of the shoulder.

What is the management of posterior instability.

Operative stabilization should be considered in patients who fail to return to activity or continue to experience pain or instabilty are psychologically stable, and failed a trail of physical therapy. The success of surgical intervention depends on the ability to correct the underlying impairments. Therefore, operativve treatment my involved more than one procedure to address the factors contributing to the instability. Factors that influence the choice of operative intervention include volitional instabilty, injury to the shoulder, degrees of instability, structural abnormalites, and direction of instabilities.

What is the management of physical therapy for lateral epicondylitis?

Physical therapy interventions including elbow joint mobilization with movement combined with exercise has been shown to have better results than corticosteroid injection at 6 weeks and to wait and see at 6 weeks but not 52 weeks. Recent research regarding cervicothoracic joint mobilization in conjunction with local treatment for lateral epicondylalgia has shown improvements in strength, pain, and tolerance to activity compared to local treatment alone. [17] Physical therapy management including only the use of ultrasound, massage, and exercise has been shown to be no better than a "wait and see" treatment method. [18] Activity modification, when possible, can help prevent recurrent episodes of lateral epicondylalgia, as well as use of a counterforce brace as needed.

What is the frequency of proximal humeral fracture?

Proximal humerus fractures have been estimated conservatively to account for 5% of all fractures. These fractures occur primarily in older patients, many of whom are osteoporotic. Like hip fractures, proximal humerus fractures are a major cause of morbidity in the elderly population. As the population base ages, the incidence of these fractures will continue to increase.

What images do you get for anterior instability?

Radiologic study of the dislocated or subluxed shoulder should include a minimum of 3 views: true anteroposterior (AP), scapular Y, and axillary. This combination of views provides the best evaluation of the bony structures. Although frequently only soft-tissue injury is present, bony pathology is present in 55% of traumatic dislocations. True AP view: Also known as the Grashey view, this view is obtained by placing the posterior surface of the scapula flat onto the radiography film. This results in a view that is 45° oblique to traditional shoulder AP radiography. A successful exposure demonstrates the glenohumeral joint space, superoinferior head subluxation, joint congruity, joint degeneration, and other articular abnormalities. Disadvantages to this view exist, including anterior and posterior glenoid overlap, which can obscure Bankart lesions. Increased soft-tissue overlap compared with an AP view lessens the quality of bony detail. Scapular Y view: This view is obtained by aiming the x-ray beam longitudinally down the axis of the scapular spine. The humeral head lies directly over the glenoid fossa. The Y shape is formed by the projection of the acromion, scapular body, and coracoid from the longitudinal axis. This view may be adequate for evaluating dislocations, but it should never replace the axillary view, which is the most sensitive for detecting subluxations. Like the true AP view, this view is a poor choice for evaluating glenoid rim fractures. Axillary view: The axillary lateral view has had many variations. The original by Lawrence, described in 1915, was performed with the patient supine, arm abducted to 90°, and the x-ray beam aimed inferior to superior with 15-30° of medial angulation, depending on the amount of abduction. The resulting radiograph allows for detection of AP subluxation/dislocation and anterior or posterior glenoid rim fractures. The West Point view is one variation of the axillary lateral view. It places the patient prone with the arm abducted to 90° and hanging over the edge of the table. The x-ray beam is directed 25° medially and anteriorly. This position improves visualization of the anteroinferior glenoid rim. West Point axillary views are the most sensitive for finding osseus glenoid fractures.[9] Internal and external rotation views: These views provide oblique visualization of the shoulder joint, with the humeral head overlapping the glenoid rim. The advantage of these views is the excellent osseous detail of the scapula, clavicle, upper ribs, and soft tissues. The high-quality bony detail is the result of the low density of the surrounding soft tissue. Internal rotation of the arm in the AP view projects the lesser tuberosity medially and the posterolateral aspect laterally, providing a good view of Hill-Sachs lesions. These views are of little value in detecting anterior or posterior dislocation/subluxation. Stryker-Notch view: This view was developed to allow visualization of Hill-Sachs lesions. It is obtained with the patient supine. The hand is placed on top of the head with the elbow flexed. The x-ray beam is directed anterior to posterior with a 10° cephalic angulation. It provides good detail of the posterolateral margin of the humeral head. Arthrography has become obsolete since the advent of computed tomography scanning. It is no longer indicated in shoulder dislocation, although a couple of studies have shown 100% sensitivity in detecting capsulolabral pathology with double-contrast computed arthrotomography. MRI is the imaging modality of choice for soft-tissue injury for most authors. It has been shown to be 91% sensitive in detecting capsulolabral injury in the early postdislocation period. Further from the injury, MRI and arthrotomography have been up to 96% sensitive and provide a better depiction of the IGHL than does computed arthrotomography. Keep in mind these adjunct studies are necessary only in a minority of patients.

What structures are being stressed during the extension moment at the elbow?

The com- bination of large valgus loads with rapid elbow extension produces tensile stress along the medial compartment re- straints (ulnar collateral ligament, flexor-pronator mass, medial epicondyle apophysis, and ulnar nerve) and shear stress in the posterior compartment (posteromedial tip of the olecranon and trochlea/olecranon fossa), and com- pression stress is produced laterally (radial head and capitellum).

What are the follow up details for coronoid fracture?

The elbow is immobilized for about a week, and then a protected mobilization program in a hinged orthosis is initiated, which prevents varus-valgus stresses on the elbow. Brace use is continued for approximately 4-6 weeks to allow the ligaments to heal. Prophylaxis against heterotopic ossification is initiated the day after surgery. The authors prefer to use indomethacin (75 mg PO) for 3 weeks following the surgery.

What is the frequency cubital tunnel syndrome?

The elbow is the most common site of compression of the ulnar nerve. Cubital tunnel syndrome is the second most common compressive neuropathy (after carpal tunnel syndrome). Cubital tunnel syndrome affects men 38 times as often as women.

What is the anatomy of the bicep attachment to the labrum?

The labrum is attached to the lateral portion of the biceps anchor superiorly. Additionally, approxi- mately 50% of the fibers of the long head of the biceps brachii originate from the superior labrum and the remaining fibers originate from the supraglenoid tubercle of the glenoid.10 The fibers of the biceps tendon blend with the superior labrum and continue posteriorly to become a periarticular fiber bundle, making up the bulk of the labrum.

What are the S&S of Olecranon Bursitis?

The olecranon process of the elbow looks like a "goose-egg" o Pain during the extension and flexion portions of the elbow range of motion

What is the anatomy of the superior labrum?

The superior labrum is rather loose, mobile, and has a ''meniscal'' aspect, while the inferior labrum appears rounded and more tightly attached to the glenoid rim. Histo- logically, the attachment of the labrum to the glenoid rim consists of loose connective fibers above the equator of the glenoid while the inferior portion of the labral attachment is fixed by inelastic fibrous tissue.10

What is the physical therapy management for posterior elbow dislocation?

While nonsurgical treatment approaches to PED can vary depending on the level of tissue involvement, there are key elements to consider throughout the clinical decision-making process. PED can occur on a continuum of severity; therefore, the treatment must be diverse as well. Treatment can vary from aggressive immediate AROM to traditional plaster immobilization for several days.[6][15] If a fracture occurs secondary to dislocation, intra-articular bone fragments and fracture position may dictate treatment.[11] Closed or nonsurgical reduction by a physical therapist is only performed if there are no associated fractures.[5][7][11] Uhl et al. described one technique for reduction: the patient hangs their affected arm over the back of a chair as the clinician tractions the ulna in a downward direction.[1] After reduction of the joint, instability is evaluated. A splint should be applied and the patient should be referred for radiographs if the joint subluxes or dislocates while assessing instability.[7] If left untreated (unreduced) patients may develop soft tissuecontracturesandlocalizedosteoporosis.[9] Thefollowingclinicaldecision-makingalgorithmfor immobilization and surgical options can be used following acute dislocations Generally following reduction the patient is placed in a posterior splint at 45-90o of elbow flexion[1][3][5][16] for three days to three weeks.[3][6][7][17][18][19] Evidence reveals detrimental effects of prolonged immobilization including flexion contractures, enhanced perception of pain, and increased duration of disability, all of which prolong the rehabilitation process.[1][5][15][20][21] Throughout the immobilization phase, wrist and shoulder function should be maintained through ROM and strengthening exercises.[6][17] Inflammation is a common sequela following PED and can be addressed using compression, ice, and effleurage.[5][19][17] When the patient no longer requires immobilization, functional treatment begins with gentle AROM and PROM exercises in a pain-free range targeting the entire UE.[3][4][6][15] Research by Haan et al. shows better outcomes when early rehabilitation is functionally-based and pain-free. Multi-angle isometric activities and Proprioceptive Neuromuscular Facilitation patterns for the elbow help decrease pain, increase ROM, and begin to target strengthening components in the preliminary stages of recovery.[4][5][17] When pain is no longer a barrier to treatment, functional progressive resistance exercises should be implemented to improve total UE muscle strength and endurance.[1][5][17] Although full extension should be a goal of rehabilitation, care must be taken to protect the vulnerable elbow and avoid hyperextension. It is important to be cautious during passive mobilization and ROM. Multiple articles have warned that aggressive PROM (especially into extension) and forceful manipulation may cause myositis ossificans and should be avoided.[1][5][6] Also, Uhl et al. suggested that any valgus stress appliehould be avoided throughout treatment so not to stress the already compromised tissues.[1] Therapeutic goals in the later phase of rehabilitation include attaining full ROM and strength capabilities of the entire affected arm, suppression of pain, and restoration of functional abilities to pre-injury level

What structures are being involved during the accleration phase

"Provided by lats, pec major, subscap, triceps, and serratus anterior Can lead to labral tears, joint instability, overuse tendonitis, tendon ruptures Elbow extension/arm acceleration is a result of centrifugal force generated by trunk rotation...... Direct result of elbow extension"

What structures provided superior stability of the shoulder?

"Superior stability refers specifically to the superior-inferior component of glenoid concavity, which resists proximal migration of the humeral head within the glenoid. Coupled with the compressive function of the rotator cuff, even with a torn supraspinatus, this component can resist the upward pull of the deltoid. Factors that affect such superior stability include a deficient superior glenoid and the biceps-labral anchorage. "

What is the nerve level and muscle for upper subscapular

1) C5-C6.....2) subscapularis......3) internal rotation

What is the nerve level and muscle for Dorsal scapular nerve

1) C5... 2) both rhomboids.... 3) scapular retraction

What is the management for posterior dislocation?

1) Non-operative treatment must strengthening the external rotators, and adequate force coupling.

What is SICK scapula?

1) Scapular malposition 2) Inferior medial border prominence 3) Coracoid pain 4) scapular dyskinesis

What are the problems with MDI

A patient with symptomatic multidirectional instability (MDI) may complain of instability symptoms but often presents only with pain. Initial treatment is conservative, focusing on strengthening the dynamic components of shoulder stability—the rotator cuff and the scapular stabilizers. A conservative approach is most often successful; however, when a period of prolonged rehabilitation fails (3-6 mo), surgical management may be undertaken to enhance static stabilization by tightening the shoulder capsule. Historically, this is accomplished with an open procedure, but arthroscopic management is evolving rapidly. The prognosis for MDI is generally good.

what is the pathology/treatment/surgery for ulnar neuritis t injury?

Description Irritation of the ulnar nerve resulting from: traction from valgus stress, compression from adhesion, osteophytes, flexor muscle hypertrophy, or friction due to subluxation Treatment options Non-operative rehabilitation, ulnar neurolysis (release) with submuscular or subcutaneous anterior ulnar nerve transposition Author's preference Unclear, but presumed ulnar neurolysis with subcutaneous ulnar nerve transposition

What is the description of elbow dislocation?

Description: Elbow dislocations often occur from a hyperextension injury in which the trochlea is levered over the coronoid process such as falling onto an outstretched arm. Another common mechanism is posterolateral rotatory dislocation resulting from combined forces of axial compression, elbow flexion, valgus stress and forearm supination creating a rotational displacement of the ulna on the distal humerus. The majority of elbow dislocations (90%) result in either posterior or postereolateral displacement of the forearm relative to the humerus.

What is the indication for proximal humeral fracture?

Diagnostic evaluation of proximal humerus fractures is critical in assessing treatment choices. Initially, plain radiographs of good quality that include Neer's trauma series are used to define the extent of injury. These fractures can be classified with the Neer or AO/ASIF classification systems. Each of these methods has certain advantages, but they also share some common problems. Both systems have limited reliability, reproducibility among observers, and consistency in findings by the same observer at different times. Therefore, these images cannot be relied upon entirely to make a treatment decision. Even if CT scans and 3-dimensional images are added, reliability and reproducibility are limited. However, an understanding of fracture type gives the physician essential information on prognosis and treatment options. The Neer classification system is based on displacement criteria of 1 cm or fragment angulation of 45°. The type of fracture then is divided into segments. Four segments are possible, including the articular segment, the lesser tuberosity, the greater tuberosity, and the surgical neck. The basis for the AO/ASIF classification is predicated on disruption of the blood supply to the articular segment, thereby increasing the likelihood of avascular necrosis. These fractures are deemed least ideally suited for internal fixation. The treatment objective in proximal humerus fractures is to allow bone and soft-tissue healing that maximizes function of the upper extremity while minimizing risk. Displaced fractures, if left untreated, have the greatest likelihood of limited functional outcomes. Most fractures are extraarticular and are minimally displaced; these fractures may be treated with supportive treatment only. Persons with stable fractures can begin rehabilitation early and typically have superior functional outcomes. Indications for treatment are displaced articular fractures and periarticular fractures. However, the "personality" of the fracture (eg, bone quality, fracture orientation, concomitant soft-tissue injuries), the personality of the patient (eg, compliant, realistic, mental status), and the personality of the surgeon (eg, surgical experience, technical familiarity, available resources) all have a tremendous effect on specific treatment indications. The most common definition of displacement is 1 cm or more between fracture fragments or 45° of angulation or more between fragments. The segments that most commonly produce these fragments are the humeral articular surface, the greater and lesser tuberosities, and the surgical neck. More recently, a greater tuberosity that is displaced 5 mm or more has come to be considered a fragment that should be reduced.

What is the problem with medial humeral fracture?

Injuries to the medial aspect of the distal humerus in young children can range from an avulsion fracture of the medial epicondyle to a much more serious Salter-Harris type IV fracture of the medial condyle, which crosses the physeal plate.[18] The distinction between these types of fractures is key to the selection of appropriate treatment. This diagnosis may be difficult to ascertain due to the limitations of radiographic visualization of cartilaginous fractures observed with the developing anatomy in this area. Inadequate reduction of the physeal growth plate and the joint surface cartilage in a medial condyle fracture can lead to serious complications. The age of the patient, the extent of initial injury and displacement, and delay in initial treatment play important roles in the clinical outcome. For medial epicondyle fractures, nonoperative management of fractures displaced up to 15 mm does not appear to be associated with functional deficit unless there is entrapment of the epicondyle fragment in the joint. Entrapment of the epicondyle fragment in the joint can block motion, and therefore, the fragment should be reduced.

What is the surgical management for lateral humeral condyle fracture?

Operative management is required in type I fractures that demonstrate delayed displacement or instability. Fragment stabilization is most frequently performed using 2 percutaneously placed smooth Kirschner wires (K- wires). All type II and III fracture patterns require open reduction and fragment stabilization.

What exercises should be avoid with a traction SLAP injury?

Patients with traction injuries should avoid heavy resisted or excessive eccentric biceps contractions.

What are the images for Monteggia Fracture?

Plain radiography Technique - Views of the forearm in orthogonal planes (planes at 90° to each other) are needed with the wrist and elbow joints included. The evaluating physician should also obtain separate radiographs of the elbow to assess the proximal radioulnar joint, ulnohumeral articulation, and the radiocapitellar joint. Findings - The ulna fracture is usually obvious, but the findings associated with the radial head dislocation may be subtle and overlooked. In order to assess the radiocapitellar joint, a line should be drawn parallel to the long axis of the radius. This line should point directly at the capitellum on any projection of the elbow. The radial head dislocation almost always points in the same direction as the apex of the ulna fracture. In children, recognizing a plastic deformation of the ulna, which may also lead to radial head dislocation, is important.

What are images for clavicle fracture?

Recommended radiographic views of the clavicle include anteroposterior view, which needs to include both the sternoclavicular joint and the shoulder girdle. Most clavicle fractures are evident on this view. Additional views that may assist with radiographic diagnosis include 45-degree cephalad and caudal views. Stress views may be used to identify patterns of displacement and are particularly helpful in the context of fractures of the distal clavicle. Bedside ultrasound has been shown to be effective in the diagnosis of clavicle fractures in children. Bedside ultrasound requires minimal formal training, is not associated with an increase in pain, and may reduce the length of stay in the emergency department

What are the staging for olceranon fracture?

Several classification systems have been suggested. The Arbeitsgemeinschaft für Osteosynthesefragen (AO) - Association for the Study of Internal Fixation (ASIF) classification, used by the Orthopaedic Trauma Association, divides the fractures into 3 broad categories as follows[7] : Type A fractures are extra-articular. Type B fractures are intra-articular. Type C fractures are intra-articular fractures of both the radial head and olecranon. Schatzker developed a classification with 6 types as follows (types A, B, and C are intra-articular fractures): Type A is a simple transverse fracture. Type B is a transverse impacted fracture. Type C is an oblique fracture. Type D is a comminuted fracture. Type E is a more distal fracture, which actually is extra-articular. Type F is a fracture dislocation.

What are the signs and symptoms of anterior shoulder instability?

Severe pain in the shoulder at the time of injury. Loss of shoulder function and severe pain when attempting to move the shoulder. feeling like your shoulder wants to slip out of place..... tenderness, deformity, and swellling, pain with moving the shoulder, especially when reaching overhead, pain with heavy lifting, pain that awakens you at night.... loss of strength.... numbness or paralysis in the upper arm and deltoid muscle from pinching, stretching,or pressure on the blood vessels or nere. Crepitation feeling and sound when the injured area is touched or with shoulder motion.... decreased or absent pulse at the wrist secondary to blood vessel damage.

What is the relevant anatomy for clavicle fracutre?

The clavicle is an S-shaped bone that acts as a strut between the sternum and the glenohumeral joint. A weak spot in the clavicle bone is present at the midclavicular region, which accounts for most fractures occurring in this region. Numerous muscular and ligamentous forces act on the clavicle, and knowledge of these differing forces is necessary to understand the nature of displacement of clavicle fractures and why certain fracture patterns tend to cause problems if not reduced and surgically stabilized. The clavicle articulates with the sternum at the sternoclavicular joint and with the acromion at the AC joint. Many ligamentous structures attach to the clavicle and provide stability for the articulations with the sternum and the acromion. The primary stabilizers of the sternoclavicular joint are the anterior and posterior capsules. Other ligamentous structures attaching here are the interclavicular ligament and the costoclavicular ligament. Stability of the sternoclavicular joint in the anterior-posterior plane is derived predominantly from the posterior capsule, with additional stability conferred by the anterior capsule. The interclavicular and costoclavicular ligaments have little effect on stability of the joint. At the level of the AC joint, the coracoclavicular ligament and the AC ligament provide stability for the joint. The coracoclavicular ligament is actually 2 separate ligaments, the conoid and the trapezoid, which both attach from the coracoid to the inferior surface of the distal clavicle. Debski et al have delineated the different functions of the conoid and trapezoid in resistance to applied loads to the AC joint.[22] The conoid is the predominant restraint to anterior and superior loading. The trapezoid is the major restraint to posterior loading at the AC joint. The AC ligament is at the superior-lateral aspect of the clavicle and overlies the AC joint. Three muscles originate on the clavicle, and 3 muscles insert on the clavicle. The muscles that take their origin from the clavicle are the sternohyoid, the pectoralis major, and the deltoid. The muscles that insert on the clavicle are the sternocleidomastoid, the subclavius, and the trapezius. These muscles may become deforming forces on the clavicle in the presence of a fracture, and the displacement of fracture fragments depends on the location of the fracture in relation to the muscular and ligamentous attachments. Many other important structures are in extremely close contact with the clavicle and are thus subject to injury in the context of clavicle fractures. The subclavian artery, which becomes the axillary artery as it passes anteriorly to the first rib, and vein are both in close proximity to the middle portion of the clavicle. Additionally, the brachial plexus also passes behind the clavicle posterolateral to the subclavian vessels and is at risk with displaced fractures of the middle clavicle. The subclavius muscle lies between the clavicle and these neurovascular structures, and, though small, it is believed to prevent more frequent damage to these structures. Reports also exist of injuries to the apices of the lung, most commonly with displaced middle third clavicle fractures.

What is the clinical anatomy of lateral epicondylitis?

The elbow joint is made up of three bones: the humerus (upper arm bone), the radius and ulna (two bones in the forearm). At the distal end of the humerus there are two epicondyles, one lateral (on the outside) and one medial (on the inside). Musculature involved is most commonly the extensor carpi radialis brevis (ECRB), but this injury can also involve the extensor carpi radialis longus (ECRL), extensor digitorum, and extensor carpi ulnaris. The radial nerve is also in close proximity to this region, and divides into the superficial radial nerve and the posterior interosseous nerve.

What is the normal range of motion of the elbow?

The elbow joint moves in several planes. Normal motion in the sagittal plane includes flexion from 0° to 140° 10° and forearm rotation of 80° to 90° in both supination and pronation

What is the presentation of Sprengel deformity?

The hallmarks of the Sprengel deformity are shoulder asymmetry and restriction of shoulder abduction. Clinically, the affected scapula usually is elevated 2-10 cm and is adducted, and its inferior pole is rotated medially. Because of this rotation, the glenoid faces inferiorly. A prominence in the suprascapular region is characteristic because of the upwardly rotated superomedial angle of the scapula, which causes the ipsilateral side of the neck to appear fuller and its normal contour to be lost. The scapula is hypoplastic, and the length of the vertebral border is decreased. Occasionally, some anterior bending of the supraspinous portion is present. Passive movement of the glenohumeral joint, including abduction and external and internal rotation, may be normal. However, scapulothoracic movements may be severely limited. In 40% of patients with a Sprengel deformity, combined abduction is limited to less than 100º. The omovertebral bone may also limit abduction by affecting scapular mobility, and can also limit neck movement if this bone is attached high in the cervical spine. Other causes of limited abduction include abnormal and weakened scapular muscles. The left side is more commonly affected than the right side. The condition may sometimes be bilateral, in which case, although it is cosmetically much more acceptable, functionally, it is more disabling.

What are the indications for anterior instability surgery?

The indications for open surgical repair depend on the individual surgeon. Shoulders in which conservative therapy has failed or any shoulder rendered unstable may undergo open repair. Frequently, classifying patients into one of two categories helps to determine treatment. Young patients with heavy physical demands may forego conservative therapy and proceed to immediate surgical repair. This recommendation is based on the high recurrence rate and the patient's desire to return to activity. In contrast, older patients who are less physically demanding may try conservative therapy prior to assuming the risks of surgical repair. These patients have a lower likelihood of recurrence and may only require conservative therapy. Surgery is also indicated if the patient is symptomatic with activities of daily living or if overhead stability is specifically needed. The patient must understand that overhead stability cannot be guaranteed.

What is the pathophysiology of lateral humeral condyle fracture?

The lateral condyle fracture is a Salter-Harris IV fracture pattern and follows physeal injury principles. For more information about injuries of the growth plate, see Salter-Harris Fractures. The fracture fragments in these patients are primarily cartilaginous as a result of the young age of the patients. The radiographic interpretation may be misleading because the visible fragment appears smaller than the actual size and, in addition, the amount of displacement is not appreciated. In lateral condyle fractures, the displacement is greater than appreciated, and incongruity of the articular surface is present. Fractures with minimal displacement must be carefully monitored, as they have a high tendency to displace. Once these displaced fractures consolidate in a malunited position, treatment is difficult, dangerous, and fraught with complications. For these reasons, surgical reduction should be performed and is recommended within the first 48 hours postfracture.[7]

What are the mechanics of the load and shift test?

The load and shift test was described by Silliman and Hawkins16 in 1993. "The patient should be seated for this part of the examination. The examiner should be behind the patient on the side to be examined. The examiner places the hand over the shoulder and scapula to steady the limb girdle and then, with the opposite hand, grasps the humeral head. As the head is 'loaded', both anterior and posterior stresses are applied and the amount of translation is noted. Next, the elbow is grasped and infe- rior traction is applied. The area adjacent to the acromion is observed, and dimpling of the skin may indicate a 'sulcus sign'. . . . if present, the 'sulcus sign' should be re- ported in centimeters (i.e., the number of centimeters the humeral head is displaced from the inferior surface of the acromion). "Glenohumeral translation is assessed with the patient supine. Here the arm is grasped in a position of approxi- mately 20° abduction and forward flexion in neutral rota- tion. The humeral head is loaded and then posterior and anterior stresses are applied. Similarly, inferior stress is applied again noting the 'sulcus sign'." Faber et al.1 described an alternative version of the load and shift test as part of a comparison of the effects of anesthesia on the results of the test in 1999. "In this test, the humeral head was loaded in such a way as to center it congruently within the glenoid fossa. The humeral head was then maximally stressed or shifted anteriorly and posteriorly so that movement of the humeral head relative to the glenoid face and the glenoid rim could be assessed. The humeral head was stressed with enough force to achieve translation to its end point. Each shoulder was examined with the patient in the supine position and the arm in approximately 20° of abduction, 20° of forward flexion, and neutral rotation. Inferior translation was evaluated by the application of an axial load with the patient's arm resting comfortably by the side." From the analysis of the effects of anesthesia, the au- thors concluded that 92% of patients had a higher grade of anterior translation during examination under anesthesia than when awake for both affected and unaffected shoul- ders. However, this did not imply a pathologic condition and reinforced the observation that both shoulders should be examined on all occasions.

What is the etiology of olecranon fracture?

The most common mechanism of an olecranon fracture is a fall on the semiflexed supinated forearm. As the hand strikes the ground, muscles are tensed to break the fall, and the powerful triceps snaps the olecranon over the lower end of the humerus, which acts as a fulcrum. The next most frequent cause of this injury is direct trauma, as in falls on, or blows to, the point of the elbow. Occasionally, the olecranon may be fractured by hyperextension injuries, such as those resulting in elbow dislocation in adults or supracondylar fractures in children. Very rarely is the olecranon broken by muscular violence, as in throwing.

What is the problem with olecranon fracture?

The olecranon is the proximal bony projection of the ulna at the elbow. Olecranon fractures are a diverse group of injuries, ranging from simple nondisplaced fractures to complex fracture dislocations of the elbow joint.

What is the etiology of shoulder dislocation?

The usual mechanism of injury is extreme abduction, external rotation, extension, and a posterior directed force against the humerus. Forceful abduction or external rotation alone can also lead to dislocation (about 30% of cases), as can a direct blow to the posterior humerus (29%), forced elevation and external rotation (24%), and a fall onto an outstretched hand (17%).[6, 7, 8] Posterior dislocations are less common (2-10%) and are the result of an axial load applied to the adducted and internally rotated arm. Classic posterior dislocations also occur as a result of electrocution or seizures because of the strength imbalance between the internal rotators (subscapularis, latissimus dorsi, pectoralis major muscles), which overpower the external rotators (teres minor and infraspinatus muscles). Inferior dislocations are rare and result from a hyperabduction force that causes the humeral neck to lever against the acromion. Diagnosing inferior dislocations is critical because of the high incidence of complications. Neurologic injuries (particularly axillary nerve lesions) are associated with inferior dislocations in as many as 60% of cases, vascular injuries occur in about 3.3% of cases, rotator cuff tears in occur in 80-100% of cases,[9] and greater tuberosity fractures and pectoralis major avulsions are also associated with inferior dislocations. Superior dislocations are extremely rare and result from an extreme force in a cephalic direction to the adducted arm. Acromioclavicular injuries and fractures of the acromion, clavicle, and tuberosities may occur with superior dislocations. Atraumatic instability is usually multidirectional and commonly occurs in individuals with generalized hyperlaxity due to connective tissue disorders, such as Ehlers-Danlos syndrome and Marfan syndrome. A small or flat glenoid fossa, excessive anteversion or retroversion of the glenoid, weak rotator cuff muscles, neuromuscular disorders, or a redundant capsule may also jeopardize the concavity-compression, adhesion-cohesion, or the glenoid suction-cup phenomena that aid in stability of the shoulder. Multidirectional instability most commonly occurs in younger populations, usually in patients younger than 30 years, and is often familial and bilateral. The first dislocation often occurs after a minor injury or after a period of disuse. Patients may experience subluxations that progress over time to actual dislocations, which spontaneously reduce. These dislocations may be voluntary or involuntary. Voluntary dislocations have been associated with psychiatric illnesses and may be used in attention seeking behavior. Surgery should be avoided in this population because the instability is likely to recur.

What is the management for SAI

There is strong evidence that non-operative rehabilitation when supervised both decrease pain in the shoulder and increase function. Non-operative treatment shoulder therefor be attempted first, assuming there is no tear that requires surgery. No data guarantees that surgical treatment is more effective than conservative treatment. Therapeutic exercise should include rotator cuff strengthening, lower and middle traq strengthening and anterior/posetior shoulder strengthening.

What are the mechanics of the Jobe's apprehension relocation test?

This combination test was described by Jobe and Kvitne6 in 1989. "These tests are performed with the patient su- pine and the arm in abduction and external rotation. During the Apprehension Test, the examiner pushes an- teriorly on the posterior aspect of the humeral head. This maneuver will produce apprehension sometimes coupled with pain in patients with recurrent dislocations. Patients with anterior subluxation will experience pain but not apprehension with this test, and patients with normal shoulders will be asymptomatic. The Relocation Test is then performed by administering a posteriorly directed force on the humeral head. Patients with primary im- pingement will have no change in their pain, whereas patients with instability (subluxation) and secondary im- pingement will have pain relief and will tolerate maximal external rotation with the humeral head maintained in a reduced position." Evaluation of the test in 1993 concluded that it was common to have pain in the position of 90° of abduction and 90° of external rotation from a variety of disorders and that this will be diminished by a posteriorly directed force.18 The test became more accurate when apprehen- sion was used as the diagnostic criterion. Sensitivity was 68%; specificity, 100%; positive predictive value, 100%; negative predictive value, 78%; and accuracy, 85% for the relocation test when apprehension was the determinant of a positive result.

A 14 year old elite gymnast complains of constant anterior shin pain of insidious onset which increases with activity and decreases with rest. Symptoms have been present for 3 weeks after joining her school track team in addition to gymnastics. Her past medical history is significant for a stress fracture of her 5th metatarsal. What recommendations regarding treatment do you have for this athlete?

b. Referral to her primary care provider for a bone scan and evaluation of mentrual function Correct: We will not be the first line of intervention for a suspected female athlete triad. A referral to her PCP is recommended not only for diagnosis of her condition, but for proper outsourcing of referrals as well.

What are the special test for pronator teres syndrome?

o Resisted pronation, point tenderness o Nerve conduction velocity test

What is the 3 MOI for a brachial plexus injury?

"1. Traction caused by lateral flexion of the neck away from the involved side (similar to the mechanism in birth trauma) 2. Direct impact to the Erb point causing compression to the brachial plexus (often associated with poor-fitting shoulder pads) 3. Nerve compression caused by neck hyperextension and ipsilateral rotation (The neural foramen narrows in this mechanism.)"

How do you stress the anterior-inferior humeral ligament?

"> Pt. is supine at the right side of he bed. Humerus must over the edge of the bed, but scapul in contact for stabilization. > Facing the patient, level with the shoulder girdle. Therapists left arm passes over the patient forearm and grasps the humerus as close to the humeral neck as possible... Keeping the patients arm as close to their side as possible, therapist rotates the GHJ joint to the barrier of external rotation.The therapist maintains the external rotation while extending the GHJ, then abducts to 90 degrees. Therapist right hand fixes the shoudler girdle. > Stress: An anterior glide together with distraction towards the therapist sternum."

What x-ray view hill-sach lesion?

"The AP view can be obtained in neutral, internal, or external rotation. Ininternal rotation, one can easily see a Hill-Sachs lesion of the posterolateral humeral head. The Stryker notch view can also be useful in detecting Hill-Sachs lesions.However, this view is of limited usefulness in detecting subluxations or glenoid fractures. "

What is the glenohumeral suction cup effect?

"The glenohumeral suction-cup effect depends upon the tendency for matched concave and convex surfaces with a flexible periphery to center and stabilize after expressing any intervening air and fluid, thereby forming a seal.Deficiencies of the glenoid labrum or of the margin of the glenoid can adversely affect this stabilizing mechanism. "

"Differences between grade I,IIand III AC

...

How does an internal impingment differ from typical subacrominal impingemen?

...

What are the special test/imaging for ACJ sprain.

1) AC compression test...2) x-ray

What are the 5 special test for instability?

1) Apprehension test (tests for anterior instability in maximal external rotation 2) Jobe's apprehension-relocation test (apprehension used as positive diagnostic criteria in testing for anterior instability) 3) Fowler's sign (decrease in apprehension in maximal external rotation when examiner applies pressure on anterior shoulder acting as an anterior stability) 4) Augmentation test (examines anterior instability by pulling forward on back of arm in maximal external rotation) 5) Anterior release test (tests for anterior instability by applying anterior stabilizing force then releasing, positive result is apprehension and pain)

How to you test the middle gleno-humeral ligament.

> Pt. is supine at the right side of he bed. Humerus must over the edge of the bed, but scapul in contact for stabilization.> Facing the patient, level with the shoulder girdle. Therapists left arm passes over the patient forearm and grasps the humerus as close to the humeral neck as possible... Keeping the patients arm as close to their side as possible, therapist rotates the GHJ joint to the barrier of external rotation.The therapist maintains the external rotation while extending the GHJ, then abducts to 45 degrees. Therapist right hand fixes the shoudler girdle. Stress: An anterior glide together with distraction towards the therapist sternum

What is the differential diagnosis of little league elbow syndrome?

Anterior capsule strain Degenerative joint disease (DJD) Distal biceps rupture Inflammatory arthropathy Lateral epicondyle avulsion fracture Loose body Medial epicondyle avulsion fracture Olecranon stress fracture Pronator syndrome Synovitis Torn brachialis muscle

What are the indications for coronoid fracture?

A displaced coronoid fracture that presents with a block to elbow motion is a definite indication for surgical stabilization.[15] Type 3 fractures need to be fixed.[16] If the fragment is too comminuted for internal fixation, then a hinged external fixator should be applied across the elbow and a protected range-of-motion program should be started.[17] Type 2 fractures with elbow instability are also best treated with a hinged external fixator. Type 1 and most type 2 fractures are usually managed nonoperatively.

What are the pain patterns for SLAP lesion?

A patient with a superior labral injury may have nonspecific complaints. Pain com- plaints are typically intermittent and are most fre- quently associated with overhead activity. Often patients exhibit mechanical symptoms of painful clicking or catching of the shoulder.2 Pain is typically elicited with specific movements and the condition is not painful at rest. We refer to this as ''mechanical pain'' as opposed to pain at rest, which is often present when rotator cuff pathology is present. Over- head athletes typically report a loss of velocity and accuracy along with general uneasiness of the shoul- der. Snyder et al48 have reported that this type of subjective complaint is present in 50% of patients. Probably the most predictive subjective complaint in the athlete is the inability to perform sporting activi- ties at a high level.

What is the Relevant anatomy for MDI?

A thorough discussion of the relevant anatomy is included in the discussion of surgical techniques available for multidirectional instability (MDI), both open and arthroscopic (see Open surgical technique and Arthroscopic surgical technique in the Treatment, Surgical therapy section). The reader is also urged to read the Complications section, especially with regard to precautions necessary when working close to the axillary nerve.

What are the outcomes for MDI surgery?

A thorough review of the literature is included under surgical management (see Surgical therapy). The review outlines the rationale, development, and results of open and arthroscopic repairs. In general, patients who have had open capsular shifts do reasonably well. Published studies indicate that recurrence of MDI following surgery is about 10%. Loss of ROM following open capsular shift repair was greater in the early case series than in the later series, particularly for external rotation and abduction. Reported complications are rare. Good results tend to persist with time as well. Stability does not seem to be lost at later follow-up on individuals with conventional open shift repair. The long-term follow-up of arthroscopic management of MDI cannot yet be assessed. Advocates of both suture and thermal techniques have reported results that are somewhat less favorable in some cases than those of open surgery at follow-ups of up to 2 years, with recurrence rates of approximately 20-30%[42, 40, 39] versus 10% in open repairs.[12, 20] -[23] However, others have reported similar results at > 4 years.[43]

What are the contraindications for MDI surgery?

According to Neer and Foster,[12] contraindications to the surgical management of MDI include willful, habitual, or voluntary shoulder instability; collagen connective tissue disorders (eg, Ehlers-Danlos syndrome,Marfan syndrome); and lack of a trial of, or noncompliance with, a supervised rehabilitation program.

What is the problem with cubital tunnel syndrom?

Affected patients often experience numbness and tingling along the little finger and the ulnar half of the ring finger. This discomfort is often accompanied by weakness of grip and, rarely, by intrinsic wasting.

What are the common special test for labral injury?

Active compression test (Obrien's), compression- rotation/grind test, Speeds test, dynamic Speeds test, clunk test, crank test, anterior-slide test, biceps-load test, biceps-load II test, pain provocation test, as well as two recently developed tests: pronated load test and resisted supination external rotation test.

What special tests should you perform with a compressive injury?

Active compression-compression rotation clunk test anterior glide

What is the adhension-cohension stability.

Adhesion-cohesion is a mechanism by which fluid on coated surfaces provides an intrinsic adherence between the surfaces. This may be affected by changes in the fluid chemistry (secondary to inflammatory disease), loss of smoothness of the surfaces (secondary to degenerative disease), and alterations in the contact areas

What is the differential diagnoses of ulnar neuropathy?

Alcohol (Ethanol) Related Neuropathy Amyotrophic Lateral Sclerosis Cervical Spondylosis: Diagnosis and Management Traumatic Peripheral Nerve Lesions

What is the differential diagnoses for medial neuropathy?

Alcohol (Ethanol) Related Neuropathy Cervical Spondylosis, Diagnosis and Management Chronic Inflammatory Demyelinating Polyradiculoneuropathy De Quervain Tenosynovitis Diabetic Neuropathy Neuropathy of Leprosy Thoracic Outlet Syndrome Ulnar Neuropathy

What is the pathology of the omoverebral connection in sprengel?

An omovertebral connection, which may be fibrous, cartilaginous, or bony, may exist in about one third of cases.[5, 10] This connection attaches the superomedial angle of the scapula to the spinous process, lamina, or transverse process of the cervical vertebrae and may be the primary cause of restricted shoulder motion in patients with a Sprengel deformity.[17] The omovertebral connection is usually unilateral, is always associated with a fixed, elevated scapula, and has a major role in determining the shape and the malpositioning of the scapula.[18, 19] According to Willet and Walsham, the omovertebral bar is homologous to the suprascapular bone in lower vertebrates.[2]

What muscle mass is near the MCL?

Anatomically, the fibers of the MCL and the common flexor-pronator tendons are intermingled and lie directly adjacent to each other just distal to the origin from the medial epicondyle. Injury to one of these structures could conceivably cause symptoms to arise in the other structure.

What secondary muscle is affected with a SLAP lesion?

Andrews et al3 reported that 45% of patients (and 73% of baseball pitchers) with superior labral lesions had concomitant partial thick- ness tears of the supraspinatus portion of the rotator cuff.

What is the sublabral hole?

Another common normal finding is a minimal recess or anterior sublabral hole that must not be confused with a SLAP lesion.34 This anterior sublabral hole can exist below the biceps attachment at or just above the 3-o'clock position (in a right shoulder) on the glenoid rim where a small notch is typically found.

What is a Bankart Lesion?

Anterior labral detachment

A basketball player comes to the sideline after injuring his finger during a play. He reports that when trying to catch the ball, he misjudged it, and the ball hit is middle finger in a fully extended position, forcing it into flexion. Pain is found on the dorsal aspect of the DIP joint. He is limited in active DIP extension. He attains x-rays which are found to be negative for fracture. Splinting is deemed appropriate for the diagnosis of a mallet finger. Research supports which of the following splints?

Any of the above 3 will work, as long as the finger remains in extension Incorrect: While splinting with a stack splint is good, it is not considered superior. See the Cochrane review by Handoll & Vaghela (2008). Also, consider reviewing the AAFP article by Leggit & Meko (2006) for Acute Finger Injuries.

What is the future of lateral humeral condyle fracture?

Arthroscopic-assisted reduction with internal fixation offers direct visualization of the articular fragment, ensuring articular congruity, and percutaneous fixation avoids an extended dissection. This option provides a more biologic approach to treatment of this fracture. Cross-training in arthroscopic techniques and fracture management is necessary.[15, 16, 17]

What are the procedures for UCL injury?

Arthroscopy is believed by some authors to be the most specific diagnostic procedure because it allows visualization of the medial compartment while valgus stress is applied;; however, clinical suspicion of a UCL injury via a good history and physical examination is probably the most reliable in making the diagnosis.

What are the mechanics of the anterior drawer test?

As described by Gerber and Ganz3 in 1984, "The test is performed with the patient supine. It should not be per- formed with the patient standing or sitting; in these posi- tions we have not been able to reproduce it reliably. The examiner stands facing the affected shoulder. Assuming the left shoulder is being tested, he fixes the patient's left hand in his own right axilla by adducting his own hu- merus. The patient should not grasp the surgeon's axilla but should be completely relaxed. To be sure that relax- ation is complete, the examining surgeon gently taps the patient's elbow. degrees of abduction, 0 degrees to 20 degrees of forward flexion, and 0 degrees to 30 degrees of lateral rotation; this position should be quite comfortable. The examiner holds the patient's scapula with his left hand, pressing the scap- ular spine forward with his index and middle fingers; his thumb exerts counter-pressure on the coracoid process. The scapula is now held firmly in the examiner's left hand. With his right hand, he grasps the patient's relaxed upper arm in its resting position and draws it anteriorly with a force comparable to that used at the knee in Lachman's test. The relative movement between the fixed scapula and the movable humerus can easily be appreciated and can be graded as with knee instability."

What is the clinical presentation for medial epicondylitis?

As medial epicondylitis is an inflammation of the tendon of the flexor group attached to the medial epicondyle of the humerus, the most sensitive region will be located near the origin of the wrist flexor group. The patient usually complains of pain about the elbow distal to the medial epicondyle of the humerus with radiation up and down the arm, most common on the ulnar side of the forearm, the wrist and occasionally in the fingers (level of quality A1)[5]. Other characteristics are the local tenderness over the medial epicondyles and the conjoined tendon of the flexor group, without evidence of swelling or erythema. The pain evokes by resisted flexion of the wrist end by pronation. The pain usually notices a weakness of hand grip (level of quality A1)[5].

What is the history of ulnar neuropathy?

Both the onset and progress of the symptoms can be variable. Although the answer is frequently negative, one should ask specifically about trauma and pressure to the arm and wrist, especially the elbow, the medial side of the wrist, and other sites close to the course of the ulnar nerve. Many patients complain of sensory changes in the fourth and fifth digits. Rarely, a patient actually notices that the unusual sensations are mainly in the medial side of the ring finger (fourth digit) rather than the lateral side, corresponding to the textbook sensory distribution. Sometimes the third digit is also involved, especially on the ulnar (ie, medial) side. The sensory changes can be a feeling of numbness or a tingling or burning. Pain rarely occurs in the hand. Complaints of pain tend to be more common in the arm, up to and including the elbow area. Indeed, the elbow is probably the most common site of pain in an ulnar neuropathy. Occasionally, patients specifically say "I have pain in my elbow," "I have pain in my funny bone," or even "I have pain in this little groove in my elbow," but usually they are not quite so explicit unless prompted. Patients rarely notice specific muscle atrophy. Weakness may also be a presenting complaint, but the complaint may be expressed in subtle ways. One traditional sign of ulnar neuropathy, Wartenberg sign, is actually a complaint of weakness. The patient complains that the little finger gets caught on the edge of the pants pocket when he or she puts the hands into the pocket. At first, that complaint seems surprising because most physicians remember that finger abduction is governed by the ulnar nerve. So the physician might think that with an ulnar neuropathy, the patient would have less tendency to have the little finger abducted and thus caught on the edge of the pocket. But adduction is also mediated by the ulnar nerve. In essence, the patient cannot abduct the fifth digit tightly against the fourth because of weakness of the interosseous muscles. In addition, the muscle that extends the fifth digit at the metacarpal phalangeal joint (extensor digiti quinti) is radially innervated and it inserts on the ulnar side of the joint. Normally this muscle is opposed by ulnar innervated muscles that flex the joints. But with an ulnar neuropathy, the muscle is relatively unopposed so it pulls the finger up and to the ulnar side. This is the perfect position to catch onto the edge of the pocket. The patient also may express the complaint of weakness by saying "my grip is weak." Many of the grip muscles are ulnar. Also, when someone tries to grip powerfully, the hand usually deviates in the ulnar direction under the influence of the flexor carpi ulnaris. If this ulnar deviation is impaired, the grip mechanism does not work optimally even for the muscles that are unimpaired. Sometimes a patient notices that his pincer grip (pinching with the thumb and index finger) is weak. Two of the key muscles involved in this movement are the adductor pollicis (which adducts the thumb) and first dorsal interosseus, which adducts the index finger. Not only may the pincer grip be weak in an ulnar neuropathy, the median innervated flexor pollicis longus partially compensates for the weakened adductor pollicis and the thumb flexes at the distal joint. Usually a patient does not notice the thumb flexion, but when demonstrated by the examiner, this flexion is considered to be Froment sign.

What is a hill Sac Lesion

Compression fracture of the posterolateral humeral head

What is the epidemiology of medial neuropathy?

CTS is the most common focal peripheral neuropathy. The reported incidence varies by location and methodology used. Prevalence rates for CTS are reported as 1-5% in the general population and 5-15% in the industrial settings. An increasing temporal trend has been reported in several studies.[7, 8] According to data from the 1980s, the prevalence of electrophysiologically confirmed symptomatic CTS is about 3% among women and 2% among men.[9] A cross-sectional survey reported in 2001 calculated the lowest possible prevalence of symptomatic CTS in the general US population as 3.72%.[10] Among residents of Olmsted County, Minnesota, the adjusted annual rates of medically diagnosed CTS increased from 258/100,000 in 1981-1985 to 424/100,000 in 2000-2005.[7] For this last period included in the study, the incidence in women was 542/100,000 and in men was 303/100,000. Generally, the most marked increases in CTS incidence were seen in younger age groups of both sexes in the first part of the study period and among older age groups in the final decades of study. The cause of the increase is unclear, but it corresponds to an epidemic of CTS cases resulting in lost work days that began in the mid 1980s and lasted through the mid 1990s. The elderly present with more severe disease and are more likely to have carpal tunnel surgery.[7] International In the general population for a Dutch community, the prevalence rate of undetected CTS was 5.8% in adult women, and an additional 3.4% already carried the diagnosis of CTS. The overall prevalence rate for men was 0.6%.[11] A primary care study in the UK from 2000 reported an annual incidence of CTS of 88/100,000 in men, and 193/100,000 in women. New presentations were most frequent in women aged 45-54 years.[12] In this study, CTS was as common as all other entrapment neuropathies combined. A study in Italy reported a mean standardized annual incidence of 329/100,000 in the Siena area (Tuscany) from 1991-1998, with 139 for men and 506 for women. The age-specific incidence for women increased gradually with age, reaching a peak from 50-59 years. In men, there was a bimodal distribution with peaks from 50-59 years and 70-79 years.[8] A French study of CTS from 2002-2004 in patients aged 20-59 years reported a mean incidence rate per 1000 person-years that was higher in employed than unemployed persons (1.7 vs 0.8 in women and 0.6 vs 0.3 in men). Higher values were blue-collar workers and lower-grade services, sales, and clerical white-collar workers.[13] Mortality/Morbidity CTS is associated with high costs to the Health Care system and society. According to 1988 data from the United States, every year an estimated 1 million adults require medical treatment for CTS.[14] About 400,000-500,000 CTS surgeries annually were reported in 1995 with an economic cost of more than 2 billion.[15] In 1999, CTS cases were associated with a median number of 27 days lost from work, the highest number of any major disabling illness or injury.[16] Race Findings of the 1988 National Health Interview survey indicate that CTS is 1.8 times more prevalent in whites than nonwhites. Sex The reported female-to-male ratio ranges from 3:1 to about 10:1. Phalen's original series in 1970 included 280 women and 96 men (female-to-male ratio 3:1).[17] Age Of the patients in Phalen's series, 58% were adults aged 40-60 years.

What is the medial therapy for medial humeral fracture?

Closed reduction and cast immobilization is adequate for nondisplaced stable medial condyle fractures. Medial epicondyle fractures also may be treated in a closed fashion if the medial epicondyle is nondisplaced, minimally displaced, or even displaced up to 15 mm (see images below). In many studies, including long-term follow-up reports, patients treated nonsurgically had results similar to those of patients treated surgically, even for fracture fragments displaced as much as 15 mm. A radiographic nonunion of the medial epicondyle fracture fragment associated with nonsurgical treatment was not found to have any functional impairment in at least 1 long-term study The only absolute indications for operative management of closed medial epicondyle fractures are the incarceration of the medial epicondyle fragment within the joint and an open fracture. An incarcerated fragment within the joint must be removed. Several closed means of reduction can be used, and the success rate with these methods approaches 40%. One such maneuver (Roberts manipulative technique) is performed under sedation and involves placing a valgus stress on the elbow while supinating the forearm and simultaneously dorsiflexing the wrist and fingers to place the forearm flexor muscles on stretch. If employed, this maneuver is usually performed in the operating room with the patient under general anesthesia. Joint distention techniques also have been described to help facilitate the closed reduction of the incarcerated medial epicondyle fracture.[22, 23, 25] Initially, the arm should be splinted in 90° of elbow flexion. Gentle active range-of-motion exercises may begin within 1 week after injury. Protective splinting may be continued for 3 weeks if necessary. Open reduction with rigid internal fixation is used The arm is placed in a posterior splint for stabilization, elevated, and treated with ice packs to decrease swelling.

What are the S&S of lateral Epicondylitis

Commonly amongst 30-54 year olds, it is less common in individuals under 30 o Patients often report weakness in their grip strength or difficulty carrying objects in their hand, especially with the elbow extended. o They have complaints of pain just distal to and localized tenderness over the lateral epicondyle. o Patients will commonly have pain with palpation of the lateral epicondyle, resisted wrist, or second or third finger extension

What is the differential diagnoses?

Compartment Syndrome Note pressure throughout the forearm - and the effect of passive stretch Severe forearm pain is an indication this may be present Open Fracture Look for bleeding or wounds Neurological injury Numbness, tingling, loss of sensation

What is the return to play for UCL injury?

Competition may be resumed when the following criteria are met: The athlete is pain free with throwing. Elbow and shoulder ROM has returned to within normal limits. Forearm strength has returned to baseline. Good throwing biomechanics have been established.

What are the complication for Monteggia fracture?

Complications include infection, bleeding, malunion, nonunion, nerve injury, redislocation of the radial head, radioulnar synostosis, and chronic pain. Many of the complications listed are significantly reduced with timely diagnosis, adequate reduction, stable surgical fixation, and appropriate postoperative care. Most nerve injuries are neurapraxias, and function usually returns within 1 to 6 months. Baseline electrodiagnostic studies are obtained early. If nerve function does not return within 2 to 3 months, surgical exploration may be indicated. If the nerve injury results from reduction or operative treatment, it should be addressed immediately. Prolonged or complete nerve dysfunction requires early splinting and therapy and may result in the need for tendon transfers. If the radial head dislocates after surgery, improper ulnar reduction must be considered. If this is the case, the hardware should be removed and a proper reduction of the ulna should take place. If dislocation of the radial head is recognized more than 6 weeks after the surgery, a radial head excision should be performed. A nonunion or malunion complication can be considered for bone grafting. Chronic pain may be the result of hardware or improper reduction. If all mechanical causes have been excluded, consulting a pain management specialist should be considered.

What are the complication for coronoid fracture?

Complications include the following: Neurovascular injury Stiffness Heterotopic ossification Instability and recurrent dislocation Posttraumatic arthritis of the elbow

What is the management for bicep rupture.

Conservative therapy includes limiting activity of the involved extremity.. NSAIDS, as well as a comprehensive physical therapy program.

What is the future of olecranon fracture?

Controversy exists regarding the amount of acceptable articular displacement for closed treatment. Certainly, several millimeters of displacement are usually well tolerated. Degenerative changes occur in fewer than 20% of these patients. The method chosen for open treatment of olecranon fractures is also controversial. Decisions regarding fragment excision versus internal fixation often are based on percentage of joint space involvement. McKeever and Buck in 1947 stated that as much as 80% of the trochlear notch can be excised without compromising elbow stability, provided that the coronoid and distal trochlea are preserved. One patient developed anterior instability following excision of 75% of the articular surface. The consensus certainly suggests that at least 50%, but likely less than 80%, of the articular surface can be excised, and a good result can still be obtained.[15]

What is the frequency for coronoid fractures?

Coronoid fractures account for less than 1-2% of all elbow fractures. Coronoid fractures have been identified in 10- 15% of elbow dislocations.[8, 9] Radial head fractures are seen in about 5-10% of elbow dislocations.[10] Coronoid fractures, especially with large fragments, are associated with more instability and an increased incidence of complications.

What are the complication little league elbow syndrome?

Cryotherapy can cause symptoms of ulnar neuropathy when this therapy is used on the medial elbow due to the close proximity of the ulnar nerve to the surface at the cubital tunnel. As previously mentioned, caution should be used when applying compression over areas of nerve entrapment (see Physical therapy in the Treatment, Acute Phase, Rehabilitation section).

What is cubital tunnel syndrome?

Cubital tunnel syndrome is the second most common compression neuropathy of the upper limb (Black, 2000) and involves compression of the ulnar nerve as it passes through the cubital tunnel at the elbow.

what is the pathology/treatment/surgery for ulnar collateral ligament injury?

Description Attenuation or acute rupture of anterior band of ligament resulting from high tensile stresses during acceleration phase of throwing Treatment options Non-operative rehabilitation, direct ligament repair, or free-tendon graft reconstruction Author's preference Ulnar collateral ligament reconstruction with subcutaneous ulnar nerve transposition

What is the description for medical collateral ligament reconstruction?

Description: The medial collateral ligament can become unstable as a result of overhead activities or throwing. Most commonly, there is a gradual onset of medial elbow pain due to repetitive stresses on the ligament. For athletes participating in overhead or throwing sports, poor mechanics, inflexibility, or fatigue can eventually lead to muscle strain, which places high valgus stresses on the MCL. A tear of the ulnar collateral ligament can usually be diagnosed through a history and physical examination. A magnetic resonance (MRI) scan and x-ray may also be done to further assess the condition of the structures in the patient's elbow, but these tests are not the sole basis for a diagnosis.

What are the complication of UCL injury?

Direct repair of the ligament is generally not indicated. Reconstruction of the UCL (eg, repair of the anterior bundle) is the more common approach to surgical treatment, with use of a palmaris longus tendon autograft. The graft is pulled through bone tunnels in the medial epicondyle of the humerus and the sublime tubercle of the ulna in a figure-8 pattern. Ulnar nerve transposition may be undertaken at the time of surgery,

What is the docking procedure for the UCL?

Dr. David Altchek and colleagues at the Hospital for Spe- cial Surgery83 have modified the reconstructive procedure through a muscle-splitting approach with a single hu- meral tunnel and Krackow stitch tendon fixation called the "docking procedure."

How long do you stop throwing with a UCL ligament injury?

During the initial 2 to 6 weeks after injury, throwing should be ceased while the thrower focuses on cryother- apy, regaining motion, and strengthening the shoulder girdle, scapular stabilizers, and rotator cuff musculature.

As you are providing sideline coverage at a baseball game you observe the third baseman take a hard hit ground ball to the nose during the 1st inning. Upon off the field assessment, you observe mild epistaxis accompanied by clear fluid drainage. The athlete notes a sweet taste in his throat. He seems to be tolerating pain well & shows no signs of deformity or difficulty breathing. He is anxious to return to play. What is your most appropriate course of action?

Error on the side of caution & refer to otolaryngologist as soon as possible for suspicion of cerebrospinal fluid leakage & risk of meningitis Incorrect: while these are great strategies to control typical epistaxis, greater suspicion for other conditions should be present. For a good review of facial injuries in sport, review Reehal (2010)

What is the pathophysiology of SICK

FIGURE 5. The pathophysiology behind why the dropped SICK scapula presents with coracoid pain is explained by the coracoid static malposition and the dyskinesis that it produces. Because of the ellipsoid shape of the thorax, as the scapula tilts anteriorly, protracts, and abducts, it tends to ride "up and over" the top of the thorax.2,3 As it does, the coracoid tilts anteroinferiorly and moves laterally from the midline. The pectoralis minor and short head of the biceps become adaptively tight. This tightness increases the scapular malposi- tion, lowers the leading edge of the acromion, and decreases the ability to achieve full forward flexion of the arm. Impingement-like symptoms result from the anteroinferior angulation of the acromion because of scapular protraction.

What is the history of cubital tunnel syndrom?

Feindel and Stratford first used the term cubital tunnel in 1958.[1, 2] They emphasized that the ulnar nerve is compressed at the elbow because of anatomic peculiarities to that region. In 1898, Curtis performed the first published case of management for ulnar nerve neuropathy at the elbow, which consisted of a subcutaneous anterior transposition.[3]

What are the no contraindications to return to play for brachial plexus injury?

Fewer than three episodes of prior burner/stinger last < 25 hours with full cervical motion without any evidence of neurologic deficit.... one episode of transient quadriparesis/quadriplegia with full range of cervical motion. no evidence of residual neurologic deficit, and no evidence of herniated disk or instabilty.

What is the follow up for lateral humeral condyle fracture?

Follow-up care is performed at 7 days with AP and lateral radiographs, with arm out of plaster, to assess maintenance of reduction. A long-arm cast is applied in supination. The patient then is seen at 3-4 weeks postoperatively, and the K-wires are removed (see image below). Physical therapy is initiated, and gentle active range of motion (ROM) is begun. The patient is next seen at 6 weeks, and AP and lateral radiographs are obtained. Return to full activity is allowed once the fracture is healed radiographically.[10, 11, 12]

What is the prognosis for medial neuropathy?

For mild cases, conservative treatment is usually adequate. Surgical decompression usually produces good results. Patients with advanced thenar muscle atrophy usually do not recover fully after surgical decompression. CTS that is caused by or aggravated by an underlying disease (eg, diabetes) has a worse prognosis.

What week do you add RTC/bicep strengthening for open anterior shoulder instability

For open stabilization proce- dures involving detachment or reattachment of the subscapularis, resistance exercises for the subscapularis muscle are introduced during the sixth postoperative week).

What is the problem with coronoid fracture?

Fractures of the coronoid rarely occur in isolation. They usually occur in combination with a radial head fracture, a dislocation of the elbow, or both (terrible triad of the elbow).[1] Large coronoid fractures often are associated with persistent elbow instability even after reduction of the dislocation. Recently, the trend has been toward fixation of these injuries to restore stability and initiate an early range-of-motion program.

What the are contraindication for lateral humeral condyle fracture?

Fractures that are not greatly displaced and are identified on a delayed basis greater than 3 weeks should not undergo surgical intervention. Healing has progressed to a point that extensive dissection would be required to achieve reduction leading to a high incidence of avascular necrosis of the lateral condyle.

What are the indication for olecranon fracture?

Fractures with significant displacement (>2 mm) or comminution may require surgical intervention.

What is the future of Monteggia Fracture?

Future research will help to identify appropriate treatment protocols to achieve optimum long-term outcome. The most important step is to educate the specialist, emergency physician, and primary care physician to correctly diagnose and treat these injuries.

What are the factor that lead to GHJ stability?

Glenohumeral stabil- ity is the result of the interplay between multiple anatomical structures that include the capsule, liga- ments, muscles, tendons, osseous configuration, and glenoid labrum.

What are the 4 classification of Sprengel Deformity?

Grade 1: The deformity is very mild. The shoulders are almost level, and the deformity cannot be noticed with the clothes on. Grade 2: The deformity is mild. The shoulders are almost level, but the superomedial portion of the high scapula is visible as a lump. Grade 3: The deformity is moderate. It is visible, and the affected shoulder is elevated 2-5 cm higher than the opposite shoulder. Grade 4: The deformity is severe. The scapula is very high, with the superomedial angle at the occiput, with neck webbing and brevicollis.

What are the special testing/imaging for SAI?

Hawkin kennedy.. Neer impingement test.... painful arch... empty can test... X-ray/MRI

What is the etiology for clavicle fracture?

Historically, clavicle fractures were thought to result from a fall onto an outstretched hand, but more recent work has revealed that, in actuality, the most common mechanism for clavicle fractures is a fall directly onto the shoulder. A fall onto an outstretched arm was, in fact, the cause of the fracture in only 6% of cases. Data from Nowak et al and Nordqvist and Petersson cite a fall onto the shoulder as the most common cause of clavicle fractures.[9, 10] Stanley et al found that about 70% of clavicle fractures were the result of trauma from traffic accidents; Pannike confirmed this statistic.[11, 12] Increasing traffic density causing more motor vehicle accidents is the proposed explanation for this statistic.

What is the management of subacrominal bursitis?

Ice, heat, and rest should be utilized until acute pain subsides. Inital management of subacrominal bursitis involves immobilization with a sling and NSAIDS. Physical therapy exercises involving strengthening and range of motion can aid in recovery.

What is the management of a Humeral fracture

If the bone are in appropriate alignement, the initial treatment consists of ice and medication to relieve pain and swelling. Immobilizations for a minimum of 6 weeks is recommended. Severe fractures, fractures that are displaced, may require surgery to restore and maintain the joint to its normal position. After immobilizations, stretching and strengthening of the injured and weakend joints and surrounding muscle are neccessary.

What are the contraindication with medial humeral fracture?

If the patient is unable to tolerate a long surgical procedure due to polytrauma, closed reduction and cast immobilization with 90° of flexion is an option.

What is the background for lateral humeral condyle fracture background?

In 1883, Stimson first described the fracture patterns in lateral condyle fractures in his book Treatise on Fractures.[1] He described the fracture as beginning in the lateral metaphysis proximal to the condyle, coursing distally, and exiting through the articular surface through the medial trochlear notch or through the capitellotrochlear groove. In 1955, Milch recognized the significance of these fracture patterns as they related to elbow stability.[2] Thus, the fracture patterns of the lateral condyle bear his name and are classified as either Milch I or Milch II fractures.[3, 4, 5]

What is the surgical intervention for little elbow syndrome?

In cases of olecranon impingement syndrome, surgery is often needed early in the treatment course to remove loose bodies from the joint. Surgery is also indicated early in cases of posterolateral rotatory instability. Many times, the collateral ligaments must be repaired before rehabilitation can begin. Arthroscopy or open arthrotomy is often needed to remove loose bodies and scar tissue in radiocapitellar chondromalacia cases.

What is the future and controversies of coronoid fracture?

In the past, coronoid fractures were treated with a longer period of immobilization (3-4 wk) in greater degrees of flexion, and this was believed to be a better alternative than operative treatment. However, with increasing understanding of the contribution of the coronoid to stability of the elbow, the trend increasingly is toward operative stabilization of these injuries and initiation of an early, protected range-of-motion program to avoid the most dreaded complication of these injuries, which is stiffness.

What are the indications of cubital tunnel syndrome?

Indications for in situ decompression of the ulnar nerve at the elbow are as follows: Mild ulnar nerve compression Documented mild slowing on an electromyograph (EMG) as the ulnar nerve passes into and through the proximal FCU Absence of pain around the medial epicondyle A nerve that does not sublux with elbow flexion Normal osseous anatomy and retrocondylar groove at the elbow and findings at surgery consistent with compression under the fibrous arcade[31] Simple decompression is easy to perform, and the complication rate is low. In contrast to other methods, in situ decompression avoids damage to the vascular supply of the nerve. The operation is less traumatic to the patient, and the documented results show this procedure to be as successful as other decompression procedures. In situ decompression requires minimal or no postoperative immobilization.[32, 33, 34] The advantage of in situ decompression is the ability to release the ulnar nerve in areas of compression with minimal disturbance of the blood supply. This procedure avoids subluxation of the ulnar nerve, which may lead to a recurrence of symptoms secondary to repeated contusion of the nerve as it snaps over the medial epicondyle. The disadvantages of simple decompression are the potentially higher recurrence rate and the risk of continued subluxation of the ulnar nerve over the medial epicondyle, if that was present preoperatively. Medial epicondylectomy The best indication for a medial epicondylectomy is nonunion of an epicondyle fracture with ulnar nerve symptoms. Other indications include a poor bed for the ulnar nerve in the retrocondylar groove or ulnar nerve subluxation.[35, 36] The advantage of a medial epicondylectomy is that it provides a more thorough decompression of the ulnar nerve than a simple release. This results in a minitransposition of the ulnar nerve. Compared to an anterior transposition, a medial epicondylectomy better preserves the blood supply to the nerve, results in less injury to the nerve, and preserves the small proximal nerve branches that might be sacrificed with an anterior transposition.[37] The disadvantage of a medial epicondylectomy is that it allows greater migration of the ulnar nerve with elbow flexion. A potential exists for elbow instability if the collateral ligaments are damaged. Bone pain and nerve vulnerability at the epicondylectomy site may occur. Compared to a simple decompression, the possibility of elbow stiffness or the development of an elbow flexion contracture is greater. In addition, a medial epicondylectomy is often a poor choice for athletes who throw because of the significant stresses placed on the medial aspect of the elbow joint. Anterior transposition The three types of anterior transposition are subcutaneous, intramuscular, and submuscular. Indications for an ulnar nerve transposition are the following: An unsuitable bed for the nerve secondary to the presence of osteophytes A tumor A ganglion An accessory anconeus epitrochlears muscle Heterotopic bone Significant bursal tissue or other mass Significant tension on the ulnar nerve as implicated with a positive elbow flexion test result or symptoms aggravated by activities requiring flexion Subluxation of the ulnar nerve with elbow flexion A deformity at the elbow secondary to a valgus elbow or a tardy ulnar palsy[38, 1] The presence of valgus instability at the elbow emedicine.medscape.com/article/1231663‐overview#showall 4/12 9/3/12 Cubital Tunnel Syndrome Softtissue coverage must be adequate for the transposition of the nerve and a medial elbow that is not subjected to repeated minor trauma. The advantage of an anterior transposition is that it moves the ulnar nerve from an unsuitable bed to one that is less scarred. The nerve is effectively lengthened a few centimeters with transposition. This decreases tension on the nerve with elbow flexion.[39] The disadvantage of an anterior transposition is that it is more technically demanding than a simple ulnar nerve decompression. The risk of complications is increased when the nerve is moved from its natural bed, and there is a potential for devascularization of the ulnar nerve. With an anterior subcutaneous transposition, several modifications are used to maintain the nerve in the transposed position. These include the use of epineural sutures; the creation of a fascial dermal or myofascial sling;[40, 41] and the creation of a subcutaneous fascial sling. A subcutaneous transposition may be the procedure of choice in athletes who throw and do not have muscular atrophy. These athletes may lose forearm strength from a submuscular transposition and a simple decompression may not provide adequate relief of symptoms. The advantage of a subcutaneous transposition is that it is easy to perform. It is a good procedure when subluxation and traction on the nerve are contributing to the patient's symptoms.[42] The disadvantage of a subcutaneous transposition is that the nerve may be hypersensitive after surgery because of its new superficial location. The potential exists for disruption of the ulnar nerve blood supply with the transposition. Intramuscular transposition is the least popular decompression method. It yields the fewest excellent results and is associated with the most recurrences with severe ulnar nerve compression. The advantage of an intramuscular transposition is that it buries the nerve deeply, yet provides a tunnel for the nerve to pass through. It also allows the nerve to be entirely surrounded by vascularized muscle tissue. The disadvantage of an intramuscular transposition is that it is a complicated procedure. It involves significant softtissue dissection. The risk of perineural scarring is increased, and the procedure may expose the nerve to repeated muscular contractions. A submuscular transposition offers the best results with the fewest recurrences with severe ulnar nerve compression.[43] A submuscular transposition is the best salvage procedure when previous surgery has failed because it places the nerve in an unscarred bed. It also works well for patients who are very thin, in whom a subcutaneous transposition may result in an area of hypersensitivity over the transposed nerve. Many consider it the procedure of choice for symptomatic athletes who throw. Contraindications for submuscular transposition include significant scarring or distortion of the elbow joint capsule, such as in a malunited fracture or in a patient who has undergone excisional arthroplasty. The disadvantage of a submuscular transposition is that it is a technically demanding procedure. Because of the extensive dissection involved, recovery for the patient is more difficult and the risk of elbow flexion contracture is 510%. Patients may also develop extensive scar formation from the procedure, and it is a difficult procedure to revise if the patient has a recurrence.

What is the MOI of inferior shoulder dislocation?

Inferior dislocations are rare and result from a hyperabduction force that causes the humeral neck to lever against the acromion. Diagnosing inferior dislocations is critical because of the high incidence of complications. Neurologic injuries (particularly axillary nerve lesions) are associated with inferior dislocations in as many as 60% of cases, vascular injuries occur in about 3.3% of cases, rotator cuff tears in occur in 80-100% of cases,[9] and greater tuberosity fractures and pectoralis major avulsions are also associated with inferior dislocations.

What is the presentation for inferior shoulder dislocation

Inferior dislocations lead to a condition known as luxatio erecta, which describes a classic presentation of the arm abducted 110-160° with the forearm resting on or behind the patient's head

What is Type 1 scapular dyskinesis

Inferior medial boarder prominence.... associated with SLAP lesion

What is the history of SLAP procedure?

Initial operative treatment of superior labral (labrum) lesions consisted of debridement only. This therapy is appropriate for type I and type III lesions, as well as for some type IV lesions, in which the biceps anchor is intact. However, in type II and in type IV lesions in which the biceps anchor is unstable, debridement alone yields poor results. Early attempts at arthroscopic fixation used a metal staple. This technique was discontinued because of concerns for chondral injury as well as the need for a second surgery for staple removal. For similar reasons, techniques involving a metal screw are no longer used. A transglenoid technique similar in concept to a transglenoid technique for a Bankart repair has been described; however, this is technically difficult and has fallen out of favor. Good results were initially reported with use of a bioabsorbable tack. However, complications, including implant breakage with resultant particulate debris and a foreign body reaction, have been reported. Suture anchors loaded with nonabsorbable suture is the preferred method of fixation.

What is the surgical Therapy for a SLAP lesion?

Initially, a complete diagnostic arthroscopy is performed. The rotator cuff should be carefully inspected for any partial thickness or full-thickness tears. The biceps anchor is inspected. Be aware of the potential normal variants as discussed earlier in this article (see Pathophysiology). Type III and type IV SLAP lesions are fairly obvious arthroscopically. The difficulty can sometimes come in differentiating a type I lesion from a type II as well as accurately diagnosing type II lesions and variants thereof. Type I lesions are often associated with a meniscoid superior labrum where the lateral aspect is draped over the rim of the glenoid superiorly and the attachment is more peripheral. This particular morphology is more susceptible to developing degenerative tears, which is the pathology observed in type I lesions. Care must be taken not to assume that this meniscoid labrum represents a displaced type II lesion. A probe is placed under the superior labrum, and a firm attachment is demonstrated. In inspecting the superior labral attachment, the key factor to evaluate is if whether more than 5 mm of superior glenoid is exposed under the labrum. A superior sublabral recess is often observed and is a normal finding. However, if this recess is greater than 5 mm, the biceps anchor is highly likely to be unstable.[36, 37] The superior labrum both anterior and posterior to the biceps root should be carefully probed. Placing the arm in approximately 70-90° of abduction and then progressively externally rotating the arm can demonstrate the peel-back sign, which is observed with type II posterior lesions as well as in combined anterior and posterior type II lesions. If more than 5 mm of the posterosuperior glenoid is uncovered or the biceps root at the level of the supraglenoid tubercle is uncovered with this maneuver, then the peel-back sign is positive and the superior labrum must be repaired. The peel-back sign is not usually observed with type II anterior SLAP lesions. A positive drive-through sign where the arthroscope can be easily passed from the superior aspect of the joint to the inferior recess without any manual distraction is observed with all 3 variants of type II SLAP lesions. This anterior pseudolaxity is usually resolved with repair of the SLAP lesion, and the drive-through sign is eliminated. Surgical treatment of a type I lesion consists of debridement. Similarly, in a type III lesion, the bucket-handle tear of the meniscus can be debrided because the biceps anchor is intact. In a type II lesion, the biceps anchor is repaired back down to the superior labrum with suture anchors. In type IV lesions, if less than 30% of the tendon is involved and the biceps anchor is intact, then the involved labrum and tendon can be resected. If more than 30% involvement is noted in an older patient, a biceps tenodesis can be performed.[38] In the younger more active individual, suture repair of the tendon, along with suture anchor repair of the labrum, should be performed. Various techniques have been described to repair the superior labrum arthroscopically.[39, 40] These include the use of metal staples, metal screws, bioabsorbable tacks, and a transglenoid technique. Metal staples and screws require a second surgery for removal and are no longer used. Good results have been reported with the use of bioabsorbable tacks; however, concerns over potential particulate debris and foreign body reaction have led many surgeons to use suture anchors loaded with nonabsorbable suture.

What is the physical presentation of little league elbow syndrome?

Inspection is important to note the carrying angle and any flexion contractures that may be present relative to the opposite side. During the initial examination, evaluate for muscle atrophy or hypertrophy, bony deformities, or the presence of swelling and ecchymosis.[1, 18, 19] Palpation of bony structures should include both epicondyles, the olecranon process, the capitellum, and the radial head. Soft-tissue palpation should include the UCL (felt best with the patient's elbow in 50-70° of flexion), the biceps tendon, the triceps tendon, and the flexor-pronator and extensor-supinator muscle complexes. Strength testing of the various muscles should be performed. Neurologic testing should include evaluation of the ulnar nerve. Palpation for tenderness, stability testing, and a Tinel test via percussion over the ulnar groove for paresthesias consistent with ulnar neuritis constitute a thorough examination. Special tests include valgus stress testing to evaluate injury to the UCL. The patient may be prone, supine, or upright. The stress test should be performed with the elbow in 20-30° of flexion with a valgus force exerted on the elbow. Opening up on the injured side, compared with the opposite uninjured side, is most reflective of an injury to the UCL. Pain without instability during valgus stress testing is more commonly seen with little league elbow syndrome. Two special tests to note are the milking maneuver , which is performed with the patient seated, and the valgus extension overload test. For the milking maneuver, the examiner grasps the thrower's thumb with the arm in the cocked position of 90° of shoulder abduction and 90° of elbow flexion. Then the examiner applies a valgus stress by pulling down on the thumb. For the valgus extension overload test, the examiner stabilizes the humerus from the outside and then pronates the forearm during extension while applying valgus stress. Pain is more likely associated with posterior impingement if this test result is positive. Conduct a complete examination of the neck, shoulders, wrist, and hand. A general inspection should include an assessment of height and weight, because a larger body habitus is associated with an increased risk of elbow injury. Perform a complete neurologic and vascular examination of the neck and upper extremity

What is the pathology/MOI of posterior/internal impingement.

Internal impingement is commonly described as a condition characterized by execessive or repetitve contact between the posterior aspect of the greater tuberosity of the humeral head and the posterior superior aspect of the glenoid rim when the arm is placed in extreme ranges of abductions and external rotation. This ultimately leads to impingement of the rotator cuff tendons and the glenoid labrum. This can lead to the PASTA leasion.

What are the special tests for volkmann contracture?

Intracompartimental Pressure Monitoring

What is the etiology of medial epidcondylitis?

Its pseudonym 'golfer's elbow' is easily remembered but misleading as 90 to 95% of all cases do not involve sportsmen (level of quality A2)[3] (level of quality D)[4]. Symptoms may be produced by sudden violence to these tendons by repeated usage or by chronic strain. In many cases trauma at work had been identified as the cause of the symptoms (level of quality A2)[3]. More specific occupational physical factors associated with medial epicondylitis are forcefull activities among men and with repetitive movements of the arm among women. Current smoker and former smoker are also associated with medial epicondylitis, so do patients who suffer from diabetes type 2 (level of quality A2)[3].

What are the mechanics for bicep load 2 test?

Kim et al.8 described a second biceps load test in 2001 for the assessment of SLAP lesions in shoulders without re- current dislocation. "The test is conducted with the pa- tient in the supine position. The examiner sits adjacent to the patient on the same side as the shoulder and grasps the patient's wrist and elbow gently. The arm to be exam- ined is elevated to 120° and externally rotated to its max- imal point, with the elbow in the [sic] 90° flexion and the forearm in the supinated position. The patient is asked to flex the elbow while resisting the elbow flexion by the examiner. The test is considered positive if the patient complains of pain during the resisted elbow flexion and also considered positive if the patient complains of more pain from the resisted elbow flexion regardless of the degree of pain before the elbow flexion maneuver. The test is negative if pain is not elicited by the resisted elbow flexion or if the preexisting pain during the elevation and external rotation of the arm is unchanged or diminished by the resisted elbow flexion." In the 2001 report, 127 shoulders were evaluated ar- throscopically. There were 38 positive tests. The sensi- tivity was 90%, the specificity was 97%, the positive pre- dictive value was 92%, and the negative predictive value was 96%.

What is the epidemiology/etiology for lateral epicondylitis?

Lateral epicondylitis is classified as an overuse injury. Overuse of the muscles and tendons of the forearm and elbow together with repetitive gripping or manual tasks can put too much strain on the elbow tendons. These gripping or manual tasks require manipulation of the hand that causes maladaptions in tendon structure that lead to pain over the lateral epicondyle. Mostly, the pain is located anterior and distal from the lateral epicondyle. Epicondylitis occurs at least five times more often and predominantly occurs on the lateral rather than on the medial aspect of the joint, with a 4:1 to 7:1 ratio. This injury is often work-related, any activity involving wrist extension, pronation or supination during manual labour, housework and hobbies are considered as important causal factors. A systematic review identified 3 risk factors: handling tools heavier than 1 kg, handling loads heavier than 20 kg at least 10 times per day, and repetitive movements for more than 2 hours per day. [2] Other risk factors are overuse, repetitive movements, training errors, misalignments, flexibility problems, aging, poor circulation, strength deficits or muscle imbalance and psychological factors. There are several opinions concerning the cause of lateral epicondylitis: Inflammation: • Although the term epicondylitis implies the presence of an inflammatory condition, inflammation is present only in the earliest stages of the disease process. [3] Microscopic tearing: • Nirschl and Pettrone attributed the cause to microscopic tearing with formation of reparative tissue (angiofibroblastic hyperplasia) in the origin of the extensor carpi radialis brevis (ECRB) muscle. This micro- tearing and repair response can lead to macroscopic tearing and structural failure of the origin of the ECRB muscle. • That microscopic or macroscopic tears of the common extensor origin were involved in the disease process, was postulated by Cyriax in 1936. • The first to describe macroscopic tearing in association with the histological findings were Coonrad and Hooper. • Histology of tissue samples shows "collagen disorientation, disorganization, and fibre separation by increased proteoglycan content, increased cellularity, neovascularization, with local necrosis." Nirschl termed these histological findings bangiofibroblastic hyperplasia. The term has since been modified to bangiofibroblastic tendinosis. He noted that the tissue was characterized by disorganized, immature collagen formation with immature fibroblastic and vascular elements. This grey, friable tissue is found in association with varying degrees of tearing involving the extensor carpi radialis brevis. Degenerative process: The histopathological features of 11 patients who had lateral epicondylitis were examined by Regan et al. They determined that the cause of lateral epicondylitis was more indicative of a degenerative process than an inflammatory process. The condition is degenerative with increased fibroblasts, vascular hyperplasia, proteoglycans and glycosaminoglycans, and disorganized and immature collagen. Repetitive eccentric or concentric overloading of the extensor muscle mass is thought to be the cause of this angiofibroblastic tendinosis of the ECRB. www.physio‐pedia.com/Lateral_Epicondylitis 3/8 9/3/12 Lateral Epicondylitis ‐ Physiopedia, universal access to physiotherapy knowledge. Hypovascularity: Because this tendinous region contains areas that are relatively hypovascular, the tendinous unit is unable to respond adequately to repetitive forces transmitted through the muscle, resulting in declining functionaltolerance. [4]

What week do you add RTC/bicep strengthening for anterior shoulder instability

Light resis- tance exercises for the rotator cuff and biceps brachii muscles are introduced during the fourth postoperative week

What are the complication for medial humeral fracture?

Medial condyle fracture Nonunion with a thickening deformity at the fracture site can occur with inadequate reduction, fixation, or immobilization. Catgut suture as a means of internal fixation has proved to be inadequate, as it has often resulted in this complication. Malunion can result in loss of motion or angulation. As with nonunion, this can result from inadequate fixation or premature mobilization. Some minor loss of motion (flexion and extension) is a common sequela of many displaced medial condyle fractures. The degree of loss is usually minimal and does not decrease function. When the loss is related to another complication, such as nonunion, malunion, or heterotopic ossification, it can be significant. A progressive cubitus varus deformity may develop due to growth inhibition or avascular necrosis of the medial humeral condyle. This also can result from premature closure of the physis. Forty degrees of varus angulation was reported in 1 case that went untreated for 4 years. This was treated with a supracondyle wedge osteotomy to restore range of motion and correct the cubitus varus deformity. Avascular necrosis of the epiphysis can be the result of loss of blood supply during an overaggressive soft-tissue dissection in attempts to adequately expose the fracture. The blood supply to the epiphysis is through the soft-tissue attachments at the medial epicondyle. A valgus deformity also can result from imperfect restoration of position. This is usually related to an overgrowth of the medial condyle. Heterotopic ossification can result in severe loss of flexion and extension. This is often associated with delayed fixation and closed head injuries. Myositis ossificans can result from overaggressive physical therapy with passive range of motion. Pronation and supination are usually not affected. Concurrent injury to the radial head may result in decreased motion. Injury to the ulnar nerve may result in a partial clawhand, muscle weakness, and partial loss of sensation. If necessary, transposition of the nerve can be performed to reduce tension and prevent further injury. Partial or complete recovery may take months. Misdiagnosis or delay in diagnosis or treatment increases the risk of impairment and complications. Medial epicondyle fracture The 2 main complications associated with medial epicondyle fractures are failure to recognize incarceration into the joint with functional loss and ulnar or median nerve dysfunction. Most of the other complications associated with medial epicondyle fractures are considered minor and do not result in a loss of function. The main complication with an unrecognized medial epicondyle fracture is loss of motion secondary to impingement of the fragment. The second major complication involves ulnar nerve dysfunction that varies 10-16%. If the fragment is incarcerated in the joint, the incidence of ulnar nerve dysfunction can reach 50%. More profound ulnar nerve dysfunction has been observed to occur with manipulative reduction attempts, especially if closed manipulation of an incarcerated fragment is attempted. A median nerve injury may occur as well; however, this is more common with an associated elbow dislocation, as depicted in the image below. inor complications include radiographic nonunion of the medial epicondyle fragment in cases in which the fracture is treated closed. Functionally, any limitation from this radiographic finding does not appear to exist. A loss of elbow extension of 10-15% can be expected in up to 20% of cases, and this appears to be correlated more with prolonged immobilization than the fracture itself. Myositis ossificans has been described as a rare occurrence and has been correlated with repeated manipulation to reduce an incarcerated fragment. A significant alteration in the carrying angle of the elbow has not been demonstrated in long-term studies and does not appear to be a major issue with these fractures.

What is Type 2 scapular dyskinesis

Medial boarder prominence... associated with SLAP lesion

What is the background information for medial humeral fracture?

Medial condyle fractures involve a fracture line that extends through and separates the medial metaphysis and epicondyle from the rest of the humerus (see image below). By definition, the fracture line must involve the trochlear articular surface Medial condyle fractures must not be confused with medial epicondyle fractures (depicted in the image below) that involve the medial column but are extra-articular. These 2 fracture patterns are separate entities and are treated differently. Both fracture patterns may be difficult to diagnose in young children, especially before the secondary ossification centers have formed

What is the medical management of lateral epicondylitis?

Medical management of lateral epicondylalgia has historically included NSAIDs, advice regarding rest, use of an elbow counterforce brace, and corticosteroid injection(s). Histologic research and the discovery of the true nature of this condition has led to more appropriate diagnosis and treatment in regard to the degeneration that is occurring at the site. Current medical treatment and research involves the use of interventions like prolotherapy, autologous blood injections, and even surgery (usually after failed conservative treatment for >6 mos)(Emedicine). Low level laser therapy has also been found to be an ineffective treatment modality for lateral epicondylalgia. [16]

What are the mechanics of the pain provocation test

Mimori et al.12 described another SLAP provocative test in 1999 (depicted in Fig. 7.). "The new pain provocation test was performed with the patient in the sitting position. Figure 6. The active compression test. A, thumb down; B, palm up. During testing, the abduction angle of the upper arm was maintained at 90° to 100°, and the shoulder was rotated externally by the examiner. This maneuver is similar to the anterior apprehension test. The new pain provocation test was performed with the forearm in two different po- sitions: maximum pronation and maximum supination. "We evaluated the severity of provoked pain based on the subjective rating by the patients themselves when the shoulder was rotated externally with the forearm in the two positions. Patients were asked 'in which position of the forearm do you feel more severe pain, in pronation or positive, a click had to be elicited, which is not the test as described by the authors (see previous discussion).

What is the epidemiology for Monteggia Fracture?

Monteggia fractures constitute less than 5% of forearm fractures, with published literature supporting 1-2%.[6, 7] Of the Monteggia fractures, Bado type I is the most common (59%), followed by type III (26%), type II (5%), and type IV (1%). Monteggia fractures are one third as common as the more familiar Galeazzi fractures.

What is the outcome for clavicle fracture?

Most fractures treated nonoperatively heal, with variable amounts of cosmetic deformity. For results or recent studies examining patterns of displacement and clinical outcomes, see Indications, above.

What are the problems with clavicle fracture.

Multiple attempts have been made to devise a classification scheme for clavicle fractures. The most common classification system is that of Allman, in which the clavicle is divided into thirds.[7] In the Allman system, group I fractures are middle third injuries, group II fractures are lateral third injuries, group III fractures are medial third injuries. This classification scheme is still used but has been revised to include many subtypes of clavicle fractures. Neer made a significant revision to the Allman classification scheme. Lateral clavicle fractures were further divided into 3 types based on the location of the clavicle fracture in relation to the coracoclavicular ligaments: Type I fractures occurred medial to the coracoclavicular ligaments. Type II fractures occurred at the level of coracoclavicular ligaments, with the trapezoid remaining intact with the distal segment. Type III injuries occurred distal to the coracoclavicular ligament and entered the acromioclavicular (AC) joint. The reason for this modification is that lateral clavicle fractures behave differently depending on the exact location of the injury. The Neer type II fracture was further divided into type IIA, in which the conoid and trapezoid ligaments both remain attached to the distal fragment, and type IIB, in which the conoid ligament is torn.[8] Other classification schemes have been presented since then; however, the Allman classification scheme with the Neer modification is the most commonly used and is listed in detail below. Group I - Fracture of middle third Group II - Fracture of the distal third Type I - Minimally displaced/interligamentous Type II - Displaced due to fracture medial to the coracoclavicular ligaments IIA - Both the conoid and trapezoid remain attached to distal fragment IIB - Either the conoid is torn or both the conoid and trapezoid are torn Type III - Fractures involving articular surface Type IV - Ligaments intact to the periosteum with displacement of the proximal fragment Type V - Comminuted Group III - Fracture of the proximal third Type I - Minimal displacement Type II - Displaced Type III - Intraarticular Type IV - Epiphyseal separation (observed in patients aged 25 y and younger) Type V - Comminuted

What are the signs and symptoms of cubital tunnel syndrome?

Nerve compression results in paresthesias along the ulnar nerve distribution affecting the small finger and ulnar side of the ring finger. In addition, with long standing cubital tunnel syndrome, residual motor weakness of the ulnar nerve innervated intrinsic muscles may be present. Severe, prolonged ulnar nerve compression may result in clawing of the ring and small fingers.

What are the complication fo SLAP?

Neurologic injury is the most common complication reported after arthroscopic shoulder surgery. These complications can be minimized by careful attention to patient positioning as well as a thorough knowledge of the neurovascular anatomy of the shoulder. When placing the patient in the beach-chair position, ensure that the head and neck are in a neutral position. Either hyperflexion or hyperextension could potentially lead to a neurologic injury. When making the Neviaser portal, care must be taken to avoid too acute an angle or placing the portal too medial to avoid injury to the suprascapular nerve. When placing the suture anchors, ensure that the anchors are well seated below the subchondral bone. If the anchors are too prominent, they may cause a chondral injury. The sutures should be tugged on to ensure that the anchors are well seated. Anchor migration, apart from compromising the repair, can result in significant chondral injury.

What is the surgical for olceranon fracture?

Nonoperative care for simple fractures is usually successful. However, fractures with significant displacement (>2 mm) or comminution may require surgical intervention. Excision and triceps advancement may be indicated for severely comminuted fractures or for patients with osteoporotic bone. Open reduction and internal fixation is preferred for displaced intra-articular fractures. Intramedullary screw fixation, with or without a wire or cable, is the most secure. Plate fixation is recommended for extensive comminuted or unstable oblique fractures not amenable to other types of treatment. Plate fixation also may be preferable in the face of an associated coronoid fracture.[4, One study retrospectively reviewed the outcome of 18 patients who underwent locking-plate osteosynthesis after open reduction for comminuted olecranon fractures. The study results found that in all cases, complete union was achieved. The data conclude that while the risk of limited elbow motion is high in cases with concomitant injuries, locking plates are an additional and often successful option for olecranon fracture fixation.[13] Preoperative Details

What are the contraindication for olecranon fractures?

Nonoperative treatment is often desirable in patients with significant associated medical conditions. Contused soft tissue healing is of paramount importance. Nonoperative treatment of even significantly displaced olecranon fractures in patients with severe medical illness, steroid use, or dementia is reasonable.

what is the medication summary for medial neuropathy?

Nonsteroidal anti-inflammatory medications (NSAIDs) are frequently prescribed for this condition; caution patients to watch for the usual adverse effects. Short-term diuretic treatment may be helpful in patients with limb swelling. A study by Chang et al in patients with mild-to-moderate CTS found that short-term, low-dose oral steroid treatment (prednisolone, 20 mg qd x 2 wk, then 10 mg qd x 2 wk) was more effective than treatment with a diuretic or an NSAID.[50] A follow-up study found that 2 weeks of prednisolone provided long-term results comparable to those with 4 weeks of treatment.[51]

What is the physical of ulnar neuropathy?

On physical examination, numerous findings offer clues to the existence of ulnar compression. In addition to assessing sensation and testing individual muscle strength, inspection of the hand may reveal a clawed posture (called main en griffe in French). Several factors contribute to the clawed appearance. Wasting of the intrinsic muscles of the hand makes it look bonier. The fourth and fifth digits extend at the metacarpal phalangeal joint because the extensors at that joint are radially innervated, whereas the flexors are innervated by the ulnar. Also, the fifth digit deviates slightly in the medial direction because, as explained for Wartenberg sign, the muscle that extends the fifth digit at the metacarpal phalangeal joint is radially innervated and it inserts on the ulnar side of the joint. The fourth and fifth interphalangeal joints flex because for them the extensor muscles are also ulnar and the natural tension of the muscles and tendons in the absence of strong muscle activity in either direction leads to flexion. The first 3 digits are extended at both the metacarpal phalangeal joints and the interphalangeal joints because of the unopposed radial nerve innervation. All these factors make the hand look somewhat like a claw. A different interpretation of the posture is that it looks like the hand gesture that a Catholic priest makes in the process of conferring a blessing, and thus it is sometimes called the benediction sign or the benediction hand. Froment sign is an observable sign that correlates with the complaint of weakness of the ability to pinch normally between the first and second digits. This sign is sometimes elicited by asking the patient to grasp a piece of paper between the thumb and index finger. Ordinarily, the grasp is tight and the patient makes heavy use of the adductor pollicis to adduct the thumb and the first dorsal interosseus to move the index finger. In addition to overt weakness of the pinch, the examiner also notes that the thumb flexes at the interphalangeal joint because the flexor pollicis longus activates in an attempt to compensate for the weakness. Thus, in addition to the weakness, the examiner sees the flexion of the tip of the thumb. Ulnar neuropathy at the elbow Positive Tinel sign at the elbow The examiner taps with a reflex hammer over the ulnar nerve in the ulnar groove and a little further distal over the cubital tunnel. The test is positive if the patient experiences definite paresthesias in the ulnar portion of the hand, especially the last 2 digits. This test is not considered highly sensitive, but it is considered to be quite specific if performed properly (eg, not hit too hard). If the examiner hits hard enough, many normal individuals experience paresthesias in the fourth and fifth digits. Assuming the complaint is unilateral, the opposite side is a good control for this. Sometimes palpating the nerve in the ulnar groove may produce a similar result. Atrophy and muscle weakness The most important ulnar hand muscles to test are the first dorsal interosseous and the abductor digiti minimi (abductor digiti quinti). In the forearm, the flexor digitorum profundus of the fourth and fifth digits (which flexes the distal phalanges of those fingers) and the flexor carpi ulnaris (flexion at the wrist in the ulnar direction) are valuable to examine. Of these latter 2 muscles, it is not uncommon for the flexor carpi ulnaris to be spared in ulnar lesions near the elbow, especially the lower (more distal) lesion near the elbow. Sparing occurs because the branch to the flexor carpi ulnaris splits off from the main trunk prior to (ie, above or proximal to) the compression.[33] The ulnar muscles should not be examined in isolation from other muscles. In particular, several key muscles with C8/T1, lower trunk, medial cord innervation should be examined, especially the abductor pollicis brevis (a thenar muscle typically involved with carpal tunnel syndrome, the major compressive median nerve neuropathy) and the median innervated long thumb and index finger flexors. If both the ulnar intrinsics hand muscles and the ulnar forearm muscles are involved, then an ulnar nerve lesion should be suspected in the region of the elbow (or, very rarely, above the elbow region). If the ulnar forearm muscles are spared, considering the possibility of a lesion at the wrist is reasonable, but extra caution is warranted in this case. Sometimes the forearm muscles are spared with a lesion near the elbow, especially if the lesion is in the lower elbow region in or around the cubital tunnel. Even for higher elbow lesions, there can be considerable selectivity in which muscles are affected because the ulnar nerve is organized into a number of separate fascicles. Sometimes some fascicles are severely affected by whatever is pinching the nerve and other fascicles are unaffected. If other C8/T1, lower trunk, medial cord muscles are affected, a C8/T1 radiculopathy or a brachial plexus lesion may be the cause. Ulnar neuropathy at or distal to the wrist Weakness of the interossei and hypothenar muscles only with no sensory loss: This would most likely be due to compression of the deep motor branch in the hand after it had separated from the superficial terminal sensory branch but before the branch to the hypothenar muscles had taken off. Interosseus weakness only with no sensory loss: This would most likely be due to compression of the deep motor branch after the branch to the hypothenar muscles has taken off. Weakness of the interossei and hypothenar muscles with sensory involvement in the fifth digit: This would suggest involvement in Guyon canal with compression of both the deep motor branch and the superficial terminal sensory branch. This might be said to be the typical or classical Guyon canal pattern. Pure sensory loss with normal dorsal ulnar cutaneous sensory nerve, normal palmar cutaneous sensory nerve, and normal motor responses: This would imply injury to the superficial terminal sensory branch alone, probably a compression distal to Guyon canal. Interossei weakness and sensory loss with preserved function in the hypothenar and dorsal ulnar cutaneous territories: This would imply a compression of the deep motor branch and the superficial terminal sensory branch distal to the point where the sub-branch to the hypothenar area (eg, the ADM) had split off the deep motor branch. Sensory examination Adding information from the sensory examination to that of the motor examination helps to localize the ulnar lesion. The image below, which has been discussed earlier in the context of the anatomy of the ulnar nerve, shows the ulnar sensory regions on the hand. Jacob et al have published a beautiful case report, complete with MRI pictures, on such a case. Although the area of the palmar cutaneous sensory nerve can extend a bit more proximal than shown, if the sensory involvement extends more than an inch above the wrist crease along the medial aspect of the forearm, the nerve roots (C8/T1) or brachial plexus most likely are involved (but in some cases this could be in addition to an ulnar injury). As previously noted, both the palmar cutaneous sensory branch of the ulnar and the dorsal ulnar cutaneous branch come off of the main ulnar branch above (proximal to) the wrist. Thus, a lesion exclusively at the wrist (Guyon canal) would miss these branches and the superficial terminal branch would be the only sensory involvement. However, a physician must be cautious in interpretation. Typically, neuropathic damage, whether generalized or related to nerve compression, affects (or is perceived to affect) the most distal parts of the nerves preferentially. A compression at Guyon canal might be perceived by the patient and might be detectable on examination only in the tips of the fingers. Thus, the compression would appear to be affecting only the superficial terminal branch

What are is the pathology/MOI of Olecranon Bursitis

Olecranon bursitis is a fairly common pathology and is where the olecranon bursa is inflamed. It is caused by an acute injury or a direct blow to the olecranon process. Another cause could be microtrauma such as rubbing their elbow over a table surface. In evenless common situations infection can cause the bursa to inflame as well.

How much limitation of ER with open bankart repair?

Open anterior stabilization is associated with a 12° loss of external rotation of the shoulder, probably secondary to shortening of the subscapularis tendon after detachment-reattachment.34

What is the indication for lateral humeral condyle fracture?

Operative management is essential for all displaced fractures and in those demonstrating joint instability or the potential for delayed joint instability. Stage I, or type I, lateral condyle fractures with less than 2 mm of displacement may be treated with immobilization. If there is a question of stability or the possibility of delayed displacement in these type I fractures, percutaneous pinning is recommended. If the degree of fracture displacement is questioned, anatomic reduction and surgical stabilization is needed. Open reduction is indicated for all displaced type II and type III fractures.

What is the standard x-rays for the elbow?

Our standard radiographic series of the thrower's elbow includes AP, lateral, axial, and two oblique views of the affected side. An oblique axial view with the elbow in 110° of flexion is helpful to demonstrate posteromedial olecra- non osteophytes.

What injuries could lead to tenderness of the radial head?

Palpation of the radial head is performed while the forearm is rotated passively. Pain with palpa- tion of the radial head may indicate a fracture, dislocation, OCD of the capitellum, or annular ligament injury

What angle should you palpate the UCL?

Palpation of the ulnar collateral ligament is performed with the elbow in approximately 50° to 70° of flexion, as this will move the overlying medial muscle mass anterior to the fibers of the ulnar collateral ligament

What is the presentation of a SLAP lesion?

Patients with superior labral (labrum) lesions often present describing a poorly defined pain that is posterior in location. They can also describe a painful popping and clicking similar to mechanical symptoms associated with a meniscal tear. Nonthrowing individuals may report a history of a fall either on an outstretched arm or in which a direct impact on the shoulder occurred. A history of a sudden deceleration injury, such as occurs when one loses control of a heavy object that is being carried, may be present. In a throwing athlete, a discrete injury with no prodromal period may be reported. In contrast, the athlete may not recall a specific injury and merely report a prodromal phase consisting of some mild posterior pain with a sense of posterior tightness. The patient's range of motion should be carefully assessed, especially in the throwing athlete.[16] Throwers often develop a loss of internal rotation in abduction. This loss of internal rotation with tightness of the posteroinferior capsule is thought to be a risk factor for the development of a SLAP (superior labrum, anterior and posterior) lesion. One should be especially cognizant of this entity in an individual who presents with loss of internal rotation at the expense of a 180° arc of motion with the arm abducted 90°. Burkhart and Morgan postulated that this finding defines a shoulder at risk of developing a type II SLAP lesion and the dead arm syndrome.[17] An acute SLAP lesion, especially a posterior type II lesion, can manifest as posterior shoulder pain in abduction and external rotation, decreased throwing velocity, and easy fatigability. This symptom complex has been labeled the dead arm syndrome. Multiple physical examination tests for a SLAP lesion have been described; however, correlation with arthroscopic findings has been poor. Furthermore follow-up studies by independent investigators have been unable to reproduce the high sensitivities, specificities, and positive-predictive values reported by the authors who originally described the tests.[18] In Snyder's initial report describing SLAP lesions, he used the biceps tension (Speed) test and the compression rotation test.[3] The Speed sign is positive when pain is elicited with resisted flexion of the fully supinated arm with the elbow extended and the arm flexed to 90°. The compression-rotation sign is performed with the patient supine, the shoulder elevated to 90°, and the elbow flexed to 90°. An axial load is then applied to the humerus to compress the glenohumeral joint, and the arm is rotated. Pain as well as mechanical symptoms elicited during this test are considered positive test results. Multiple other tests have been described.[19] The O'Brien sign, or the active-compression test, is elicited by first placing the arm in 90° of forward flexion and 10° of adduction.[20, 21, 22] The arm is then fully internally rotated into the thumbs-down position. The patient is then asked to resist downward pressure to the arm that is applied by the examiner. Differentiate deep-seated shoulder pain from that localized to the anterosuperior aspect of the shoulder because the latter is associated with acromioclavicular (AC) joint pathology. The test is then conducted again but with the arm in full supination; the pain should be decreased in this position as compared with the initial position for the test result to be considered positive. A positive Speed test as well as a positive O'Brien sign is thought to be consistent with an anterior type II SLAP tear. Kibler described the anterior slide test to help diagnose anterior SLAP lesions.[23] The patient is instructed to place both hands on the hips. The examiner stabilizes the scapula with one hand over the acromion. The other hand is used to axially load the humerus in anterior and superior direction. Pain with this motion is considered to be positive for an anterior based SLAP lesion. Kim et al described the biceps tension test II.[24] The shoulder is placed in 120° of abduction and full external rotation, and the elbow is flexed to 90° and fully supinated. The patient is then instructed to flex against resistance. Pain with this is consistent with a SLAP lesion. Kim et al also described the biceps tension test I to help determine the presence of a SLAP lesion in the patient with unidirectional anterior instability. An anterior apprehension test is first performed, which in this subgroup of patients is positive for instability. Resisted elbow flexion with the arm fully supinated should decrease the sensation of instability if the biceps superior labral complex is intact. In the presence of a SLAP lesion, no alleviation of the instability sensation occurs. The Jobe relocation test has been used to help diagnose posterior type II SLAP lesions.[25] The patient is placed in the supine position. The arm is placed in 90° of abduction and maximum external rotation. Pain in this position that is alleviated with a posteriorly directed force to the proximal humerus is consistent with a posterior type II lesion. Differentiate the sensation of pain in this test as opposed to that of instability found in an anteriorly unstable shoulder. Patients with type III and type IV lesions are more likely to report mechanical symptoms, although eliciting these on physical examination is often difficult.[26] Despite the multitude of described tests for a SLAP lesion, none has proven to be reliable to date. Follow-up independent studies have demonstrated poor sensitivities, specificities, and positive predictive values.[27] Check for rotator cuff impingement signs on examination.[28] The prevalence of rotator cuff tears, either partial or full-thickness, in patients with SLAP lesions has been noted to be in the 30-40% range.

What is the presentation with shoulder dislocation?

Patients with anterior dislocations usually present with the arm in slight abduction and externally rotated. The humeral head can often be palpated in the front of the shoulder. Internal rotation and adduction are limited. Movement is usually very painful as a result of muscle spasms. Patients with posterior dislocations present with the arm internally rotated and adducted. External rotation is severely limited. A posterior prominence is usually palpable, the anterior shoulder is flattened, and the coracoid process is more prominent. Historically, these dislocations have been missed or misdiagnosed as a frozen shoulder. Inferior dislocations lead to a condition known as luxatio erecta, which describes a classic presentation of the arm abducted 110-160° with the forearm resting on or behind the patient's head.[10, 11]

What is the presentation for posterior shoulder dislocation

Patients with posterior dislocations present with the arm internally rotated and adducted. External rotation is severely limited. A posterior prominence is usually palpable, the anterior shoulder is flattened, and the coracoid process is more prominent. Historically, these dislocations have been missed or misdiagnosed as a frozen shoulder

What is the pathology of the periscapular muscles of sprengel?

Periscapular muscles: The spinoscapular muscles may be fibrotic and contracted, with the trapezius muscle the most commonly affected.

What is the goal with phase 1 of elbow rehab?

Phase I Immediate motion phase Improve motion, diminish pain and inflammation, retard mm atrophy

What is the goal with phase 2 of elbow rehab?

Phase II Intermediate phase Enhance elbow and UE mobility, improve muscular strength and endurance, reestablish neuromuscular control

What is the patholigy and MOI of a posteiror dislocation?

Posterior are the second most common form of dislocation. although still only account for 3% of shoulder dislocations. These can occur during seizures and when falling onto an outstretched hand.

What are the special tests/images for posterior instability

Posterior drawer test. posterior apprehension test, jerk test... load and shift... MRI

What are the relative contraindications of a brachial plexus injury.

Prolong symptomatic burner/stinger ir transient quadriparesis lasting > 24 hours. ... Three or more previous episodes of either a stinger/burner or two episodes of transient quadriparesis/quadriplegia, the patient must have full cervical range of motion and strength without neck discomfort.

What is the activity for medial neuropathy?

Prolonged, repetitive use of the wrist (especially with force) may aggravate this condition. Provide an optimal work environment. Ergonomic changes in the workplace can be helpful. Work should be placed 10-12 inches in front of the eyes. Elbows should be postured at an angle of 85-100°. Shoulder should be in the vertical position, with abduction no greater than 20°. Wrist should be in the neutral position, without ulnar or radial deviation, with minimal flexion or extension. Environmental conditions such as temperature extremes and vibration should be minimized.

What is Type 3 scapular dyskinesis

Prominence of the superomedial border.. associated with rotator cuff tears.

What a good lower trap therex with ER strengthening?

Prone ER at 90 abd

What therex address the lower trap

Prone full can Y

What is the differential diagnosis for lateral epicondylitis?

Radial Tunnel Syndrome [9] Posterior Interosseous Nerve (PIN) Syndrome Elbow osteoarthritisOsteoarthritis Pain Loss of range of motion Fractures Distal Radial Fracture Radial Head Fracture Olecranon Fracture Cervical Radiculopathy Radiating arm pain corresponding to the dermatomes Neck pain Parasthesia Muscle weakness in myotome Reflex impairment/loss Headaches Scapular pain Sensory and motor dysfunction in upper extremities and neck

What are the postoperative details for clavicle fracture?

Recommendations for postoperative management of clavicle fractures consists of sling support for 2-4 weeks. During this time, the patient performs active range of motion exercises of the elbow and hand and pendulum exercises for shoulder motion. At about 2 weeks, the patient may begin low impact activities. After 3-4 weeks, the patient begins assisted range of motion of the shoulder and progresses to active range of motion as tolerated.

What is the surgical consideration for elbow dislocation?

Reduction Considerations: Ideally, the elbow should be radiographed before and after reduction to assist in assessment of associated bony injury. Complete examination includes assessment of neurovascular assessment, although associated neurologic injury is uncommon. Stretch neurapraxia of the ulnar nerve is the most common neurologic injury Examination includes assessment of the collateral ligaments via varus and valgus stress testing performed at 0 and 30 degrees of flexion.

What are the causes of UCL injury?

Repetitive throwing motions are the most common cause of UCL injury in the athlete. Traumatic valgus stress to the elbow during a fall or with the arm outstretched may lead to UCL rupture in association with elbow dislocation.

What are the complication of the lateral humeral condyle fracture?

Results of lateral condylar fractures are quite good when treated appropriately and in a timely fashion. Complications of lateral condylar fracture management include lateral condylar overgrowth or spur formation (30%), cubitus varus, nonunion, malunion, valgus angulation, ulnar nerve palsy, and avascular necrosis. These complications are either biologic problems, which arise from the healing process, or technical problems, arising from management errors.[13] Biologic-related problems include lateral condylar overgrowth or spur, which is due to overgrowth of the avulsed periosteal flap from the proximal fragment. This spur may give the appearance of a cubitus varus (pseudovarus) and cause difficulty in patients with a small carrying angle. In general, it should not cause a cosmetic or functional problem. This overgrowth usually undergoes remodeling and disappears over time. Cubitus varus occurs in approximately 42% of patients sustaining a lateral condylar fracture, regardless of treatment. The cause of cubitus varus is not clearly evident. However, it probably is due to lateral condylar physeal stimulation or to slight reduction incongruence. Deformities usually are mild, and surgical correction is not necessary. Technical-related problems of lateral condyle fracture treatment include delayed union, nonunion, and cubitus valgus. Delayed union of lateral condyle fractures usually occurs in patients treated nonsurgically. The elbow usually is not painful. The fragment usually is stable and undergoes uneventful union over time. A nonunion is considered present if no healing is evident at 12 weeks following injury. This may be caused by the pull of the extensor musculature, inadequate fixation or stabilization (immobilization), and failure to recognize the fracture. When the fragment is nondisplaced and is diagnosed relatively early, treatment with a compression screw can be performed. If the nonunion is well established, exploration and removal of the interposed fibrous tissue is recommended, followed by insertion of 1 or 2 compression screws. Perform bone grafting if significant fragment separation exists. Definitive treatment can safely be delayed until the patient becomes symptomatic or reaches skeletal maturity. Occasionally, a fishtail deformity of the distal humerus is seen because of the loss of ossific contact between the capitellum and trochlea. This results in a gap or a deficiency of the lateral trochlear buttress. This deformity usually does not result in any significant dysfunction and is treated nonoperatively. A cubitus valgus deformity may occur if there is nonunion or malunion of a lateral condyle fracture. The deformity rarely is caused by lateral condylar epiphysiodesis. In simple valgus malunion cases, a medial closing wedge osteotomy is performed. In cases of angular deformity and nonunion, treatment is complex and difficult. Address and stabilize the nonunion, and perform a medial closing wedge osteotomy to correct the angular malalignment. This may be performed simultaneously, or it may sequentially be staged. Care must be given to the amount of dissection performed to avoid avascular necrosis of the lateral fragment.[14] Avascular necrosis of the lateral fragment in lateral condylar fractures is iatrogenic and most often occurs in cases treated late or in nonunions and delayed unions. This complication is the result of aggressive dissection during open reduction. emedicine.medscape.com/article/1231199-‐‑treatment#showall 3/6 9/3/12 Lateral Humeral Condyle Fracture Treatment & Management Acute neurologic injuries are rare. Tardy ulnar nerve palsy occurs late in the treatment and follow-up of lateral condyle fractures and usually is due to cubitus valgus. The average time for presentation of ulnar nerve neuropathy is 22 years following the fracture. This ulnar neuropathy can be treated with ulnar nerve transposition, cubital tunnel release, or medial epicondylectomy. Although rare, myositis ossificans may occur.

What are the special tests for TOS

Roo's test... allen's test.. adson test

This type of force poses the greatest risk of of serious injury to the brain during a collision:

Rotational Correct: Rotational (angular) movements occur secondary to the fixation of the brain at the foramen magnum and craniospinal junction and the relative tethering of the midbrain as it passes through the tentorial hiatus. Energy directed to the head may cause transmission of force in a rotatory direction, often producing diffuse brain injury with shearing of the white-matter fiber tracts. Biomechanical studies concerning athletic injuries have suggested that angular head accelerations have a higher incidence of injury.

What is the pathology and MOI of bicep brachii rupture?

Rupture of the bicep tendon can be caused by several different mechanisms, Acute tears are caused by excessive loading or rapid stressing of the elbow joint such as during weightlifting activities. Partial tears can be the result of chronic impingement and fraying of the tendon under the acromion of the scapula and can also occur with rotator cuff injury.

What is the Patholigy and MOI of a subacrominal impingement?

SA is defined as the mechancial compression of the subacrominal structures between the coraco-acrominal arch and the humerus during active elevation of tha arm above shoulder high. SA occurs when the subacrominal space is narrowed and weak structures in that space get compressed. Several causes

What therex produces low capsular strain of the capsule for infraspinatus.

SL external rotation

What are the complication for shoulder dislocations

Shoulder dislocations result in various associated arthroscopic findings and various vascular and neurologic complications.[38] One must be astute when examining patients for neurovascular compromise, both prior to and after reduction attempts. Most patients with a first-time dislocation also have a Bankart lesion (ie, an avulsion of the anterior capsulolabral complex from the glenoid rim with disruption of the medial scapular periosteum).[39] Variations of this injury include a bony Bankart lesion, in which the labrum remains intact but a fracture occurs through the anterior glenoid rim. A Perthes lesion is similar to a Bankart lesion, except the medial scapular periosteum remains intact; thus, the labrum may appear normal on MRI and arthroscopy unless the arm is abducted and externally rotated away from the neutral position. An anterior labroligamentous periosteal sleeve avulsion (ALPSA) lesion differs from the Bankart lesion in that the anterior labrum is medially displaced. It heals in an abnormal position, leading to an incompetent anterior inferior glenohumeral ligament. Hill-Sachs lesions commonly occur and are compression fractures that result from impaction of the posterolateral humeral head against the anterior/inferior glenoid rim, which can occasionally result in a loose body.[40] One study examined whether failures of arthroscopic Bankart repairs related to Hill-Sachs lesions can be treated by "remplissage" (filling in) of the defect with rotator cuff tendon. The data noted no significant statistical difference in the range of motion between patients treated with arthroscopic Bankart repair alone versus Bankart and remplissage. An identical rate of recurrence was found in both groups, and one third of patients experienced posterosuperior pain.[41] Rotator cuff tears are rare in young individuals but common in older patients. Approximately 30% of patients older than 40 years have a cuff tear, as do about 80% of patients older than 60 years. Greater tuberosity fractures also occur with dislocations in older patients, and these have been associated with a lower incidence of recurrent dislocations. Older patients are less likely to have a Bankart lesion and more likely to have a cuff tear, a greater tuberosity fracture, or an avulsion of the capsule and subscapularis from the lesser tuberosity. Younger patients more commonly have labral tears. Coracoid fractures may also occur as a result of an anterior dislocation or a difficult reduction attempt. Vascular injuries are rare, but they may occur with anterior or inferior dislocations, especially in older patients with atherosclerosis of the axillary artery. The humeral head displaces the artery anteriorly over the head, and the pectoralis muscle acts as a fulcrum against the artery, leading to rupture. Arteriography should be performed if a vascular injury is possible. Because of the proximity of the 2 structures, arteriography should be strongly considered any time a brachial plexus injury is observed. Most commonly, patients present with delayed vascular compromise secondary to an intimal injury and resultant occlusion. Acute obstruction or rupture occurs in about 3.3% of cases of luxatio erecta.[11] Pseudoaneurysm may also occur, especially after recurrent dislocations. Subclavian vein thrombosis may result from a venous injury and present with unilateral swelling and pain. Neurologic injuries are more common than vascular injuries, particularly axillary neurapraxias, which are found in about 8-10% of patients with anterior dislocations. Patients have weakness in abduction and external rotation, as well as numbness over the lateral aspect of the upper arm. Among possible neurologic complications, these lesions have the poorest prognosis. Radial nerve injuries must also be considered in cases of axillary nerve damage because both arise from the posterior cord. These injuries may result in weak thumb, wrist, and elbow extension, as well as numbness on the dorsal aspect of the hand. Long thoracic nerve palsies may also result from traction on the nerve, leading to scapular winging due to paralysis of the serratus anterior. Suprascapular nerve palsies cause weakness in abduction and external rotation. Dorsal scapular nerve injuries cause weakness in abduction. Musculocutaneous injuries lead to weak elbow flexion and supination, as well as lateral forearm numbness. Arthroscopic findings after shoulder dislocation include the following: Bankart lesions in 80-89% of patients Anterior capsular insufficiency in 74% of patients Hill-Sachs lesions in 67% of patients Inferior glenoid labral tears in 51% of patients Glenohumeral ligament insufficiency in 50% of patients Partial or complete rotator cuff tears in 13% of patients Dysplastic glenoid in 13% of patients Biceps tendon lesions in 12% of patients Brachial plexus injuries in 11% of patients Posterior glenoid labral tear in 11% of patients Axillary nerve injuries in 8-10% of patients SLAP lesions in 8% of patients Partial subscapularis tear in 8% of patients Loose bodies in 5% of patients

What is the pathology and MOI of a Anterior dislocation?

Shoulder dislocation is a very common traumatic injury across a wide range of sports. In most cases, the head of the humerus is forced forward when the arm is external rotation and abducted. This causes an anterior dislocation, which makes up 95% of all shoulder dislocations. Remember the term TUBS: Traumatic, unidirectional, bankart, surgery.

What is the Etiology of MDI?

Shoulder instability has been classified by Hawkins and colleagues,[4] as well as others, in various ways, including direction (eg, anterior, posterior), degree (dislocation vs subluxation), etiology (eg, traumatic, atraumatic, overuse), and chronology (eg, acute, recurrent, fixed). For this article, 2 major categories are especially useful[5, 6, 4] : The mnemonic device TUBS: T raumatic etiology U nidirectional instability B ankart lesion S urgical repair When considering the diagnosis of MDI, it is helpful to remember the mnemonic device AMBRII: A traumatic etiology M ultidirectional instability B ilateral involvement R ehabilitative initial management Rotator I nterval tightening with I nferior capsular shift repairs

What are the mechanics of the Fowlers signs?

Silliman and Hawkins16 described the diminution of ap- prehension with the posteriorly directed force in Jobe's relocation test as "Fowler's sign." They further described the release test: "If the arm is suddenly released when stressed in external rotation abduction the patient has a dramatic increase in pain." They also described the aug- mentation test: "Similarly, one can augment the pain with external rotation and abduction by pulling forward on the back of the arm."

What is the future of medial humeral fracture?

Some authors advocate routine ulnar nerve transposition, while others believe this to be unnecessary unless the ulnar nerve has been injured. Controversy exists regarding how to manage a fracture that has remained untreated for several weeks or longer. Formation of callus and fibrous tissue may obliterate the fracture site and cause a malunion that makes accurate dissection and reduction less accurate. Misdiagnosis or inadequate early treatment increases the risk of complications such as loss of movement and angulation. Some suggest conservative treatment for fractures older than 4 weeks, while others have demonstrated some restored function in treating these fractures at the time of delayed diagnosis, although results are imperfect. Supracondyle wedge osteotomy is advocated to restore anatomic angulation and motion loss from previous injury. Controversy exists regarding whether this should be performed during growth or after the physis has closed. The major controversy involving medial epicondyle fractures involves the management of displaced medial epicondyle fractures. Good results have been reported with both operative and nonoperative treatment of the displaced medial epicondyle fracture. The medial epicondyle is the origin of the medial collateral ligamentous complex (see image below). Some advocate operative treatment of high-demand athletes, since even minor amounts of valgus instability can result in significant disability. Others recommend nonsurgical management because several long-term studies do not appear to substantiate significant valgus instability, even in individuals who went on to have radiographic nonunion of the epicondyle.

On a Monday in clinic you see a new patient who complains of sharp pain in left shoulder. Upon further examination, the patient cannot recall any recent specific injury to the shoulder but that he did have a hockey game Sunday morning in which he remembers getting checked into the boards. He remembers that he could not continue to play due to left flank pain. Identify the most likely visceral structure supporting your differential diagnosis below

Spleen Correct: Spleenic pain leads to Kehr's Sign-Left shoulder pain, LUQ tenderness to palpation, abdominal guarding & rebound tenderness, & systemic symptoms. For a good review on abdominal injuries, and referred pain, review Ryan (1999), Abdominal Injuries in Sport.

What is Sprengel deformity?

Sprengel deformity is a complex anomaly that is associated with malposition and dysplasia of the scapula.[5, 6, 7, 8, 9] This condition also involves regional muscle hypoplasia or atrophy, which causes disfigurement and limitation of shoulder movement.

What is the epidemiology of Sprengel deformity?

Sprengel deformity is the most common congenital malformation of the shoulder girdle.[10] The male-to-female ratio is 3:1.

What is the medial therapy for lateral humeral condyle fracture?

Stage I fractures with less than 2 mm of displacement may be treated with immobilization. Maintain cast immobilization for 3-4 weeks at 90° of flexion and forearm supination. Close follow-up is necessary because of the high incidence of late displacement and subsequent malunion. Obtain follow-up radiographs with the arm out of plaster for better fracture evaluation and assessment of possible displacement. If any question remains regarding joint stability or the possibility of delayed displacement, perform closed pinning. If fracture identification is delayed by 6 weeks or longer, continue closed treatment regardless of displacement. A high incidence of avascular necrosis occurs with delayed open reduction and fixation.

What the best x-ray view for hill sac lesion?

Stryker notch view

What are the signs and symptoms of bicep rupture?

Sudden sharp pain felt in the upper arm... tenderness to palpation at the shoulder joint... audible snap/// soft mass my be felt in the upper arm..

What is the MOI of a superior shoulder dislocation

Superior dislocations are extremely rare and result from an extreme force in a cephalic direction to the adducted arm. Acromioclavicular injuries and fractures of the acromion, clavicle, and tuberosities may occur with superior dislocations

What is the epidemiology of SLAP lesion?

Superior labral (labrum) lesions are unusual. The reported prevalence in patients undergoing shoulder arthroscopy has ranged from 3.9-6%. In the largest series reported, SLAP (superior labrum, anterior and posterior) lesions represented 6% of 2375 patients who underwent shoulder arthroscopy. The average age was noted to be 38 years, and 91% of the patients were male. The reported incidence of types I-IV is variable, with type II lesions being the most common. Type I lesions have been reported to account for 9.5-21% of all cases; type II, 41-55%; type III, 6-33%; and type IV, 3-15%.

What are the indications for shoulder dislocation for surgery?

Surgery may be indicated if patients are unable or unwilling to change their occupation or avoid participating in high-risk sports and if they have recurrent dislocations or subluxations

What are the contraindications with shoulder dislocation surgery?

Surgery should be avoided in patients with voluntary shoulder dislocations associated with psychiatric illnesses because the instability is likely to recur.

What are the contradictions for anterior instability surgery?

Surgical repair is relatively contraindicated in older patients with low physical demand, who have little chance of recurrence. Conservative therapy, including physical therapy to strengthen the rotator cuff, is indicated rather than exposing these patients to the risks of operation. Patients who have been asymptomatic in their activities of daily living also need not be exposed to the risks of surgery. These patients are best monitored for any recurrent dislocations. A certain contraindication exists in the case of multidirectional instability of the shoulder. In these atraumatic dislocations, patients are able to voluntarily dislocate and relocate their shoulder. Predisposing factors include psychiatric dislocations, laxity due to repetitive injury as occurs in competitive swimmers, and congenital collagen abnormalities such as Ehlers-Danlos syndrome and Marfan disease. The history and physical examination must be used to identify these patients. If these patients are treated as patients with unidirectional dislocations, operative therapy will fail. The amount of inferior capsular redundancy in multidirectional instability requires an operative procedure addressing the possibility of future inferior instability.

What are the indications of SLAP

Surgical treatment of superior labral (labrum) lesions is indicated in patients with persistent symptoms despite 3 months of nonoperative treatment. These patients should have persistent posterior shoulder pain, with or without mechanical symptoms, and findings on physical examination consistent with a SLAP lesion and ideally have supportive MRI findings. However, note that physical findings as well as MRI findings are often equivocal. If a high clinical suspicion exists, with the patient continuing to be symptomatic after an appropriate amount of conservative treatment, then strong consideration should be given to diagnostic arthroscopy.

What is the functional anatomy of UCL?

The UCL originates at the posterior distal aspect of the medial epicondyle and inserts into the base of the coronoid process. At 90 o of flexion, it provides 55% of the resistance to valgus stress at the elbow. In full extension, the UCL, bony architecture, and anterior capsule equally maintain valgus stability. The UCL is composed of 3 bands: anterior, posterior, and transverse. The anterior band, which arises from the anteroinferior surface of the medial epicondyle and inserts on the sublime tubercle of the ulna, provides the major contribution to valgus stability.

What is the sport biomechanics of the UCL?

The acceleration phase of the overhead throw causes the greatest amount of valgus stress to the elbow.[5] Extension occurs at a rate of up to 2500 o per second and continues to 20 o of flexion. During this phase, the forearm lags behind the upper arm and generates valgus stress while the elbow is primarily dependent on the anterior band of the UCL for stability. During the acceleration phase, valgus stress can exceed 60 Newton meters (Nm), which is significantly higher than the measured strength of the UCL in cadavers. The valgus force can, therefore, overcome the tensile strength of the UCL and cause either chronic microscopic tears or acute rupture.

What is the anatomy for Monteggia fracture?

The annular and radial collateral ligaments stabilize the radial head. These ligaments stretch or rupture during radial head dislocation.[11] The radial head articulates with the humeral capitellum and the radial notch of the proximal ulna. The radius and ulna are closely invested by the interosseous membrane, which accounts for the increased risk of displacement or injury to the radius when the ulna fractures. The distal ulna and radius also articulate at the distal radioulnar joint.[3] The ulna provides a stable platform for rotation of the radius and forearm. The ulna and interosseous membrane also may provide stable platforms for dislocation of the proximal radius, leading to the Monteggia fracture. The posterior interosseous nerve travels around the neck of the radius and dives under the supinator as it courses into the forearm. The median and ulnar nerves enter the antecubital fossa just distal to the elbow. The close proximity of these nerves may lead to injuries when a Monteggia fracture occurs. Neural injuries are generally traction injuries and result from stretching around the displaced bone or from energy dispersed during the initial injury.

What is the etiology of anterior instability?

The cause of anterior glenohumeral instability can be traumatic or atraumatic. Either mechanism leads to the loss of balance in the surrounding muscular and capsular structures. Traumatic injury to any one component of the shoulder soft tissue leads to instability. This traumatic insult most commonly comes in the form of an anterior shoulder dislocation. Atraumatic causes leading to multidirectional instability are not the focus of this article. These causes include repetitive atraumatic injury, Ehlers-Danlos syndrome, Marfan syndrome, congenital absence of the glenoid, deformities of the proximal humerus, and emotional and psychiatric instability

What are the intraoperative details for a coronoid fracture?

The coronoid can be approached posteromedially through a posterior midline incision after lifting the ulnar origin of the extensor carpi ulnaris (ECU) subperiosteally. In cases of a Monteggia fracture-dislocation, the coronoid may be approached through the interval between the ECU and the anconeus laterally and the flexor carpi ulnaris (FCU) medially. The radial head may be approached between the anconeus medially and the ECU laterally. This will help prevent formation of a synostosis between the radius and the ulna. After exposing the fracture site and cleaning the edges, the fragment is anatomically reduced and fixed by means of an interfragmentary screw (from posterior to anterior, or from anterior to posterior if the fragment is small or osteoporotic). The fracture may also be stabilized using heavy nonabsorbable sutures or suture anchors.[25, 26] The results from one study noted that suture lasso fixation of coronoid fractures for terrible triad injuries results in fewer complications and greater stability compared with screw or suture anchor fixation techniques. A higher rate of implant failure was noted with internal screw fixation, while the suture anchor technique resulted in a higher rate emedicine.medscape.com/article/1230817-‐‑treatment#showall 1/5 9/3/12 Coronoid Fracture Treatment & Management of malunion and nonunion.[27] In patients with highly comminuted coronoid fractures, reconstruction using a piece of the radial head (Esser technique) or a piece of the olecranon (Moritomo technique) has been described.

What is the anatomy of radial head fracture?

The elbow consists of the distal humerus, and the proaximal ulna and radius. The articulations of these bones make up various constrained joints within the elbow- Radiocapitellar Joint Articulation of the lateral condyle of the distal humerus and the radial head Allows elbow flexion, extension and forearm rotation making it a less constrained joint Constrained by the radialcollateral, lateral ulnarhumeral, and accessory collateral ligaments Ulnotrochlear Joint (the elbow) Articulation of the medial condyle of the humerus and the olecranon process of the ulna Allows only elbow flexion and extension making it the most constrained of the articulations Constrained by the Medial Collateral Ligament - including anterior, posterior, and transverse bundles Radioulnar Joint Articulation of the radial head and the proximal ulna Allows forearm pronation making it the least constrained of the articulations Constained by the annular ligament which wraps around the radial head Nerve Ulnar nerve passes just behind the medial epicondyle Posterior interosseus nerve wraps around the radial neck Median nerve is anterior along with the Brachial artery

What is the clinical presentation for radial head fracture?

The expected presentation of a radial head fracture would include:[1] Difficulty with pronation and supination Swelling throughout the elbow Difficulty and Pain with attempts to both flex and extend the elbow Pain along the lateral aspect of the elbow Typically the patient will carry their arm in a protective posture of approximately 80 degrees of flexion, and be resistant to all attempts to move the arm. The shoulder and the wrist and hand should be assessed to ensure these structures were not also injured in the fall.

What is the management of ACJ sprain?

The frist treatment that is best used is ice application within 24 hours to 48 hours and anti-inflammatories. Also use a splint that is tied to the shoulder. Large dislocations when the clavicle has shifted, surgery is neccessary. In a type 3 lesion and severe dislocation, there is sometimes a surgical procedure needed. Conservatice intervention largely revolves around pain control. Bracing and or taping can assist with return to play. Often pain from an AC injury last an long time.

what are the 4 ways the labrum adds stability to the shoulder?

The glenoid labrum enhances shoulder stability in 4 distinct ways: (1) it produces a ''chock-block'' effect between the glenoid and the humeral head that serves to limit humeral head translation10,38,57,60; (2) it increases the ''concavity-compression'' effect be- tween the humeral head and the glenoid10,31,38,57,60; (3) it contributes to the stabilizing effect of the long head of the biceps anchor46,57,60; and (4) it increases the overall depth of the glenoid fossa.

What is the medial therapy for olceranon fracture?

The goals of olecranon fracture treatment must be individualized to the needs of the patient. In young active individuals, restoration of the articular surface, preservation of motor power, restoration of stability, and prevention of joint stiffness are important. In older patients, minimization of morbidity is the most important goal. An understanding of the extent of associated injuries is critical prior to initiating treatment. Additional fractures or disruptions of collateral ligaments render the elbow unstable.

What is the background information for Olecranon fracture?

The human's unique prehensile skill largely depends on the integrity of the bones, ligaments, and muscles around the elbow joint. The elbow not only bends the arm but also permits pronation and supination of the hand. Fractures of the olecranon are common and are usually detected easily.[1, 2, 3] Images below show repaired olecranon fractures. Buijze et al compared the stiffness and strength of locking compression plate fixation to one-third tubular plate fixation in a cadaveric comminuted olecranon fracture model with a standardized osteotomy. Five matched pairs of cadaveric elbows were randomly assigned for fixation by either a contoured locking compression plate combined with an intramedullary screw and unicortical locking screws or a one-third tubular plate combined with bicortical screws. Construct stiffness was measured by subjecting the specimens to cyclic loading while measuring gapping at the osteotomy site, and construct strength was measured by subjecting specimens to ramp load until failure. The authors found no significant difference in fixation stiffness and strength between the 2 fixation methods, and all failures consisted of failure of the bone, not of hardware.[4] In a study by Buijze and Kloen, the authors noted that in patients managed with plate fixation for olecranon fractures, placement of an axial intramedullary screw may obstruct the placement of bicortical screws in the ulnar shaft. As a solution, they assessed the effectiveness of unicortical screws with a contoured locking compression plate. In the study, 19 patients with an acute comminuted olecranon fracture were managed with a contoured locking compression plate and intramedullary screw fixation, 16 of whom were available for follow-up at a minimum of 12 months after fixation. All 19 fractures healed, and the mean time to fracture union was 4 months. The mean Disabilities of the Arm, Shoulder and Hand score was 13. According to the Mayo Elbow Performance Index and the Broberg and Morrey grading system, 15 of the 16 patients followed had a good or excellent outcome. In 9 patients, hardware removal was necessary;; after removal, the mean elbow extension deficit improved from 34o to10o,andthemeanflexion improved from 118o to 138o.[5] According to Iannuzzi and Dahners, in comminuted fractures of the olecranon (Mayo type IIB), it may be difficult or even impossible to preserve the olecranon's normal articulation with the trochlea of the humerus. The authors therefore describe a modified technique for reconstructing these fractures when it is not possible to achieve a stable anatomic reduction and fixation;; in this technique, the comminuted fragments are excised and the proximal olecranon fragment is advanced past the resulting defect and fixed to the distal ulna. The authors present 2 cases with clinical follow-up and note that satisfactory preservation of range of motion and elbow stability were achieved in each case.[6]

What is the MOI of a clavicle fracture.

The most common location for a clavicle fracture occurs in the shaft, which is the middle third of the clavicle................. A clavicular fracture usually results from impact of falling of the tip of the shoulder or a direct blow to the shoulder. Less commonly, an indirect stress, such as falling on an outstretched hand or on the tip of the elbow.

What is the most common surgical procedure for cubital tunnel syndrome?

The most common surgical procedures include simple decompression, medial epicondylectomy and anterior nerve transposition (subcutaneous and submuscular).

What is the pathophysiology of ulnar neuropathy?

The nerve, axon, and myelin can be affected. Within the axon, fascicles to individual muscles may be involved selectively. Axonal involvement leads to motor unit loss and amplitude/area reduction. Conduction block implies impaired transmission through a segment of nerve. In the absence of changes indicating axonal damage, conduction block implies myelin damage to the involved segment. Significant slowing of conduction and/or significant spreading out of the temporal profile of the recorded response (ie, abnormal temporal dispersion) with preserved axonal integrity suggests demyelination.

What is the management of an anterior dislocation?

The patient needs to seek medical attention as soon as possibl. The shoulder needs to be reduced by a trained medical profressional. It may also be recommended to use a sling.

What is the epidemiology of MDI?

The prevalence of MDI (atraumatic shoulder instability) in the general population is not known. Traumatic shoulder instability is a much more common surgical indication

What band of the UCL prevents valgus force on the elbow?

The primary soft tissue restraint to valgus force at the elbow is the anterior band of the ulnar collateral ligament

What is the prognosis for coronoid fracture?

The prognosis for a complex fracture dislocation of the elbow is definitely poorer than that for a simple elbow dislocation, which has been shown to have good long-term results.[29, 30, 31, 32, 33] Prognostic factors include the following: Size of the fragment: type 3 fractures have the worst prognosis, with only 20% having good results in the series presented by Regan et al[8] The degree of damage to the articular cartilage at the time of injury The extent of soft-tissue injury[34] The stability obtained at the time of reduction The duration of immobilization[35]

What is the recovery phase of little league elbow syndrome?

The recovery phase begins once the patient's pain is resolved or is improved enough so that strengthening exercises can begin. Flexibility and strengthening programs are the main goals of therapy. Various modalities are used to prevent inflammation and speed the recovery from each session of therapy. Take care not to proceed though this phase too quickly, as the overuse syndrome can return. The athlete may begin with simple ball squeezing and newspaper crumbling with the affected hand. This can cause gentle strengthening of the forearm muscles. The athlete then progresses to gentle wrist flexion and extension exercises. Instruct patients to start out doing the wrist flexion and extension exercises with a can of soup, which is about 7.5 ounces. Once these exercises can be accomplished with a very light weight (ie, 2-4 lb), the patient may progress to elbow flexion and extension exercises, along with wrist pronation and supination activities. Recommend that patients also perform these exercises at home, possibly with a common household hammer. A hammer provides the athlete with a handle to grip, and the tool usually weighs between 18-26 ounces. Patients can also increase or decrease the resistance on pronation-supination activities by sliding their grip up or down the shaft of the handle. One study found that athletes who reside in warm-weather climates are more susceptible to throwing-related injuries than athletes who reside in cold-weather climates due in part to the time spent participating in throwing activities.[16]

What is the pathology of the sprengel Scapula?

The scapula is dysplastic and is located higher than normal in the neck or upper thoracic region. This bone is smaller than normal in the vertical plane and appears larger horizontally. The inferior angle is rotated medially, causing the glenoid to face inferiorly. An inverse correlation exists between the superior displacement and rotation of the scapula; with a higher scapula, the rotation is decreased. Convexity of the upper (supraspinous) portion of the scapula is increased and curvature of the clavicular shaft is decreased, forming a narrower scapuloclavicular space, which may contribute to brachial plexus compression postoperatively.

What is the shear angle for the moving valgus stress test?

The shear range refers to the range of motion that causes pain while the elbow is being extended with val- gus stress. The shear angle is the point that causes maxi- mum pain

What is the problem with proximal humeral fracture?

The shoulder links the upper extremity to the thorax. Optimal functioning of the upper extremity requires mobility and power that allow a range of performance, from powerful, explosive movements (eg, throwing a baseball 100 mph) to very accurate, fine movements (eg, performing microsurgery, playing the violin). Tasks of daily independence require the ability to position the hand throughout the range of an imaginary sphere. In addition to limiting function, disorders of the shoulder can cause pain, which, in turn, can affect the patient's work and sleep. Therefore, fractures of the proximal humerus can be devastating to quality of life. These fractures can also cost society a significant loss of productivity from otherwise viable members of the workforce.

What is the pathology and MOI for subacrominal bursitis?

The subacrominal bursa is a sac of fluid that separates the acromion from the rotator cuff. The bursa is underneath the coracoacromial ligament, acromion bone and the deltoid muscle. Subacromial bursitis is caused by upper extremity muscle weakness, overuse of the adjacent shoulder, calcium deposition, glenohumeral instability, degeneration of the ACJ or coracoid impingement.

What are the mechanics of the sulcus sign?

The sulcus sign has never been truly described; however, the earliest reference to this test (and the one used by most articles) is that by Neer and Foster.13 Even though the test is not described, there is a photograph of a patient in whom "inferior subluxation is produced by downward traction on the arm." If a depression is observed between the lateral edge of the acromion and the humeral head on gentle downward traction of the humerus, the sulcus sign can be said to be positive. A clearer description with quan- tification was provided by Silliman and Hawkins16 (to be described here later).

What is the background for the ulnar neuropathy?

The ulnar nerve is an extension of the medial cord of the brachial plexus. This is a mixed nerve that supplies innervation to muscles in the forearm and hand and provides sensation over the medial half of the fourth and the entire fifth digit of the hand, the ulnar part of the palm, and the ulnar portion of the posterior aspect of the hand (dorsal ulnar cutaneous distribution). The most common site of entrapment is at or near the elbow region, especially in either the region of the cubital tunnel[1] or the ulnar groove. The second most likely location of entrapment is at or near the wrist, especially in the area of the anatomic structure called Guyon's canal.[2] However, entrapment can occur in the forearm between these 2 regions, below the wrist within the hand, or above the elbow. As diagnostic and surgical methods have evolved over the past century, our ability to recognize and describe sites of entrapment has improved. However, the terminology has become confusing because not all clinicians use the terms in the same way. Let us first look at ulnar entrapments in the elbow region[3] , the most common location. The 2 most commonly used (and misused) terms for such entrapments are tardy ulnar palsy[4] and cubital tunnel syndrome[5] . In 1878, Panas first described what we now often call tardy ulnar palsy.[6] He presented 3 cases in which either prior trauma or osteoarthritis gradually caused damage to the ulnar nerve. The basic idea behind using the word tardy was that the problem appeared late after an injury or a long course of osteoarthritis (possibly together with an old injury) as opposed to a more immediate or early palsy in which the ulnar nerve showed dysfunction directly after trauma, such as what might occur in an injury that caused either total or partial transection. Subsequent to Panas' paper, other case reports appeared. John Murphy published the first case in American literature in 1914.[7] Walter Brickner reported a case in 1924.[8] The initial cases of tardy ulnar palsy were usually associated with trauma (eg, fractures in the region of the elbow), and the typical site of nerve entrapment was the ulnar groove, ie, the location between the medial epicondyle of the humerus and the olecranon.[9, 10] So in addition to a time-based definition (ie, tardy=appears some years after trauma), an anatomical aspect of the term came to pass (ie, tardy=usually in or very near the ulnar groove).[11] Later, physicians began to recognize ulnar entrapments in the humeroulnar arcade (HUA). This is the region of the aponeurosis of the 2 heads of the flexor carpi ulnaris (FCU) muscle. The aponeurosis is a fibrous or membranous sheet that connects muscles to bones or other structures that the muscles move. The aponeurosis can be thought of as a flattened tendon. The first description of an ulnar nerve entrapped in this region, together with its surgical decompression, was given by Buzzard and Sargent in 1922.[12, 13] The next published description was by Osborne in 1957. In 1958, Feindel and Stratford reported 3 more such cases and coined the term cubital tunnel syndrome to describe the effects of the ulnar nerve entrapment[14] at the HUA. Numerous other reports then followed. Our current state of knowledge is still incomplete, but now we can identify approximately 5 sites in the elbow region at which the ulnar nerve is most likely to be compressed. The word approximately is used deliberately, because some of the sites are so close together that certain authorities categorize them differently to get a different number. This article principally follows the classification of Posner[15] , with some comments about the classification of other authors. The sites, according to Posner, are as follows: Above the elbow in the region of the intermuscular septum Halikis et al[16] divides this into 2 regions—the arcade of Struthers[17, 18] and the medial intermuscular septum. Via the standard anatomic definition, the arcade of Struthers is a thin fibrous band that usually extends from the medial head of triceps to the medial intermuscular septum. It is often said to be about 6-10 cm proximal to the medial epicondyle. Considerable anatomic variation exists and, in fact, there is outright controversy about the arcade of Struthers.[19] One such controversy is trivial as no evidence exists that Dr. Struthers discovered this structure or even knew about it. His name was attached to it by Kane et al in their 1973 paper.[20] An autopsy study by Siqueira of 60 upper limbs found a structure reasonably approximating the definition given above in 8 limbs (13.5%).[19] Ulnar nerve entrapment occurred in none of them (but there was no reason to clinically expect that there might have been). Bartels et al could not find this structure in their dissections and they doubt that it exists.[21] Wehrli and Oberlin have described a different structure in the same region, the internal brachial ligament rather than the arcade of Struthers, that might be involved in ulnar entrapment in some cases.[22] Interestingly, Struthers did describe the existence of this structure, but not in relation to ulnar nerve entrapment. Wehrli and Oberlin advocate "cancelling the concept of the arcade of Struthers." In contrast, von Schroeder and Scheker find yet another structure, a fibrous tunnel in roughly the same region.[23] They say that the ulnar nerve goes through this tunnel and could be trapped therein and are in favor of naming their structure the arcade of Struthers. Settling this controversy is beyond the scope of this article. Suffice it to say that in rare cases, the ulnar nerve is compressed considerably above the ulnar groove and that surgeons may find it entrapped in a fibrous/ligamentous structure that may correspond to one of the terms mentioned above. Medial epicondylar region Ulnar compression[24] in this region is generally from a valgus deformity of the bone. If a patient is placed in standard anatomical position with palms rotated toward front, thumb away from midline, valgus deformity means the elbow would be deformed away from midline of the body. Epicondylar groove This is the same as the ulnar groove. It is a bit distal to the medial epicondyle (or at least to the beginning of it). Using slightly different terminology, Campbell lumps the medial epicondylar region and the epicondylar groove together as the area of the retrocondylar groove. Halikis et al consider the medial epicondylar region and the epicondylar groove to be the area of the medial epicondyle.[16] Both the medial epicondylar region and the epicondylar groove are generally considered to be the classical location (or locations) for the tardy ulnar palsy. In the author's personal experience, electromyographers and orthopedic surgeons more commonly refer to a tardy ulnar palsy at the retrocondylar groove, thus using the Campbell terminology. The region of the cubital tunnel The main source of compression is a thickening of the Osborne ligament. Campbell's classification is basically the same for this region, except he no longer uses the term cubital tunnel. He refers to this as the region of the HUA, apparently because he believes so many clinicians loosely use the term cubital tunnel to refer to a place anywhere in the elbow. Halikis et al divide this region into 2 parts—the cubital tunnel and the Osborne fascia.[16] This is a good example of the difficulty with the terminology. Different terms are used for locations that are virtually the same. For all practical purposes, certainly for anything one can distinguish on EMG, Osborne ligament=Osborne fascia=the HUA. The cubital tunnel is the space bounded by the following: The medial epicondyle (medial border) The olecranon (lateral border) The elbow capsule at the posterior aspect of ulnar collateral ligament (floor) The humeroulnar arcade (Osborne fascia or ligament) (roof) After the ulnar nerve passes distal to the elbow,[26, 27, 7] it makes several important divisions. The first branches to come off are those that go to the FCU. Further distally, the branches to the flexor digitorum profundus muscles of digits 4 and 5 arise. As the ulnar nerve courses down the forearm toward the wrist, the dorsal ulnar cutaneous nerve leaves the main branch. A little further down, the palmar cutaneous branch takes off. Thus, neither of these 2 branches goes through Guyon canal.[2] The remainder of the ulnar nerve enters Guyon canal at the proximal portion of the wrist. This is bounded proximally and distally by the pisiform bone and the hook of hamate bone. It is covered by the volar carpal ligament and the palmaris brevis muscle. Although the nerve could be injured or entrapped at any point along its course, the 4 most common locations in relation to Guyon canal are shown in the following image

What are the 3 stages of ulnar neuritis?

There are three stages of ulnar neuropathy [30]. The first stage includes an acute onset of radicular symptoms. The second stage is manifested by a recurrence of symptoms as the athlete attempts to return to competition. The third stage is associated with persistent motor weakness and sensory changes. Once the athlete presents in the third stage of injury, conservative management may not be effective.

What is the common direction for multidirectional instabilty

There is excessive mobility of the glenohumeral joint in all directions: anterior, posterior, and inferior. However, there may be a predominance of one direction, typically anteroinferior or posteroinferior.

What structures are at risk with SICK and GIRD?

This combination is particularly dangerous to the posterosuperior labrum, the undersurface of the posterior supraspinatus tendon, and the anterior infe- rior capsular structures. The reason for this is that both problems cause the thrower to abduct in extension (toward second base), rather than in the plane of the scapula, and hyperangulate in external rotation with a low arm body angle (below the horizontal) during the late cocking phase of throwing

What are the complication with clavicle fracture?

Thoracic outlet syndrome has been reported in the literature as a complication of nonunion, malunion, or excessive callus formation with clavicle fractures. The thoracic outlet is the space for passage of the subclavian vessels and brachial plexus. The borders of the thoracic outlet are the scalene muscles, the first rib, and the clavicle. When this space is narrowed for any reason, compression of the structures may occur and produce symptoms of thoracic outlet syndrome

What are the indications for clavicle fracture?

Traditionally, clavicle fractures have been treated nonoperatively, and the consensus was that they all heal. While it is true that, if all clavicle fractures are considered together, the vast majority will heal with nonoperative management, including a figure-of-eight brace or a simple shoulder sling, studies have found that in cases of specific fracture patterns and locations, not all clavicle fractures behave the same way. Medial third fractures Current management of medial clavicle fractures has remained nonoperative, and results have remained consistently good. Significant displacement is rare because of the extensive ligamentous attachments. However, if significant displacement occurs with this fracture, further imaging studies are warranted. A CT scan should help define the nature of the fracture displacement and the status of the nearby neurovascular structures. Middle third fractures The focus of treatment of middle third fractures remains nonoperative. Nonoperative treatment can be divided into 2 categories: simple support of the extremity, as in a sling or a sling and swath, and reduction and immobilization, typically with a figure-of-eight brace. These treatment options are applicable for almost all middle third clavicle fractures, with the exception of those that are severely displaced or shortened. The image below illustrates the displacing forces. No proven benefit of any specific technique of immobilization exists, so the choice of immobilization should depend on the comfort and functional demands of the patient. Immobilization may be discontinued when pain and palpable motion are no longer present at the fracture site. Stiffness is usually not a problem after nonoperative treatment of clavicle fractures. If the patient does require some rehabilitation, it should include forward elevation and external rotation. Laborers may return to light lifting after 6 weeks and full duty at 12 weeks. Athletes may return to contact sports after 3 months. Grassi et al examined 40 patients who were treated nonoperatively with a figure-of-eight brace and 40 patients treated with open reduction and intramedullary fixation with a 2.5-mm threaded pin for uncomplicated midclavicular fractures.[15] Patients who were treated nonoperatively had fewer complications and faster return to normal daily activities, heavy lifting, and sports. Overall, patients in both groups were satisfied with their results; however, 35% of the group who underwent intramedullary fixation had some adverse events during their recovery. Most of these problems were minor. However, 3 patients experienced refracture after removal of the intramedullary pin. These patients were then subsequently treated with a figure-of-eight brace, and union then occurred. Given the excellent results obtained with nonoperative treatment of uncomplicated midclavicular fractures, nonoperative treatment in a figure-of-eight brace or regular support sling is recommended. Operative treatment is best suited for more complicated fractures of the middle third of the clavicle. Evidence is mounting in support of operative treatment for displaced midshaft clavicle fractures. The Canadian Orthopaedic Trauma Society performed a multicenter prospective randomized trial comparing the outcome of nonoperative treatment versus plate fixation for displaced fractures of the midshaft.[5] Mean time to radiographic union was significantly shorter in the operative group (16.4 weeks versus 28.4 weeks). Additionally, functional outcomes were improved at all time points measured in the operative group. This study provided level I evidence in support of plate fixation for completely displaced midshaft clavicle fractures in the active adult population. Hill et al examined a subset of clavicle fractures in which initial shortening of the fracture was greater than 2 cm. They found a high rate (15%) of nonunion in this population.[16] Also, final shortening of more than 2 cm was associated with unsatisfactory results. Open reduction and internal fixation of these injuries is recommended for patients with displaced middle third clavicle fractures with greater than 2 cm of shortening. Wick et al reviewed 39 nonunions of midclavicular fractures treated nonoperatively and found a correlation between initial fracture shortening of greater than 2 cm and nonunion.[17] These patients subsequently underwent open reduction and internal fixation with subsequent union of the fracture. The major patient complaint for all of these nonunions was pain, and all patients had complete or near complete resolution of their symptoms. Wick, however, still recommended a trial of conservative treatment prior to open reduction and internal fixation ofthesefractures. Distal third fractures Much controversy exists in the literature regarding the appropriate management of fractures of the distal third of the clavicle. Incidence of nonunion of displaced distal third fractures is high, and current recommendations are to fix these injuries surgically. Neer found that although distal third clavicle fractures are rare, they account for approximately half of all clavicular nonunions.[18] Many different procedures have been described to fix these fractures, and intramedullary fixation is gaining popularity. However, a problem exists with migration of intramedullary wires. Many articles have been published focusing on the treatment of distal third clavicle fractures. As mentioned previously, these injuries account for about 12-15% of all clavicle fractures. Fractures of the distal clavicle are further divided into types I-III. In type I injuries, the coracoclavicular ligaments are intact and the fracture is usually minimally displaced or nondisplaced. The first image below illustrates displacing forces; the second image illustrates a type I fracture. Type II fractures are at the level of the coracoclavicular ligaments and are further subdivided into IIA and IIB fractures. In IIA fractures both the conoid and trapezoid ligaments remain intact and the fracture is medial to the ligaments. Type IIB fractures involve a disruption of the conoid ligament, with the trapezoid ligament remaining intact and attached to the distal fracture fragment. Included in the IIB fracture is the more rare variant in which both the conoid and trapezoid are ruptured. Type IIB injuries tend to have significant displacement of the fracture fragments because of the loss of the downward restraint on the medial fragment from the coracoclavicular ligaments. Type II fractures are depicted in the images below. Type III injuries are distal to the coracoclavicular ligaments and involve the acromioclavicular joint. Type III injuries are usually minimally displaced or nondisplaced and are treated nonoperatively. Chen et al reported 10 of 11 patients to have good-to-excellent results with their technique of repair.[19] The conoid ligament is reconstructed with Mersilene tape, and the torn ligament is primarily repaired as well. The fracture is fixed with a No. 7 or smaller steel wire. The wire fixation and the Mersilene tape provide stability for the fracture, allowing the repaired coracoclavicular ligament to heal. All fractures in Chen's study united within 6 months, and 10 of 11 fractures maintained the coracoclavicular reduction. Nine of 11 patients had full relief if pain and restoration of full range of motion, and 10 of 11 patients were satisfied with the surgery and stated they would undergo the procedure again for treatment of this fracture. Kao et al reported on 7 patients with displaced type IIA fractures and 3 patients with IIB fractures who all underwent open reduction and internal fixation with Kirschner wires (K-wires) and a tension band.[20] Also included were 2 patients with comminuted distal clavicle fractures. Kao et al's technique spared the soft tissue around the fracture site, including the AC joint, with dissection limited only to the fracture site. Eleven of 12 fractures formed bony unions with this technique, and these patients experienced pain-free range of motion. Another surgical option for lateral clavicle fractures involves using a Dacron arterial graft as a sling around the medial fracture fragment and the coracoid. This acts to stabilize the medial fragment in a reduced position in the superior/inferior plane. This procedure was performed on 11 acute distal clavicle fractures, and all of these patients' fractures united with full range of motion. Four other patients were included in this study who were previously diagnosed as having established nonunions. These patients underwent fixation of the nonunion with a lag screw, iliac crest bone grafting, and stabilization with a Dacron sling. These patients all subsequently developed bony union of the fracture site with full range of motion. Of note, the Dacron sling did cause some slight erosion of the clavicle that was in contact with the sling; however, this did not progress and did not cause any problems for the patients. The sling is also thought to allow for the return of function of the coracoclavicular ligaments. Once the coracoclavicular ligaments reconstitute, the Dacron sling becomes redundant. The use of Wolter clavicular plates for unstable, comminuted distal clavicle fractures was reported to result in good bony union and range of motion in all 16 patients in a series by Mizue et al.[21] This procedure, however, requires a second operation for removal of the plate and is only recommended for injuries that are severely comminuted and unstable.

What are two special test to examine stability of the elbow?

Two special tests used to examine stability of the elbow joint complex are the manual valgus stress test (assessing medial stability) and the valgus extension overload test (assessing for presence of posteromedial olecranon osteophyte).

What are the contraindications for a coronoid fracture?

Type 1 and type 2 fractures usually do not require operative stabilization. Highly comminuted type 3 fractures pose a significant problem during open reduction and internal fixation (ORIF) and may be better treated with a hinged external fixator.

What is the 3 classification of a bankart lesion?

Type I - Pure capsular lesion Type II - Partial labral detachment Type III - Complete detachment of the inferior glenohumeral-labral complex

What are the causes of ulnar neuropathy?

Ulnar nerve at or near the elbow Compression during general anesthesia Blunt trauma Deformities (eg, rheumatoid arthritis) Metabolic derangements (eg, diabetes) Transient occlusion of brachial artery during surgery[38] Subdermal contraceptive implant[39] Venipuncture[40] Hemophilia[41] leading to hematomas Malnutrition leading to muscle atrophy and loss of fatty protection across the elbow and other joints Cigarette smoking[42] Ulnar neuropathy at or distal to the wrist (ie, at Guyon's canal) Ganglionic cysts Tumors Blunt injuries with or without fracture Aberrant artery Idiopathic

What is the artery of the labrum?

Vascular supply to the labrum arises mostly from its peripheral attachment to the capsule and is from a combination of the suprascapular circumflex scapular branch of the subscapular and the posterior circum- flex humeral arteries.

What is the pathology and MOI of Volkmann's contracture?

Volkmann's contracture is a shortening of the muscles of the forearm as a result of a trauma such as: fractures, crush injuries, burns and arterial injuries. It is a rare disease, counting for 0.5% of injuries. The intracompartmental pressure occurs when there is a bulging caused by a trauma. This results in a lessening amount of space for the muscles, nerves, and blood vessels lying within the fascia. Possible causes of this include animal bites, fractures within the forearm, bleeding disorders, burns, excessive exercise and injections of medication at the forearm.

What is the surgery for clavicle fracture.

When a midshaft clavicle fracture requires surgical fixation, there are 2 methods of fixation that are commonly performed. Both methods involve open reduction of the fracture, followed by either insertion of an intramedullary device or fixation with a plate and screws.[6, 24] Intramedullary fixation requires a small incision over the fracture site, carried down sharply to the clavicle, without stripping the periosteum. A Steinman pin is the placed in a retrograde fashion past the fracture site. It is recommended that the Steinman pin be threaded in the proximal fragment to prevent migration. If a smooth pin is used, bend the distal tip to prevent migration after crossing the fracture site. Cancellous bone grafting is indicated in cases of comminution and/or bone loss. Surgical fixation with a plate and screws is another option for midshaft clavicle fractures. An incision is made in line with the clavicle and carried down to periosteum sharply, with caution to leave thick skin flaps for closure. The periosteum is then stripped to expose and reduce the fracture. Plate and screw fixation is then performed using any of a wide variety of plates. Recommendations vary from semitubular plates, dynamic compression plates, low contact dynamic compression plate, or double plating. However, fixation of these fractures with semitubular and reconstruction plates is not as strong biomechanically as fixation with dynamic compression plating or the newer locking-plate technology. Obtaining purchase in 6 cortices on either side of the fracture is recommended. Lag screw fixation is also appropriate when the fracture pattern allows. Again, cancellous bone grafting is suggested in fractures with comminution and/or bone loss. Mehmet et al conducted an evaluation of the biomechanical properties and the stability of a locking clavicle plate (LCP), a dynamic compression plate (DCP), and an external fixator (Ex-fix) in an unstable displaced clavicle fracture model under torsional and 3-point bending loading. For both torsion and bending, an overall significant difference was found among the 3 constructs in terms of failure loads; a significant difference was also noted between the LCP and the other 2 models in terms of initial stiffness. The LCP is significantly more stable than the DCP and Ex-fix under torsional and bending cyclic loading in a displaced fracture clavicle model.[25] When using plate and screw fixation to treat clavicle fractures, the surgeon must remember that the hardware will likely be prominent. Proper closure of these incisions is imperative to decrease the risk of painful, prominent hardware. Many techniques of surgical fixation of distal clavicle fractures have been described in the literature. In general, surgical fixation is recommended for type II distal clavicle fractures. Treatment of these fractures requires direct visualization and reduction of the fracture fragments through a vertical incision. After the fracture is visualized and reduced, the coracoclavicular interval is stabilized. Stable fracture fixation can be achieved in many ways, including combinations of a coracoclavicular screw, Dacron or Mersilene tape, tension banding, Kirschner wire (K-wire), and clavicular plates. Regardless of the exact technique used, the general principles of fracture reduction and fixation and stabilization of the coracoclavicular interval apply.

What is the differential diagnosis of medial epicondylitis?

When diagnosing a medial epicondylitis, the therapist always has to consider other pathologies such as[1][6]: - a compression neuropathy of the ulnar and the median nerve - ulnar collateral ligament instability - ulnar neuritis - Obsessive compulsive disorder (OCD) - Medial collateral ligament sprain - Adhesive_Capsulitis - Cervical_Radiculopathy - Osteoarthritis - Flexor-pronator strain The use of Polk's_Test may help the clinician to diagnostically differentiate between lateral en medial epicondylitis. Other various orthopedic testing procedures including Cozen's_Test, Golfer's_Elbow_Test, Mill's_Test and Kaplan's test are used for the clinical differentiation between lateral en medial epicondylitis (level of quality D) [4].

What angle will a patient with elbow swelling hold there arm?

When there is an elbow effusion, the athlete will often hold the elbow flexed 70° to 80°, the position that corresponds with the greatest capsular volume

What is the postoperative management for medial humeral fracture?

With all degrees of injury, immobilization must continue until solid union is demonstrated. Radiographs must be repeated until the union is ensured. This may be as early as 3 weeks for nondisplaced fractures and is usually approximately 6 weeks for displaced fractures but may be several months. This immobilization must be balanced with the need for physical therapy to prevent loss of range of motion. For fractures treated with ORIF, the arm should be put in a cast in 90° of flexion for 3 weeks and then placed in a posterior mold for 3 weeks with supervised active flexion and extension out of the mold. Internal fixation allows for this early physical therapy without compromising the reduction. Active range of motion with physical therapist supervision is critical to prevent excess loss of flexion and extension. Passive range of motion should be avoided because it can result in damage to contracted soft tissues and has been associated with myositis ossificans.

A 16 y/o high school football player is referred to your clinic with dx of shoulder pain. He reports that he was trying to make a tackle with his arm outstretched and "when he grabbed the person it felt like his arm got pulled out of socket." He thinks he heard a pop, but says it felt like his shoulder went back into place right away. He reported weakness and tingling in his arm, with most being focused on the upper part of the shoulder. Over the last few days he has had difficulty lifting his arm over his head and lifting objects. During the physical examination the physical therapist finds decrease sensation over the deltoid and also weakness with abduction, flexion and lateral rotation. His deep tendon reflexes are 2+ bilaterally. He reports no sensation changes in his hands or below his elbow. Which nerve is likely involved?

a. Axillary Correct: The axillary nerve innervates the deltoid, teres minor, and long head of the triceps. Its sensory distribution covers the shoulder joint, often described as the "regimental badge." The axillary nerve is the most commonly injured nerve after a shoulder dislocation.

A 14 year old elite gymnast complains of constant anterior shin pain of insidious onset which increases with activity and decreases with rest. Symptoms have been present for 3 weeks after joining her school track team in addition to gymnastics. Her past medical history is significant for a stress fracture of her 5th metatarsal. Which of the following are predisposing factors for female athlete triad in this athlete?

a. Her past history of a stress fracture Incorrect: Age is not a predisposing factor for female athlete triad

A 22 year old college senior is wrestling in the final round of a tournament. During the match, his arm is taken into 90 degrees abduction and 90 degrees ER, and then is taken back into pure extension. He immediately dislocated, but is able to be reduced on the sideline by the attending orthpaedic surgeon. Which of the following factors is the most important regarding his risk for recurrent instability?

a. His degree of athletic participation Correct: If this individual will continue to wrestle, versus stop all overhead recreational pursuits, it will greatly affect his risk for re-dislocation

An 8 year old male presents to physical therapy with his legal guardian (grandmother). He is complaining of unilateral hip pain on the right for the past 3 days. He is of middle-eastern descent, ectomorphic, and has no history of trauma. He has a slight limp. Upon physical examination, you note the following: ROM (in degrees) L: hip flexion 125, hip extension 18, IR 43, ER 38, abduction 40, adduction 18 R: hip flexion 121, hip extension 15, IR 35, ER 32, abduction 38, adduction 18 MMT L: hip flexion 5/5, hip extension, 4+/5, hip abduction 4+/5 R: hip flexion 4/5 with pain, hip extension 4/5, hip abduction 4-/5 Special Testing Positive hip scour on R Positive Thomas test bilaterally Positive Ober's test bilaterally Positive resisted SLR on R Positive femoral log roll on R What is this individuals most likely diagnosis, and prognosis?

a. Legg-Calve Perthes Disease; this needs requires referral to MD as soon as possible. Given the age of the patient, his outcome is unfavorable. Incorrect: While referral to an MD ASAP is warranted with SCFE, this is not SCFE. SCFE typically presents in children 10-17, with a loss of hip abduction and flexion. See the SCFE quick review for more about the pathology.

A 65 year old male presents to you 6 months s/p L THA, with a posterolateral approach. He reports that he is an avid tennis player, and that he would like to return to tennis. However, he isn't sure if this is advisable after surgery. Assuming an unremarkable recovery, what do you recommend?

a. Recommend he return to doubles tennis Incorrect: According to recommendations established by 1999 Hip Society Survey, singles tennis is not recommended after THA, regardless of the timeframe. See Healy et al., (2001) or Vogel et al. (2011).

Research does NOT support the use of _________ in the non-operative management of traumatic anterior shoulder dislocation.

a. Shoulder immobilization with a traditional sling for 1-2 weeks Correct: According to Hayes et al., (2002), immobilization in a traditional sling is not warranted after a primary dislocation. Like most joints of the body, we want to promote safe ROM (passive or active) after injury, not strict immobilization.

A 16 year old gymnast was doing her mat routine, when she slipped and fell, landing on on outstretched hand. At the time of injury, she was taped, and continued to tumble. Her initial x-rays were negative after the competition. She was diagnosed in the emergency room with a sprain. Now 4 weeks later, she complains of difficulty with upper extremity weight bearing position. An MRI was ordered. What do you suspect given the MRI below?

a. TFCC tear Incorrect: What you can see of the scaphoid, it appears intact and healthy. However, you would need more slides to assess its integrity.

What are the special test if UCL tear?

o MRI o Valgus stress test o Moving valgus stress test

What was the normal range for a positive shear angle?

with surgical assessment, the mean shear range was 120° to 70° from flexion to extension (range, 135°-15°), with a mean shear angle of 90° (range, 70°-110°)

what are the special tests for elbow dislocations?

x-ray

what is the post op rehabilitation consideration for medial collateral ligament reconstruction?

• Early rehabilitation emphasizes immediate, controlled motion and avoidance of excessive stretching. • A brace is worn post-operatively 5-6 weeks with a gradually increase in available motion. • The elbow should not extend beyond 20 degrees during the first two post-operative weeks

What are the post rehabilitation consideration for ulnar nerve transposition?

• Pain is often relieved post-operatively and other sensory symptoms show consistent improvement with time. • Weakness may persist and intrinsic muscular atrophy is the least likely to improve after ulnar nerve transposition. • The potential for full motor recovery after operation is greatly reduced in patients who experienced preoperative symptoms for greater than one year or who have intrinsic muscle atrophy before surgery.

What are the rehabilitation for the elbow dislocation?

• Prolonged rigid immobilization has been associated with the least satisfactory elbow motion outcomes and should be avoided. • Elbow flexion generally returns first and maximum motion is usually achieved by 12 weeks. • Elbow extension is slower to return and make take 5-6 months to return. • Heterotopic ossification is common after elbow dislocation (up to 75% of patients); however, it rarely limits motion. • Mechanical testing shows an average loss of 15% of elbow strength following elbow dislocation.

What is the capsuloligamentous restraint

"Matsen et al stress that the aforementioned components provide midrange stability. Midrange stability refers to stability in the mid range of motion ROM), where the ligaments and capsule provide little tension-dependent static stability. These factors act independently of the capsuloligamentous restraints. The capsule serves as a passive leash that can restrain glenohumeral motion within a given ROM. The insertion of the capsule upon the glenoid labrum glenohumeral ligaments are ideally positioned thickenings within the capsule that serve to check large forces encountered within the capsule during specific arm positions and activities. Numerous studies have elucidated the role of the capsuloligamentous complex in the static stabilization of the shoulder, and it has been shown that the inferior glenohumeral ligament is clearly the most crucial component.[7, 8, 9] The value of the dynamic supports of shoulder stability (ie, scapular stabilizers) cannot be overstated. The proper compressive function of the rotator cuff is essential for glenohumeral stability and remains the primary focus of rehabilitative management for this problem. Deficits of shoulder proprioceptive function have been reported in MDI.[10] "

What are the signs and symptoms of posterior dislocation?

1) Significant pain...2) inability to move the arm from its current position...3) possible numbness. 4) deformity

What are the 8 SLAP special test

1) Snyder's biceps tension test (resisted shoulder flexion with elbow extension and forearm supinated 2) Compression rotation test (rotating the humerus while compressed in attempt to trap torn labrum) 3) Anterior slide test (resisted shoulder extension while shoulder internally rotated and hand on hip, positive result is pain in front of shoulder or pop/click) 4) Crank test (joint load and rotation while shoulder in extreme elevation in scapular plane, positive test is reproduction of patient's symptoms or pain) 5) O'brien's active compression test (distinguishes b/t superior labral and acromioclavicular abnormalities, resisted elevation with pronation and supination; positive labral abnormality = pain with pronation and reduced/eliminated with supination) 6) Pain provocation test (externally rotated shoulder/elbow flexion in pronation then supination; positive labral abnormality= pain only with pronation) 7) Biceps load test (resisted elbow flexion while in shoulder external rotation and 90̊ elbow flexion; positive= no decrease in apprehension and increase in pain) 8) Biceps load II test (same as biceps load test with more shoulder elevation

What are the 4 laxity special Test

1) Sulcus sign (inferior subluxation produced by downward traction on the arm) 2) Anterior drawer test (assesses amount of anterior translation of humeral head on fixed scapula) 3) Posterior drawer test (assesses the amount of posterior displacement of the humeral head on the scapula) 4) Load and shift test (assesses amount of anterior and posterior glenohumeral translation of loaded humeral head). An alternative version of the load and shift test described by Faber et al. examines glenohumeral translation in various positions

What is the pathology and MOI of a ACJ sprain

1) Type 1: is a partial injury to the capsule. Paired with an incomplete tear of the acromioclavicular ligament and joint capsule... 2) Type 2: is a moderate injury. A partial tear of the ligament coracoclavicular with a rupture of the capsule and the acrominclavicular ligament. Often described as a subluxation. 3) Type 3 injury: a severe injury. Complete ruture of the acromioclavicular ligament but also the coracoclavicular ligament. This can lead to displacement of the clavicle and therefore very unstable. This is often described as a dislocation. Type 4: posterior dislocation.. Type 5: dislocation with serve upward displacement of the clavicle into or through the traqezius... Type 6: This is a rare injury. Dislocation of the clavicle inferiorly. locked under the coracoids process.

What are the 4 structures that increase the stability of the shoulder

1) produces a "chock block" reducing humeral head translation; 2) contributes to the concavity compression effect; 3) stabilize long head of the biceps anchor; 4) increase depth of the glenoid fossa.

What are the 8 common injuries of the elbow?

1) ulnar collateral ligament tears, 2) ulnar neuritis, 3) flexor-pronator strain, tear, or tendinitis, 4) medial epicondyle apophysitis or avulsion, 5) valgus extension overload syndrome with olecranon osteophytes, 6) olecranon stress fractures, 7) osteochondritis dissecans (OCD) of the capitellum, 8) loose bodies

What are the intraoperative medial humeral fracture?

A medial approach may be used. A longitudinal incision is made over the medial supracondyle ridge of the humerus and continued just distal to the medial condyle. Branches of the medial antebrachial cutaneous nerve should be identified and preserved. The ulnar nerve is identified and protected and may be transposed anteriorly. The fracture surfaces are identified and cleaned, and the joint space is cleaned and irrigated to remove loose particles. The condyle fragment is then reduced and secured at a minimum of 2 sites to prevent rotation (see image below). Kirschner wires (K-wires) or cancellous screws may be used. Plate-and-screw fixation is another option. The reduction should be confirmed radiographically. The wound is closed, and the arm is splinted in 90° of flexion with the forearm in the neutral position Medial epicondyle fracture A longitudinal incision is made just anterior to the medial epicondyle. The fragment is usually displaced distally and anteriorly. As with any fracture reduction, periosteum and bone fragments are cleared from the fracture site to allow anatomic reduction. The ulnar nerve must be identified and protected. Ulnar nerve transposition is usually unnecessary. With the elbow flexed and pronated, the fracture fragment is reduced and pinned with 1-2 Kirschner wires (K-wires). A lag screw is then placed to maintain and compress the fracture fragment. Elbow stability and range of motion are assessed. A posterior splint is then applied for at least 7-10 days until range of motion is initiated.[22, 23, 25, 29, 34] If the epicondyle is fragmented, excision of the fragment and fixation of the flexor-pronator origin and medial collateral ligament to bone with an alternative form of fixation (such as suture anchors) may be used. Excision of the fragment does not appear to yield results comparable to those of nonoperative treatment.

What is the prognosis for ulnar neuropathy?

A motor amplitude of 10% of normal or a greatly reduced recruitment of motor units indicates a low likelihood of significant or full recovery. Typically, nerves regenerate at a rate of 1 mm/d. In some cases, regeneration is accompanied by pain and paresthesias, which are thought to be secondary to random ectopic impulse generation of affected nerves. A diameter greater than 3.5 mm on the initial sonogram of the ulnar nerve at the elbow is associated with persistent symptoms or signs regardless of treatment (ie, conservative treatment or surgical treatment[78] ). The outcome does not correlate with clinical features at baseline or with the duration of symptoms prior to treatment. The presence of motor conduction velocity slowing or pure conduction block across the elbow signifies a favorable outcome; these are considered independent prognostic factors[79] . A favorable surgical outcome is more likely for sensory function than for motor function; however, overall, a favorable outcome occurs in 85-95% of cases. Unfavorable or poor surgical outcome is characterized by the following: Age older than 50 years Coexisting diabetes or other causes of peripheral polyneuropathy Atrophy and ongoing denervation of ulnar-derived muscles Absent ulnar sensory responses Postoperative position of the ulnar nerve in relation to the medial epicondyle

What is the diagnostic procedure?

As epicondylitis is essentially a musculotendinous condition, diagnosis is essentially clinical. Radiographs are typically negative unless the chronicity of the condition had allowed periostitis to develop on the affected epicondyle (level of quality D)[4]. The diagnosis of medial epicondylitis is based on local pain at the elbow, tenderness at the epicondyle on palpation and pain at the epicondyle on resisted isometric flexion and pronation of the wrist (level of quality A1)[7] (level of quality A2)[3]. In particular the Golfer's_Elbow_Test, an orthopedic test, is described as being helpful to diagnose medial epicondylitis (level of quality D)[4].

What is the non-operative for olceranon fracture?

As stated in Contraindications, nonoperative treatment is often desirable in patients with significant associated medical conditions. Contused soft tissue healing is of paramount importance. Nonoperative treatment of even significantly displaced olecranon fractures in patients with severe medical illness, steroid use, or dementia is reasonable. Skillful neglect is the treatment of choice for these patients. An Ace wrap with sufficient padding to protect the elbow is the only requirement. Patients with wide separation of fracture fragments lose significant, but not complete, elbow extension power. Late pain from an ununited displaced olecranon fracture generally is not a problem, but the extensor power is compromised. Approximately 70% of the extensor power is estimated to be lost when the fracture is displaced more than 1.5 cm. Nondisplaced fractures with intact extensor mechanisms may be treated nonoperatively. Three weeks of casting usually is sufficient. The elbow can be placed at any degree of flexion. Displacement generally can be reduced by placing the elbow in more extension. Patients can be comfortable with the elbow extended 135°. However, it is often more convenient to immobilize the elbow at 90°. Regaining both flexion and extension can be difficult. At first, patients are cautioned to limit flexion to 90°, at least until evidence of radiographic healing is satisfactory.

What is the presentation for clavicle fracture?

Because of the clavicle's subcutaneous position, injury is often obvious and is confirmed at the time of initial observation. Despite the relatively simple diagnosis, some aspects of clavicle fractures must be addressed. In particular, note any skin abrasions or other wounds in proximity to the fracture site to determine if the fracture is an open injury. Note any tenting of the skin, as this will likely cause pressure necrosis of the skin and an increased chance that the fracture will become an open injury. In addition to observing the status of the skin, perform a complete neurovascular examination of the involved extremity, including comparative blood pressure measurements if injury to the subclavian artery is suspected. Keep in mind that the excellent collateral circulation to the upper extremity may mask injury to the subclavian artery. The subclavian vessels and brachial plexus run in close proximity to the clavicle and are at risk for injury with displaced clavicle fractures. Katras et al reported on a series of 7 patients with blunt trauma and an injury to the subclavian artery.[13] Four of the 7 patients had an associated clavicle fracture, and all 4 of these patients were involved in motor vehicle accidents. Only 1 of the 4 had a brachial plexus injury in addition to the clavicle fracture and subclavian artery injury. Kendall et al reported a fatality from an isolated clavicle fracture from transection of the subclavian artery.[14] This was the first such report in the literature; the fatality may have been due to the fact that the fall was not witnessed and the patient had lain unassisted for an unknown period of time. The patient never regained spontaneous circulation, and the injury to the subclavian artery was diagnosed at autopsy. The postmortem examination revealed a midclavicular fracture with transection of the subclavian artery. A 2.6-L hemothorax and damage to parietal and apical pleura were noted, but no other injuries were present. Although this case is unique, it does emphasize the need to be aware of the potentially catastrophic complications of damage to the vascular structures in close proximity to the clavicle. Some findings on physical examination and workup for trauma that should alert the physician to the possibility of an injury to the subclavian vessels are hematoma overlying the clavicle, presence of a bruit over the region, diminished or absent pulses in the extremity, first rib fracture, brachial plexus injury, and a wide mediastinum on chest radiograph.

What is the background information of medial neuropathy?

Carpal tunnel syndrome (CTS) is the most common focal peripheral neuropathy. CTS is caused by entrapment of the median nerve at the wrist as it traverses through the carpal tunnel. Acute CTS is a rare compartment syndrome of the carpal tunnel that occurs after major trauma, typically distal radius fracture. Diagnosis is based on clinical history and examination and does not require electrophysiological testing to proceed with surgery as soon as possible to relieve the pressure on the median nerve. Chronic CTS is a much more common condition and of more gradual onset, with intermittent symptoms initially and slow progression. The condition is often bilateral and almost always more prominent in the dominant hand. The syndrome is characterized by pain, paresthesia, and weakness in the median nerve distribution of the hand that are typically provoked by sleep or activities involving repetitive hand use. Electrodiagnostic studies are helpful to confirm the diagnosis. Treatment in mild cases may be nonsurgical and includes wrist splinting, but many patients require either open or endoscopic carpal tunnel release surgery with usually excellent outcome.

What is the presentation of anterior shoulder instability?

Children presenting with a dislocated shoulder may relate a couple of possible mechanisms. These mechanisms occur in a number of ways and are similar to those in adults. Most commonly, the child falls on the outstretched hand, forcing the arm into abduction and external rotation, levering the humeral head out of the glenoid cavity. Activities promoting this injury include contact sports, falls from heights, fights, and motor vehicle accidents. Other mechanisms have been described, including elevation with external rotation and direct blows. A history of prior dislocations suggests a high likelihood of anterior glenohumeral instability. Studies have shown a 70-100% recurrence rate in various population groups of patients younger than 20 years. As with physical examinations of any joint, beginning by observing the shoulder is important. Note any atrophy of the biceps, supraspinatus, or infraspinatus. Gross deformities can also suggest the direction of a dislocation. Range of motion of the shoulder must be tested for restriction or hypermobility. Atraumatic instability generally manifests with hypermobility of the shoulder, whereas traumatic instability typically results in bilaterally symmetric motion. Generalized joint laxity is demonstrated by extending the elbow, wrist, metacarpal-phalangeal, and distal interphalangeal joints. External rotation can be increased as much as 28° or decreased as much as 14° after dislocation. Next, the examiner manually assesses translation of the humeral head in the glenoid fossa. The humeral head is grasped in one hand, and the clavicle and scapula are stabilized in the other as the examiner pushes anteriorly and posteriorly. This is known as a shoulder drawer sign. Compared with the unaffected shoulder, the affected shoulder often demonstrates increased laxity. Remember that as much as 50% of posterior translation may be normal. A sulcus sign is demonstrated by pulling inferiorly on the relaxed shoulder. A sulcus observed between the acromion and proximal humeral head is considered a positive finding. This finding indicates that the shoulder has multidirectional instability. The key finding in anterior glenohumeral instability is a positive apprehension test. The arm is placed in abduction, extension, and external rotation while stressing it in anterior translation. If the patient becomes apprehensive and reports pain, this is considered a positive finding. The relocation test involves placing the shoulder in the position of apprehension and applying a posteriorly directed force on the humeral head. The result is considered positive if this relieves the patient's apprehension. Impingement signs must also be evaluated because as many as 10% of patients experience impingement after dislocation. Evaluate for the Hawkins sign and perform the Neer impingement test.

What is the medical care for medial neuropathy test?

Conservative treatment is usually recommended for mild-to-moderate carpal tunnel syndrome (CTS), at least initially. Guidelines from the American Academy of Orthopaedic Surgeons suggest that if symptoms fail to resolve within 2-7 weeks with a particular treatment, the clinician should move on to a different form of therapy.[39] Wrist splint: A lightweight plastic/Velcro splint in a neutral position that allows semifree finger movement is recommended. The wrist splint should be worn primarily at night (regularly) and as needed during daytime (during manual activity). Precautions should be taken to prevent a persistently stiff wrist caused by prolonged immobilization. Activity modification: Reduce wrist flexion, extension, rotation, finger flexion, and forceful gripping. A local steroid injection[40] may be particularly helpful in patients with mild CTS and intermittent symptoms. Local injection may also have a diagnostic or prognostic role as a predictor of response to surgical release. See a detailed description in the eMedicine article Steroid Injection, Carpal Tunnel. Corticosteroids (methylprednisolone acetate [DepoMedrol] 10-20 mg or triamcinolone acetonide [Kenolog] 10-20 mg) are injected adjacent (proximal) to the carpal tunnel, after local anesthesia. Care must be taken not to inject the carpal tunnel, any tendon, or the nerve itself. Such an injection may increase the intracarpal tunnel pressure and cause additional nerve injury. The effect of steroid injections may be seen within a few days and often lasts for several weeks or months. The effect is usually only temporary and usually wears off by 1 year. It may be particularly helpful in pregnant patients or those with temporary medical conditions such as hypothyroidism. Repeated use beyond 2-3 injections is not recommended due to the greater risk of damage to the flexor tendons, including tendon rupture. Other complications include increased median nerve deficit, local infection, and reflex sympathetic dystrophy. A study comparing daily application of lidocaine 2.5% plus prilocaine 2.5% (EMLA) with a single injection of methylprednisolone acetate found that the anesthetic cream was effective and well tolerated.[41] Oral medications are listed in the Medication section below. A short course of nonsteroidal anti-inflammatory medication is often recommended, if there is no contraindication, without clear evidence of its effectiveness. Alternative therapies: These include acupuncture; yoga-based programs for stretching, strengthening, and relaxation; and chiropractic therapy.

What is the presentation of coronoid fracture?

Coronoid fractures are usually seen in patients with elbow dislocations. The patient usually presents with a history of a fall on the outstretched hand and a deformity of the elbow. The presence of an unstable reduction is suggestive of an associated coronoid or radial head fracture. An irreducible dislocation, on the other hand, should arouse suspicion of soft tissue (brachialis, median nerve) or bony (medial epicondyle) interposition in the joint. Distal vascularity and neurologic status should always be tested in these cases. emedicine.medscape.com/article/1230817-‐‑overview#showall 2/6 9/3/12 Coronoid Fracture In children, coronoid fractures have a bimodal age distribution, with peaks at age 8-9 years and at age 12-14 years.[13] Coronoid fractures in children are often associated with elbow dislocations, olecranon fractures, medial epicondyle fractures, or lateral condyle fractures. In children younger than 6 years, Blasier has described an unusual flap injury of the unossified coronoid in which the elbow is dislocated and a small flap of the articular surface gets flipped back into the joint.[14] This usually appears on the lateral radiograph as a small flake of bone in the anterior portion of the joint. This flake is a clue to the underlying pathoanatomy and the extent of injury.

What is the etiology of a for coronoid fracture?

Coronoid fractures were believed to result from avulsion of a bony fragment of the coronoid by the brachialis, which inserts onto the coronoid process. This, however, does not explain the mechanism of type 1 and some type 2 fractures, as the brachialis attaches to the base of the coronoid process of the ulna. (See Workup, Imaging Studies, below, for a discussion of fracture types.) These fractures probably occur from shear forces at the time of the dislocation when the trochlea pushes off a piece of the coronoid.Coronoid fractures were believed to result from avulsion of a bony fragment of the coronoid by the brachialis, which inserts onto the coronoid process. This, however, does not explain the mechanism of type 1 and some type 2 fractures, as the brachialis attaches to the base of the coronoid process of the ulna. (See Workup, Imaging Studies, below, for a discussion of fracture types.) These fractures probably occur from shear forces at the time of the dislocation when the trochlea pushes off a piece of the coronoid.

What is the frequency of olecranon fracture?

Despite the fact that the olecranon is a very heavy, strong process of bone, it is fractured rather frequently in adults. This is due partly to its exposed position on the point of the elbow, where most direct injuries to the elbow occur, and partly to the tremendous cross strain put on the olecranon during falls on the flexed forearm. The olecranon process is rarely broken in children, because in early life, it is short, thick, and relatively much stronger than the lower end of the humerus. Usually, children sustain supracondylar fractures of the humerus instead. Open fractures occur in 2-31% of cases. Neurologic injuries to median, radial, or ulnar nerves may occasionally occur. Ulnar neurapraxia has been reported in 2-5% of cases. Generally, symptoms resolve with conservative treatment, but late neurolysis or transposition may occasionally be required.

What are the images for medial humeral fracture?

Diagnosis is usually based on standard anteroposterior (AP) and lateral radiographs of the affected elbow.[7, 26, 29, 30] Distinguishing a fracture of the medial epicondyle from a fracture of the medial condyle can be difficult in the developing elbow.[1, 2, 3, 4, 5, 6, 7, 8, 30] Since cartilaginous structures are usually not visible radiographically, the exact location of injury may not be obvious. With its complicated and variable pattern of ossification, trauma to this region presents a difficult diagnostic challenge. Because the medial epicondyle lies largely outside the joint capsule, fractures of this structure usually do not produce distention of the joint capsule. Therefore, if a positive fat-pad sign accompanies soft-tissue swelling, fracture extension distally into the joint capsule to include the trochlear ossification center and medial condyle should be considered. Radiographic clues to unstable medial condyle fracture in a young child include soft-tissue swelling, a chip or flake of bone from the metaphysis, and the presence of a positive fat-pad sign.[30] Widening or irregularity of the apophyseal physis may be the only sign in slightly displaced or nondisplaced fractures of the medial epiphysis. If the medial epiphysis is absent, the fragment may be incarcerated totally into the joint or hidden by the overlying ulnar or distal humerus. The lack of a fat-pad sign cannot be used to exclude medial condyle injury. If the joint capsule is ruptured, no fat-pad sign is exhibited. Therefore, it may be necessary to examine the elbow under anesthesia to determine if instability is present that would indicate a more extensive injury. A widely displaced fracture-separation of the medial epicondyle in a patient whose trochlear ossification center has not yet appeared can indicate that the cartilaginous trochlea may also be fractured and attached to the epicondyle. This possibility should be considered and may warrant surgical exploration. Arthrography may be used to determine the extent of a fracture and to help distinguish an epicondyle fracture from a condyle fracture.[31] Magnetic resonance imaging may be used to evaluate soft-tissue injury and may be helpful in evaluating cartilaginous injury. Interobserver and intraobserver reliability is poor for assessing epicondyle displacement on plain radiographs.[32] Three-dimensional CT scanning is a more accurate way to assess displacement of an epicondylar fragment and indeed plain films may underestimate displacement by up to 1 cm.

What is the Diagnostic procedure for MDI?

Diagnostic arthroscopy must always be preceded by a thorough EUA. In an EUA, it is important to examine both shoulders, comparing the symptomatic side with the asymptomatic side. Typically, with relaxation afforded by general anesthesia, the clinical diagnosis is obvious, even if unsuspected preoperatively. Again, increased anterior and posterior laxity that exceeds the normal range combined with a positive sulcus sign is easily demonstrated. Arthroscopy can be performed with the patient in either the beach-chair or lateral decubitus position. Surgeon preference may dictate patient position considerations. However, if open anterior capsular shift is planned, an upright or semi-upright beach-chair position allows for ease of transition to open surgery without significant modification of position. If arthroscopic management of capsular patholaxity is planned, there is little difference between these variations. To facilitate a complete and systematic glenohumeral joint evaluation, views from both anterior and posterior portals are necessary. This approach allows more thorough labral and capsular visualization. Moreover, it is essential to evaluate for concomitant pathology, including articular surface rotator cuff pathology, SLAP lesion, labral tears, Bankart lesion and Hill-Sachs defect, and humeral avulsion[17] of the glenohumeral ligament (HAGL) lesion. All of these are atypical in straightforward MDI. Typical characteristics of MDI are a loose capsule with poor development of the glenohumeral ligaments and a normal, attenuated, or unimpressive labrum. Capsular tissues typically are thin. The axillary recess or pouch and the rotator cuff interval are spacious and patulous. The articular surfaces most often are normal or show minimal chondromalacia. A Hill-Sachs lesion is absent. Moving the arthroscope within the shoulder of an individual with MDI is easy, even without traction in the beach-chair position. A "positive drive-through sign" is typical. This means that it is very easy to move the arthroscope across the glenohumeral joint between the humeral head and glenoid fossa without axial arm traction or distraction. Subluxation of the humeral head on the glenoid is obvious, even without supplemental traction. Finally, assessment of the subacromial space also is important, especially in the patient with suggestive impingement history and findings. Evaluation in this location includes scrutiny of the bursal cuff surface, as well as the coracoacromial arch, for signs of cuff and subacromial abrasion. A patient with secondary impingement from an underlying glenohumeral instability may demonstrate impressive subacromial findings that are suggestive of impingement. These findings should provoke consideration of primary versus secondary impingement and review of the clinical presentation, EUA, and glenohumeral arthroscopic findings so that appropriate management is selected.

What is the presentation for Monteggia Fracture?

Following the above mechanism, patients present with elbow pain. Depending on the type of fracture and severity, they may experience elbow swelling, deformity, crepitus, and paresthesia or numbness. Some patients may not have severe pain at rest, but elbow flexion and forearm rotation are limited and painful. The dislocated radial head may be palpable in the anterior, posterior, or anterolateral position. In type I and IV lesions, the radial head can be palpated in the antecubital fossa. The radial head can be palpated posteriorly in type II lesions and laterally in type III lesions. The skin should be closely inspected to ensure an open fracture is not present. Pulses and capillary refill should be documented. A negligible hematoma may be present at the site if no direct trauma is associated. Motor function must be thoroughly tested because the branches of the radial nerve can become entrapped, causing weakness or paralysis of finger or thumb extension. The sensory branch is not usually involved but also should be checked. Bado indicated that spontaneous recovery is the usual course, and exploration is appropriate if function does not begin to return within 2 to 3 months. Monteggia fractures in the pediatric population typically manifest with unique features that have led to a decreased emphasis on the direction of the radial head dislocation and an increased focus on the character of the fracture of the ulna. When the various fracture types occur in the immature bone of children, distinct patterns result and influence treatment considerations. Plastic deformation of the ulna in association with anterior radial head dislocation represents up to 31% of anterior Monteggia lesions. Poor recognition of this injury pattern can lead to recurrent or persistent dislocation because the radial head reduction remains unstable until the plastic deformity is corrected. Incomplete fractures of the ulna and greenstick fractures represent other variants that must be corrected along with the radial head dislocation. Monteggia fractures in children based on type of ulnar injury are as follows: Plastic deformation Incomplete (greenstick or buckle) fracture Complete transverse or short oblique fracture Comminuted or long oblique fracture

What is the epidemiology of medial humeral fracture?

Fracture of the medial condyle is very rare, especially when compared to frequency of other elbow fractures. In one study, radiographs of 589 elbow fractures in children younger than 16 years were reviewed. The most common fractures were the supracondyle fracture (55%), radial neck fracture (14%), lateral humeral condyle fracture (12%), medial epicondyle fracture (8%), and olecranon fracture (7%).[19] No cases of medial condyle fracture were reported.[20] Fractures of the medial condyle are so rare that they receive little coverage in most popular textbooks and may not be mentioned at all in others. The incidence is described in the literature from less than 1-2% of all elbow injuries in children. Most displaced medial condyle fractures occur when the trochlea is not ossified completely. In some studies, the average age has varied (ie, age 10, 11, 9.5, and 7 years). Although a child of any age can sustain this fracture, it is most common during the developing years (ie, in children aged 7-14 years). Medial epicondyle fractures are much more common than medial condyle fractures. In a large combined series of 5228 fractures of the distal humerus, medial epicondyle fractures constituted 14.1% of all distal humeral fractures and 11.5% of all fractures occurring around the elbow. Medial epicondyle fractures are 4 times as likely to occur in males, and most cases occur in children aged 9-14 years. Peak incidence is in children aged 11-12 years. The reported incidence of association with elbow dislocation reaches 55% in some series, and the fragment may be incarcerated in the joint in approximately 15-18% of cases

What is the epidemiology for ulnar neuropathy?

Frequency United States In the general population, abnormalities in the ulnar nerve at the elbow in asymptomatic subjects are common (about 40%). The elbow is the second most common site of nerve entrapment in the upper extremity, the first being the wrist (ie, carpal tunnel syndrome). Mortality/Morbidity Delayed recognition of ulnar neuropathy at the elbow or wrist or unsuccessful surgical intervention can lead to loss of function due to prolonged axonal degeneration. Sex No gross anatomical differences in the course of the nerve are noted between the sexes. However, the following have been noted.[28] Men develop perioperative ulnar neuropathies[15] at the elbow[29] more frequently than women.[30] Women have more fat content in the medial elbow overlying the tubercle of the ulnar coronoid process (2-19 times more). The tubercle of the coronoid process is 1.5 times larger in men. Age According to the older literature, most cases of ulnar compression neuropathy occur in patients older than 35 years.[31] This is consistent with an independent anatomical study of 200 cadavers showing that the ulnar nerve is largest at the entrance to the cubital tunnel and that this enlargement is of maximal size in males older than 35 years.[32] A prospective study of 76 patients showed that increased age is highly correlated with a greater tendency toward ulnar neuropathy.[30]

An 18 year old female basketball player sustained a Grade III inversion sprain to the right ankle during the first basketball game of the season. She presents to your clinic for physical therapy following a physician visit with her family physician the previous day with an order to evaluate and treat. Radiological studies with X-rays completed and increased joint space noted but no fractures diagnosed. She presents with bilateral Axillary crutches in a right Non-weight bearing 2 point gait pattern. Her right ankle is not braced in any manner and is moderately swollen with an ecchymosis pattern on the lateral aspect of the ankle and foot consistent with reported injury. When questioned re: weight bearing status, the young female reports that she just felt that staying off it was the right thing to do but received no specific instructions regarding her walking pattern. Her ankle AROM is right: 0/5/55 and left 10/0/80 in the sagittal plane AND right 0/0/15 (inversion) left 15/0/30(inversion). Upon confirmation with the physician's office - there are no weight-bearing restrictions. Which of the following activities would be completed to establish a normal walking pattern?

Gait training in a WBAT 3 point, step through gait pattern with necessary assistive device, coupled with active range of motion exercises to normalize ankle mobility Correct: This individual will need decreased weight bearing, and as such, the crutches can assist. Otherwise, the AROM exercises will help to restore motion. Calf stretching tends to be contraindicated in a grade III inversion sprain.

What are the mechanics of the posterior drawer test?

Gerber and Ganz3 also described this test: "The patient must be supine. The examiner stands level with the af- fected shoulder. Assuming the left shoulder is being tested, he grasps the patient's proximal forearm with his left hand, flexes the elbow to about 120 degrees, and positions the shoulder into 80 degrees to 120 degrees of abduction and 20 degrees to 30 degrees of forward flexion. The examiner holds the scapula with his right hand, with his index and middle fingers on the scapular spine; his thumb lies immediately lateral to the coracoid process, so that its ulnar aspect remains in contact with the coracoid while performing the test. With his left hand, the exam- iner slightly rotates the upper arm medially and flexes it to about 60 degrees or 80 degrees; during this manoeuvre, the thumb of the examiner's right hand subluxates the humeral head posteriorly. This posterior displacement can be appreciated as the thumb slides along the lateral as- pect of the coracoid process toward the glenoid, and the humeral head abuts against the ring finger of the exam- iner's right hand. This manoeuvre is painfree but often associated with a slight to moderate degree of apprehen- sion, enabling the patient to identify the position of insta- bility with certainty." An analysis of the interobserver reliability of the sulcus sign and the laxity tests using Altchek's grading system showed that overall reproducibility was 47% with a kappa value of less than 0.5.10 The intraobserver reproducibility was only 47%. Most of this discrepancy was with grades 0 and 1. When these grades were combined, the intraob- server reproducibility increased to 73%.

What is the history for proximal humeral fracture?

Hippocrates first documented a proximal humerus fracture in 460 BC and treated it with traction. In 1869, to improve treatment, Krocher classified fractures of the proximal humerus. In 1934, Codman developed a classification that divided the proximal humerus into 4 parts, based on epiphyseal lines. In 1970, Neer's classification expanded on the 4-part concept and included anatomic, biomechanical, and treatment principles, providing clinicians with a useful framework to diagnose and treat patients with these fractures.[4] Treatment initially consisted of closed reduction, traction, casting, and abduction splints. In the early 1930s, operative treatment for displaced fractures gained popularity, which continued in the 1940s and 1950s. Humeral head replacement for severely displaced fractures of the proximal humerus was introduced the 1950s. In the 1970s, the AO/ASIF (Arbeitsgemeinschaft für osteosynthesefragen/Association for the Study of Internal Fixation) group popularized plates and screws for fracture fixation, and humeral head prostheses were redesigned. Currently, fixation methods that involve limited fixation and limited dissection are becoming more popular, and prosthetic replacement for severe fracture is being refined further

What is the background for clavicle fracture?

Historically, clavicle fractures have been considered best treated nonoperatively, with good outcomes. Management typically included the use of either a shoulder sling or a figure-of-eight brace. The vast majority of these fractures healed, with variable amounts of cosmetic deformity. Studies have examined the different patterns of displacement and clinical outcomes of clavicle fractures according to their location. The images below illustrate clavicle fractures. Medical literature has focused predominantly on fractures of the middle and lateral clavicle. The literature is still lacking concerning the management of medial clavicle fractures. According to current literature, medial clavicle fractures respond well to nonoperative management. Controversy remains concerning operative versus nonoperative treatment of middle and lateral clavicle fractures.[1, 2, 3, 4] Recent studies In a multicenter, prospective trial by the Canadian Orthopaedic Trauma Society of 132 patients with a displaced midshaft fracture, outcome and complication rates were compared for nonoperative treatment and plate fixation. Constant Shoulder scores and Disability of the Arm, Shoulder and Hand (DASH) scores were greatly improved in the operative fixation group. Mean time to radiographic union was 28.4 weeks in the nonoperative group and 16.4 weeks in the operative group. There were 2 nonunions in the operative group and 7 in the nonoperative group. Symptomatic malunion occurred in 9 of the nonoperative patients and in none of the operative ones. Most complications in the operative group were hardware-related (5 cases of local irritation or prominence of the hardware, 3 wound infections, and 1 mechanical failure). At 1 year after injury, the operative-group patients were more likely to be satisfied with the appearance of the shoulderandwith the shoulder in general than the nonoperative-group patients.[5] Smekal et al compared elastic stable intramedullary nailing (ESIN; 30 patients) with nonoperative treatment (30 patients) of fully displaced midshaft clavicular fractures in adults 18 to 65 years of age. They were randomized to either operative or nonoperative treatment with a 2-year follow-up. Fracture union occurred in all patients in the operative group; nonunion occurred in 3 nonoperative patients. Medial nail protrusion occurred in 7 cases, and implant failure with revision surgery was necessary in 2 patients after additional trauma. DASH scores were lower in the operative group during the first 6 months and 2 years after trauma. Constant scores were significantly higher after 6 months and 2 years after intramedullary stabilization. Patients in the operative group showed a significant improvement of posttraumatic clavicular shortening and were also more satisfied with cosmetic appearance and overall outcome.

What is the staging of lateral humeral condyle fracture?

In 1964, Milch described an anatomic classification system, dividing lateral condyle fractures into 2 subtypes. These subgroups are based on the location of the fracture line.[4, 8] A Milch type I fracture extends through the ossification center of the lateral condyle and exits at the radiocapitellar groove, as depicted in the images below. This produces a Salter-Harris type IV fracture pattern. The lateral crista of the trochlea remains intact and therefore has less tendency to dislocate laterally. This pattern is less common. A Milch type II fracture extends across the physis and exits through the apex of the trochlea, as depicted in the images below. This produces a Salter-Harris type II or type IV fracture pattern. The lateral crista is in the fracture fragment, and the trochlea is no longer intact, rendering the elbow unstable. This is the more common fracture pattern. n 1975, Jakob described a classification system based on displacement of the fracture fragment. This system is subdivided into 3 stages, and some authors believe that it is more useful than the Milch classification system.[9] A Jakob stage I fracture is nondisplaced with an intact articular surface. In a stage II injury, the fracture extends through the articular surface, and there is moderate rotational displacement. A stage III injury demonstrates complete displacement and capitellar rotation with elbow instability.

What is the procedure for little league elbow syndrome?

In a review by Chumbley et al, the authors described a lidocaine nerve block that may be used to diagnose radial tunnel syndrome.[12] Injection of 1 mL of lidocaine (1%) is given 4 fingerbreadths distal to the athlete's lateral epicondyle. This injection relieves pain and causes a deep radial palsy in radial tunnel syndrome. However, when, at another time, a second injection is given more proximally in the area of the lateral epicondyle and the symptoms are not alleviated, the diagnosis of radial tunnel syndrome is confirmed.

What is the pathophysiology of anterior instability?

In an unstable shoulder, many findings are possible. Each finding can occur alone or in combination with other lesions. These lesions include the Bankart lesion (in 85% of cases), Hill-Sachs lesion (in 77% of cases), anterior glenoid rim damage (in 73% of cases), capsular redundancy, subscapularis deficiency, and glenoid fossa deficiency. Absence of pathological findings is also possible. In 1923, Bankart described the "essential lesion" in posttraumatic anterior glenohumeral instability as the detachment of the capsule from the fibrocartilaginous glenoid ligament. In this lesion, the humeral head translates forward, shearing the inferior glenohumeral ligament (IGHL) with the anteroinferior labrum from the glenoid rim. Rowe's review of 28 patients in which he examined shoulder pathology after traumatic anterior dislocation disputed Bankart's claim to the essential lesion. Rowe's results showed "there was no evidence that there is a single essential lesion responsible for the recurrent dislocations of the shoulder." The Bankart lesions occurred in 27-100% of cases. Other lesions were just as variable. Subscapularis laxity ranged from being present in every case to not being present in any cases. Hill-Sachs lesions were present in 26-100% of cases. Anterior glenoid trauma of all variations occurred in 2-52% of cases. The next logical question is which of these lesions actually causes the recurrent instability observed after traumatic dislocations. This remains a debated topic. The most accurate conclusion is the one Rowe came to in 1978 when he declared that no single lesion is responsible for the recurrent dislocations of the traumatized shoulder.

What is the etiology for posterior elbow dislocation?

In children under 10 years, PEDs are the most common type of joint dislocation.[1] In adults, they are the second most commonly dislocated joint proceeded by shoulder dislocations.[1][3][4][5][6] Elbow dislocations annually affect between 6 and 7 people per 100,000.[4] Approximately 90% of all elbow dislocations are directionally classified as posterior or posterolateral and are more commonly seen in the non-dominant upper extremity (UE).[1][3][4] Typically, this injury is caused by a traumatic fall onto an outstretched arm resulting in an hyper-extension injury.[5][6] However, more recent research has suggested that axial compression, elbow flexion, valgus stress, and forearm supination lead to a rotational displacement of the ulna on the distal humerus.[1] Most commonly, the dislocation is associated with a damaged or torn anterior capsule.[7][8] PED can be classified as simple (74%) or complex (26%). A simple dislocation is absent of fractures while a complex dislocation has related fractures.[4] Fractures may exist on the radial head, coronoid process, olecranon, humeral condyles, or capitellum.[7] These fractures may lead to disruption of the medial collateral ligament (MCL), lateral collateral ligament (LCL), or interosseous membrane.[7] 'Terrible triad' is a term used to describe a severe complex dislocation with intra-articular fractures of the radial head and coronoid process.[4] Elbow dislocations are staged depending on the disruption of the following stabilizers: the ulnohumeral articulation, MCL, and LCL.

What are the outcome for SLAP surgery?

In general, good results have been reported with arthroscopic superior labral lesions repairs. Morgan et al reported on their results in 102 patients.[6] About 83% of their patients had excellent results, and 14% had good results. All pitchers returned to pitching, 84% at preinjury level. Pagnani et al reported that 12 of 13 overhead athletes were able to return to full preinjury level of activity after an arthroscopic SLAP (superior labrum, anterior and posterior) repair.[42] Field and Savoie,[43] as well as Burkhart et al,[17] reported similar findings. In Field and Savoie's study, all of the athletes were able to return to sports activities without limitation. Kim et al evaluated their results in 34 individuals who underwent arthroscopic suture anchor repair of an isolated SLAP tear.[24] Approximately 94% had a satisfactory result as determined by using the University of California Los Angeles (UCLA) shoulder score. Thirty-one (91%) of the patients regained their preinjury level of function. However, they did note poorer outcomes in overhead athletes as compared with those not participating in overhead athletic activities.

What is the epidemiology of anterior instability?

In studies of anterior glenohumeral instability, a wide array of incidences have been proposed. Most report an average age of initial shoulder dislocation early in the third decade of life. The vast majority, 85-95%, of these are anterior dislocations. One fourth of all patients with dislocations present with a family history of the same problem. Rowe's 1956 analysis of 500 shoulder dislocations found only 8 occurred in children younger than 10 years; the largest cohort of dislocations occurred in the subsequent decade of life, in patients aged 10-20 years.[6] The age at the time of dislocation is the biggest determinant of future recurrence. Recurrences result from anterior glenohumeral instability and occur most commonly in patients younger than 20 years. Reported recurrence rates in patients younger than 20 years vary from 70-100%.

What is the pathophysiology of medial humeral fracture?

In the developing elbow, fracture through the medial condyle involves part of the cartilaginous or partially ossified trochlea and the ossified medial epicondyle. With the insertion of the common flexor tendon of the forearm and the medial collateral ligament on the medial epicondyle, the fracture fragment tends to rotate around the axis of the epicondyle and can present at various degrees of rotation. Complete rotation puts the fracture surface facing the anterior and medial side of the elbow, with the medial epicondyle pulled distally and the articular surface facing posteriorly and laterally. In addition, the surrounding soft tissues can be torn, and there may be damage to the articular surface of the ulna. Damage to the articular capsule or medial collateral ligament of the elbow may be present. The ulnar nerve and the vasculature surrounding the elbow joint are also at risk. The blood supply to the epiphysis enters with the attachment of the medial collateral ligament and the common flexor tendon. Separation of these structures at the time of injury or at surgery may lead to avascular necrosis. The medial epicondyle fragment is usually displaced distally, although at least 2 cases of displacement of the fragment proximally have been reported (see images below). The fracture line usually involves only the apophysis; however, occasionally a fragment of metaphyseal bone is found attached to the avulsed fragment. If the fragment is incarcerated within the joint, the fragment may become adherent to the coronoid process of the ulna. When the fragment is incarcerated within the joint, the universal finding is a thick fascial band that binds the ulnar nerve to the underlying muscle. This thick fascial band is responsible for ulnar nerve dysfunction, either acutely or as a late finding. Other elbow fractures may be associated with medial epicondyle fractures, and care must be taken to recognize the full extent of the injury. Associated injuries include radial neck fractures, olecranon fractures, and coronoid process fractures.

What phase of throwing will the patient report increase elbow pain?

Inathleteswithmedialelbowinstability,nearly 85% will experience pain during the acceleration phase of Downloaded from http://ajs.sagepub.com by TERESA SCHUEMANN on May 30, 2007 © 2003 American Orthopaedic Society for Sports Medicine. All rights reserved. Not for commercial use or unauthorized distribution. Vol. 31, No. 4, 2003 Elbow Injuries in Throwing Athletes 623 throwing, whereas less than 25% will experience pain during the deceleration phase.

What is the indications for surgery for MDI?

Indications for surgical treatment of multidirectional instability (MDI) include the presence of persistent symptoms to a disabling degree and failure of conservative management, including a supervised rehabilitation program and a trial of activity modification or restriction. A reasonable trial of conservative treatment is 3-6 months. Any patient for whom conservative management has failed may be counseled regarding the option of surgical treatment. The following points must be considered: Does the patient have sufficient present disability to make surgery a worthwhile endeavor? Has the patient shown satisfactory effort and dedication to the preoperative rehabilitation program? Is the patient willing to continue postoperative rehabilitation for at least 6 months, and does the patient understand the importance of shoulder rehabilitation and muscle strengthening to the stability of his or her shoulder? Is the patient willing to comply with postoperative limitations with regard to immobilization, activity limits, and work and sports restrictions? Is the patient willing to accept the possibility of lost ROM? Is the patient willing to risk axillary nerve injury and other operative complications?

What are the indications for Monteggia fracture?

Indications for treatment of Monteggia fractures are based on the specific fracture pattern and the age of the patient (ie, pediatric or adult). Most pediatric fracture patterns can be managed conservatively with closed reduction and long arm casting. However, most adult fractures require open reduction and internal fixation techniques. The radial head dislocation should be reduced emergently. Closed reduction under sedation should be performed within 6-8 hours of the injury. This is usually achieved with supination of the forearm, but it may require traction and direct pressure on the radial head. If closed reduction is unsuccessful, the patient should be taken to the operating room within this same time frame for open reduction. Delay in reduction of the radius may lead to permanent articular damage, further nerve injury, or both. An open fracture requires emergent operative intervention. In closed injuries, once the radial head is reduced, the forearm is splinted and operative fixation of the ulna fracture may be carried out in an elective fashion. Adults usually require operative internal fixation to stabilize the ulna and prevent further displacement forces on the radiocapitellar joint. Closed Monteggia fractures in pediatric patients are generally treated in a closed fashion. A posterior long arm splint with the elbow in 90° of flexion and full supination is the immobilization method of choice for types I, III, and IV. Type II injuries (posterior lesions) are best splinted in 70° elbow flexion with supination.

What are the mechanics for anterior slide test?

Kibler7 described this test in 1995. "The patient is exam- ined either standing or sitting, with their hands on the hips with thumbs pointing posteriorly. One of the exam- iner's hands is placed across the top of the shoulder from the posterior direction, with the last segment of the index finger extending over the anterior aspect of the acromion at the glenohumeral joint. The examiner's other hand is placed behind the elbow and a forward and slightly supe- riorly directed force is applied to the elbow and upper arm. The patient is asked to push back against this force. Pain localized to the front of the shoulder under the examiner's hand, and/or a pop or click in the same area, was consid- ered to be a positive test. This test is also positive if the athlete reports a subjective feeling that this testing ma- neuver reproduces the symptoms that occur during over- head activity." This test is depicted in Figure 4. The re- sults showed a sensitivity of 78.4% and a specificity of 91.5%. The authors commented that the test was useful as Figure 3. The compression-rotation test. American Journal of Sports Medicine Figure 4. The anterior slide test. an aid to diagnosis, but was not in itself sufficient to be relied on completely.

What is the clinical presentations for lateral epicondylitis?

Lateral epicondylitis has an annual incidence of 1-3% within the general population. [5] The difference between men and women on lateral epicondylitis is still controversial. Patients with lateral epicondylitis are typically 35 to 54 years. It's less common in people under 30 years. Symptoms last, on average, from 6 months to 2 years. 89% of the patients recover within 1 year without any treatment except perhaps avoidance of the painful movements. [5][6][7] Patients often report weakness in their grip strength or difficulty carrying objects in their hand, especially with the elbow extended. They have complaints of pain just distal to and localized tenderness over the lateral epicondyle. Patients will commonly have pain with palpation of the lateral epicondyle, resisted wrist, or second or third finger extension (Cozen's sign). [8]

What is the history for little league elbow syndrome?

Little league elbow syndrome occurs most commonly in pitchers, but it is also seen in infielders, catchers, and outfielders. This condition can also occur in other overhead or throwing sports, such as tennis and football (quarterback position). It is important to identify the player's position on the sports team during history taking, because this makes determining the magnitude of the stress placed on the elbow and the subsequent risk of injury easier. Skeletal age is an indicator of the stage of skeletal maturity and is a major determining factor in regard to these potential types of injuries. Little league elbow injuries during childhood are usually due to repetitive microtrauma to the apophysis and ossification center of the medial epicondyle. During adolescence, increased throwing force and valgus stress result in avulsion, delayed union, or nonunion of the medial epicondyle. In young adulthood, the medial epicondyle is fused, and injuries to the UCL are more common. Throwing history is important. Types of pitches, an accurate pitch count of approximate numbers of competitive pitches per game per week and/or season is necessary information. (See the 2008 USA Baseball Medical & Safety Advisory Committee recommendations for youth pitch counts.[11] )The level of play and time of season should be noted. Recent changes in pitch types, counts, or other alterations in training should be carefully noted. For example, fastballs and change-up pitches result in less medial elbow stress than curveballs and sliders. Curveballs thrown at a young age, regardless of previous pitching experience, are associated with an increased risk of little league elbow syndrome and more serious injuries such as medial epicondylar avulsion fractures due to shear forces over a immature growth plate. It is vital to obtain the location, timing, and duration of symptoms (usually pain). Elbow pain in a thrower is usually a chronic overuse injury. However, an acute inciting event that changes or worsens the symptoms may prompt an athlete to seek an evaluation. Pain is most commonly localized to the medial epicondyle, although patients may also present with lateral or posterior elbow pain. Medial elbow pain during the cocking and/or acceleration phases of throwing is typical. Pain during the deceleration phase is more likely to be associated with posterior elbow injuries. Radiation of symptoms is important to note, because patterns such as radiation of symptoms into the forearm with flexor-pronator tendinitis are common, as are paresthesias into the ring and little fingers with ulnar neuritis. Handedness is important only because symptoms usually manifest in the dominant extremity. Past history of injuries such as shoulder, back, or knee injuries that can easily alter the biomechanics of throwing may place the elbow at increased risk for overuse injuries. A general health assessment is also important.

What are the mechanics for the crank test

Liu et al.11 described the crank test in 1996: "The crank test is performed with the patient in the upright position with the arm elevated to 160° in the scapular plane. Joint load is applied along the axis of the humerus with one hand while the other performs humeral rotation. A posi- tive test is determined either by 1) pain during the ma- neuver (usually during external rotation) with or without a click or 2) reproduction of the symptoms, usually pain or catching felt by the patient during athletic or work activ- ities. This test should be repeated in the supine position, where the patient is more relaxed. Frequently, a positive crank test in the upright position will also be positive in the supine position. "Tricks in performing this test are to make sure eleva- tion is kept as extreme as possible (not at 90° for the apprehension or relocation test), and axial load is applied followed by stress relocation." Figure 5 is a depiction of the crank test. This description was produced after a study of 62 pa- tients in whom the test was positive in 31 and the diag- nosis was confirmed at arthroscopy. The sensitivity was 91% and the specificity was 93%. The positive predictive value was 94% and the negative predictive value was 90%.

What are the indication for medial humeral fracture?

Medial condyle fracture Salter-Harris type IV injuries with 2 mm or more of displacement usually require ORIF. A displaced medial condyle fragment or instability of the fragment with closed reduction is an indication for open reduction with rigid internal fixation. Accurate apposition of the fracture surfaces is important to reduce the risk of growth plate disturbance and to prevent loss of motion due to articular incongruence. Nondisplaced fractures can be treated without surgery. Medial epicondyle fracture For nondisplaced or minimally displaced fractures, nonoperative management is the procedure of choice (see images below). More controversy exists with displacement of 5-15 mm. Similar functional results have been reported with operative and nonoperative surgical management. Long-term functional assessment has demonstrated similar results even with radiographic nonunion being apparent on most of the fractures treated nonoperatively. Irreducible incarceration of the medial epicondyle fragment and open fracture are indications for operative management. Excision of the comminuted medial epicondyle fragment has been associated with less optimal results.[24] Other controversial relative surgical indications include complete ulnar nerve dysfunction after an injury or reduction attempt and valgus instability in high-demand athletes

What is the physical examination of UCL injury?

Medial elbow tenderness and swelling are the most notable findings. Tenderness is commonly found approximately 2 cm distal to the medial epicondyle. UCL tenderness may occasionally be difficult to differentiate from flexor pronator tendinitis, but the pain of flexor pronator tendinitis is aggravated by resisting forearm pronation. Loss of elbow range of motion (ROM) is occasionally observed. With acute rupture, ecchymosis may be observed over the medial elbow. Pain may be reproduced when the patient makes a clenched fist. Valgus stress with the elbow in 25° of flexion (elbow abduction stress test) reproduces pain and may cause joint opening. The affected side should be compared with the contralateral elbow as a reference for baseline laxity. Some throwing athletes have a baseline asymmetry;; therefore, preseason documentation of baseline elbow laxity in elite athletes, especially pitchers, may be helpful for comparison if an injury occurs during the season.

What is the medical care for ulnar neuropathy?

Medical and other nonsurgical treatments can provide significant help in cases of ulnar neuropathy. Vasculitic and metabolic causes can be evaluated and diagnosed to facilitate treatment of the underlying condition. The physician can address pain or other sensory symptoms using trials of various classes of pain medications including: Nonsteroidal anti-inflammatory drugs (NSAIDs), many classes Tricyclic (and related) antidepressants Anticonvulsants Narcotics (generally considered to be a last resort) Occupational therapy and work hardening programs are also beneficial. Therapists may use and design splints to restrict the range of joint motion and cushions to ameliorate the effects of pressure.[64] Use of a night splint is a common occupational or physical therapy technique that aims to limit the flexion and extension of the elbow at night. This has shown some efficacy in clinical trials.[65] Therapists also use nerve gliding, sliding, or tensioning exercises which seek to promote smoother movements of the nerve within the cubital tunnel and to reduce adhesions and other causes of physical nerve compression.[66] A randomized, controlled study of conservative methods to treat mild and moderate ulnar neuropathy at the elbow indicated that simply giving patients information about how to avoid injuring the ulnar nerve by avoiding or reducing movements or positions that compromise the nerve produced significant symptomatic improvement. Interestingly, adding splinting or nerve-gliding treatments to the program of providing information did not add a significant additional benefit.

What is the pathophysiology for coronoid fractures?

Morrey and coworkers, in their biomechanical study of the elbow, concluded that approximately 50% of the elbow stability comes from the congruent articulation between the trochlea and the ulna.[11] Closkey et al studied the stabilizing function of the coronoid process under axial load to the elbow.[12] They found no significant difference, at any flexion position, in posterior axial displacement between intact elbows and elbows in which 50% or less of the coronoid process was fractured (types 1 and 2) (P = .43). Differences in posterior axial displacement were significant, across all flexion positions, between intact elbows and elbows in which more than 50% of the coronoid process was fractured (type 3) (P = .006). They concluded that in response to axial load, elbows with a fracture involving more than 50% of the coronoid process displace more readily than elbows with a fracture involving 50% or less of the coronoid process, especially when the elbow is flexed 60o and beyond.

What is the presentation of olecranon fracture?

Most olecranon fractures are isolated. However, additional injuries to the same extremity are possible. Careful examination, including that of the shoulder, clavicle, humerus, wrist, hand, and forearm, is essential. Typically, the elbow incurs both soft tissue injury and joint effusion. Examine the skin, radial and ulnar pulses, and function of the ulnar, median, and posterior interosseous nerves. Carefully assess isolated injuries, as fracture of the coronoid process of the radial head and Monteggia fracture dislocations have a significant impact on elbow stability. When a supracondylar humerus fracture occurs in conjunction with an olecranon fracture, exposure of the humerus can be obtained by using the olecranon fracture site. Similarly, when an associated coronoid and/or radial head fracture exists, reduction and fixation can be achieved via a direct posterior approach through the displaced olecranon fragment. Although olecranon fractures generally are isolated injuries, a high index of suspicion for associated injuries is warranted in the evaluation of patients with multiple trauma. Twenty percent of patients with high-energy trauma have associated injuries (eg, long bone fracture, skull fracture, splenic injury, pulmonary contusion, axillary artery rupture). A transverse or slightly oblique break near the base of the olecranon is the usual fracture. In oblique fractures, the fracture line tends to slope down and back and emerges on the posterior border of the olecranon. In other instances, a small piece of bone is pulled off of the proximal end of the olecranon.

What is the presentation for proximal humeral fracture?

Most patients with fractures of the proximal humerus present to an acute care facility with pain following trauma. Pain and loss of function with swelling of the involved extremity are the most common symptoms on initial presentation. Document symptoms of paresthesias or weakness in the involved extremity. Obtain a detailed history of the mechanism of injury (eg, whether the injury was the result of a direct impact to the lateral shoulder or the result of an indirect mechanism, as in a fall onto an outstretched hand). Indirect causes of proximal humerus fractures result in greater degrees of fracture displacement. Determine whether seizure or electrical shock was involved, as these indirect mechanisms are associated with posterior dislocations. Obtain the medical history, and stabilize any problems, if possible, prior to proceeding with operative management. Physical examination Swelling and ecchymoses usually are present about the shoulder and upper arm. Extensive ecchymosis may become visible 24-48 hours following injury. It may spread to the chest wall and flank, and may involve the entire extremity. Palpate the entire upper extremity and chest wall to evaluate for associated injuries. To determine fracture stability, gently rotate the humeral shaft while palpating the humeral head to assess whether unified motion is present. Note any movement or crepitus. In high-energy injuries, inspect the skin closely for any disruptions that may allow fracture contamination (ie, open wounds). Pulsatile or expanding hematomas may indicate a vascular lesion. It is essential to determine the presence of any associated neurovascular injury. The axillary nerve is the nerve most commonly injured in proximal humerus fracture. Carefully assess sensation over the deltoid muscle and isometric deltoid motor function. Additionally, perform distal neurological testing for brachial plexus injuries. Examination of peripheral pulses is helpful, but does not exclude axillary disruption, because distal pulses may be intact due to collateral circulation around the scapula. Inspect the proximal shoulder girdle for an expanding mass, which may be the only sign of arterial rupture. If vascular injury is suspected, obtain an angiogram and vascular surgery consultation immediately. Evaluate associated injuries (eg, pneumothorax, other traumatized areas) with radiographic studies. Radiographic examination of the shoulder should include Neer's trauma series, which consists of a true anteroposterior (AP) view of the glenohumeral joint, y-view, and axillary view. Modifications of the axillary view, such as a Velpeau view or CT scan, can be obtained to evaluate the relationship of the humeral head to the glenoid. It is estimated that the initial treating physician nevertheless misses 50% of all fracture dislocations.

What is the physical therapy management of medial epicondylitis?

Nonsurgical treatment The main goal of the conservative treatment is to obtain pain relieve and an inflammation reduce. These two things will help to achieve a proper rehabilitation and later a return to activities. Nonsurgical treatment can be divided into three phases. - Phase 1: The patient immediately has to stop the offending activities. It's not recommended to stop all activities or sports since that can cause atrophy of the muscles. The therapy starts with 'PRICEMM', which stands for 'prevention/protection, rest, ice, compression, elevation, modalities and medication'. The affected elbow should be iced several times a day for about a quarter. This improves the local vasoconstrictive and analgesic effects. As for medication the patient can take nonsteroidal anti-inflammatory medication (NSAID). If the patient's condition doesn't improve, a period of night splinting is adequate. This is usually accompanied www.physio‐pedia.com/index.php/Medial_Epicondylitis 5/7 9/3/12 Medial Epicondylitis ‐ Physiopedia, universal access to physiotherapy knowledge. with a local corticosteroid injection around the origin of the wrist flexor group. Some examples of a physical therapy modality are ultrasound and high-voltage galvanic stimulation (but there's not yet a study that notes their efficacy). Counterforce bracing is recommended for athletes with symptoms of medial epicondylitis. It can also aid when the patient is returning to sport. - Phase 2: As soon as we see an improvement of phase 1, a well guided rehabilitation can be started. The first goal of the second phase is to establish full, painless, wrist and elbow range of motion. This is soon followed by stretching and progressive isometric exercises. These exercises first should be done with a flexed elbow to minimize the pain. As soon as the patient has made some progress the flexion of the elbow can be decreased. As the flexibility and the strength of the elbow area return, concentric and eccentric resistive exercises are added to the rehabilitation program. The final part of this phase is a simulation of sport or occupation of the patient. - Phase 3: When the patient is able to return to his sport it is necessary to take a look at his equipment and/or technique. These precautions ought to be taken to allow a safe return to activities (level of quality D)[8]. Postoperative management 7 to 10 days after the operation, the splint and skin sutures are removed. At this point the physical therapy can start. The beginning of the treatment is characterized by gentle passive and active hand, wrist and elbow exercises. 3 to 4 weeks later gentle isometrics can be done and at 6 weeks the patient can start with more resistive exercises. At last a progressive strengthening program has to be followed. In normal cases the patient can return to activities 3 to 6 months after the operation (level of quality D)[8].

What are the images for SLAP lesion?

On plain radiography of the shoulder, an anteroposterior view of the shoulder in internal and external rotation, outlet, and axillary views should be obtained.[29] Findings are usually normal. Occasionally, a SLAP (superior labrum, anterior and posterior) fracture, which represents a superior humeral head compression fracture, can be observed. Plain radiographs should be carefully reviewed for other potential pathology, such as an os acromiale, an anterior acromial spur, or a degenerative AC joint. Nonenhanced MRI has proven to be unreliable in determining the presence of SLAP tears.[30] It is useful to evaluate potential concomitant pathology, such as partial thickness or full thickness rotator cuff tears. It is also valuable in detecting the presence of a paralabral cyst. Ganglion cysts encroaching on the spinoglenoid notch are associated with superior, usually posterior, labral tears. The use of contrast medium as in magnetic resonance arthrography offers improved visualization of intra-articular structures and is thought to improve the ability to accurately detect SLAP tears; however, reported results continue to be highly variable.[31, 32, 33, 34] Two useful signs on MRI are those of increased signal intensity in the posterior third of the superior labrum and a laterally curved intensity. The sublabral recess does not usually extend to the posterior third of the superior labrum, and therefore, high signal intensity between the labrum and the glenoid in this region is considered to be consistent with a superior labral tear. Another MRI finding considered to be highly suggestive of a superior labral tear is laterally curved signal intensity. On the contrary, a normal sublabral recess results in a medially curved area of signal intensity. The findings of a retrospective review study conclude that while multidetector computed tomographic arthrography showed limitations in the overall percentage of correct classification, it showed high accuracy and interobserver reliability in the diagnosis of SLAP lesions.[35]

What is the surgical intervention for Monteggia fracture?

Open fractures require emergent surgical consultation. The initial treating physician may reduce the radial head dislocation and splint this fracture. Otherwise, an orthopedic surgeon should be consulted immediately to reduce the radial head. Anatomic reduction of the ulna is usually required prior to radial head reduction. Unless the fracture is open, surgical treatment is performed on an elective basis. While most adults require operative treatment, most pediatric fractures are treated closed. Operative fixation of complete fractures of the ulna with proximal radioulnar joint dislocation is recommended in children. The complete disruption of bone continuity is likely to be associated with substantial soft-tissue trauma in these injuries. Shortening and angulation of complete fractures after cast immobilization is not uncommon. Anatomic reduction of the ulnar fracture and radial head often requires operative treatment. In the past, transverse and short oblique fractures were adequately treated with intramedullary wire fixation. Intramedullary wires, however, cannot be relied on to maintain reduction of complete fractures that are either long oblique in pattern or comminuted; the wires therefore are not used anymore. These fractures are likely to displace or even shorten and, consequently, should be fixed with a plate and screws. As a result of the rapidity of osseous repair and the tolerance of cast immobilization in children, the use of plate-and-screw constructs that are smaller (typically a one-third tubular or semitubular plate) and shorter (2 or 3 holes [4 or 6 cortices] proximal and distal to the fracture) than those recommended for adults are usually adequate.

What are the postoperative for olecranon fracture?

Operative management of olecranon fractures should provide sufficient fixation for immediate motion. Typically, patients are immobilized for only a brief time to assist wound healing and are then started on range-of-motion exercises at 10 days. However, muscle strengthening is not emphasized until bone healing is visualized radiographically. Patients may return to work involving vigorous use of the extremity at 3-4 months postoperatively.

What is the medial therapy for Monteggia fracture?

Pain should be managed as needed in the immediate period. If the fracture is open, the status of the patient's tetanus immunization should be determined and addressed as indicated. Intravenous antibiotics should be administered to patients with open fractures. Open wounds should be irrigated with sterile saline solution and dressed with sterile, moist gauze. The radial head should be reduced in the emergency department if possible. Pediatric patients should undergo closed reduction and splint application emergently. Closed reduction in children is easiest when performed under procedural sedation or general anesthesia. Ketamine 1-2 mg/kg IV or 3-4 mg/kg IM is a very useful drug for sedation. An image intensifier should be available with real-time and static images to verify anatomic reduction of the fracture and congruent relationship of the radiohumeroulnar joint. The position of the elbow when immobilized depends on the fracture pattern as described earlier. Nonoperative treatment is successful for most Monteggia injuries in children because (1) the majority of the fractures are inherently stable, (2) they require a shorter time for both the osseous and the ligamentous injuries to heal, (3) children have little trouble regaining motion lost through stiffness, despite immobilization of the fractures for the duration of the initial healing period (3-6 wk), and (4) the potential may exist for remodeling of mild, residual angular deformities (< 10°).

What is the examination for posterior elbow dislocation?

Physical therapy examination should include a vascular and neuromuscular screen, observation, palpation, muscle testing, ROM, and special ligamentous tests. The following information outlines ways to test for potential impairments after PED. A vascular assessment should include palpation of the brachial, radial, and ulnar arteries. During the neuromuscular screen, dermatomes, myotomes, and reflexes should be evaluated with emphasis on the ulnar, median, and radial nerves. Observe the elbow for any ecchymosis, rubor, or deformities. During palpation, a disrupted triangle sign may indicate joint dislocation. The triangle sign is obtained by palpating the tip of the olecranon, medial, and lateral epicondyles while in elbow flexion, resulting in a triangle configuration.[9] Texas State physical therapy student applies supination with a valgus stress and axial compression while taking the patient from extension into flexion. The elbow will sublux at 40 degrees of flexion if instability exists. Further flexion will reduce the joint with an audible and palpable clunk. It is essential to palpate for associated fractures in the elbow complex. The elbow extension sign can be used to rule out a fracture. Specific muscles that attach to the elbow should be evaluated including the elbow and wrist flexors/extensors and supinators/pronators. Documentation of elbow ROM is necessary when following the progression of rehabilitation. Other outcome measures include the Mayo Elbow Performance Index (MEPI) and the Disabilities of the Arm, Shoulder, and Hand (DASH).[4] Patients with PED may also have concomitant collateral ligament ruptures at the elbow. If this occurs, the patient will be at greater risk of developing recurrent instability.[3][7] To assess for this, the following special tests should be performed: varus and valgus stress test, the lateral pivot-shift test (Posterolateral Rotational Instability Test) (see Image 3), and apprehension testing. Physical therapists should be alert for the following potential complications associated with PED: neurological deficits including hypoaesthesia of the hand in the ulnar nerve distribution, concomitant fractures, myositis ossificans, and degenerative changes in the joint.[3][6][9][11][13] Radiographs are indicated when there is no response to care after four weeks of conservative treatment, significant activity restriction for more than four weeks, or non-mechanical pain is present.[14] As with all patients, clinicians should be aware of red flags listed in Table 3 below.

What is the post-op for anterior shoulder instability?

Postoperatively, the shoulder is kept in an arm sling for 3-4 weeks. The shoulder may be internally rotated and flexed for hygiene maintenance, but external rotation is prohibited. After 4 weeks, full active flexion is encouraged and assisted with physical therapy. At 6 weeks, internal and external rotation of the shoulder is begun using 5- and 10-pound weights. At 3 months, full and unrestricted weightlifting is allowed. Competitive throwers should be restricted from unrestricted athletics for 6 months. At 3 months, the patient should have regained 70% of external rotation and elevation of the shoulder. At 6 months, 75-100% of normal motion and strength may be observed.

What are the imaging studies for a UCL injury?

Radiography Findings from routine radiography can occasionally be diagnostic if an avulsion fragment is seen, and in a minority of patients, this study can also reveal secondary findings that are suggestive of UCL injury, such as ossification of the ligament. Plain radiographs are also helpful to rule out other causes of elbow pain, such as epitrochlear osteophytes, epicondylar fractures, posterior olecranon fossa loose bodies, ligamentous calcification, or capitellar lesions.[6] Manual or instrumented valgus stress radiography can be used to document increased joint opening and ligamentous laxity. Significant asymmetry may be observed in traumatic elbow injuries such as dislocations, whereas laxity in a throwing athlete may not be so obvious, with only a very subtle asymmetry. Gravity stress radiography—with the patient supine, the shoulder in maximal external rotation, and the weight of the forearm resisted by the UCL—may also be helpful.[7] Plain arthrography: This imaging modality is not indicated because dye leak has been shown to be inconsistent in cases of chronic laxity, and only an acute event may be anticipated to exhibit a positive finding. Magnetic resonance imaging (MRI): Plain MRI is a useful study;; however, because of the relatively small size of the UCL, the overall sensitivity of MRI is 57-79%[8] MR arthrography: This is the most useful imaging modality, with a sensitivity of 97% for UCL injury, and can provide detailed definition of the UCL and associated injuries.[9] Ultrasonography: This modality allows for rapid evaluation of the UCL. A ruptured UCL on an ultrasound appears as a discontinuity of the ligament with fluid in the gap between ends or as nonvisualization of the ligament. Sprains appear as thickening, decreased echogenicity of the ligament, and/or edema when compared with the normal ligament.[10] Computed tomography scanning (CT) with intra-articular contrast: This technique has been studied in small numbers of patients. CT scanning with intra-articular contrast appears to be highly sensitive and specific for both acute and chronic injuries,[8] but more data are needed before widespread use can be recommended

What is the imaging for little league elbow syndrome?

Radiography can be very helpful to the physician when evaluating an injured elbow. Radiographs can help the physician to rule out medial or lateral epicondyle avulsions, loose bodies, or DJD. Myositis ossificans of the brachialis muscle can be seen on radiographs, which often mimics anterior capsule strain. Calcification of the tendons can be found in chronic cases of tendinosis. Occasionally, olecranon stress fractures can show a translucent line on regular radiographs. This finding is rarely visible during the period of the first 2-3 weeks when the athlete experiences symptoms. Olecranon osteophytes or loose bodies in the fossa can be seen in posterior impingement syndrome. Radiocapitellar chondromalacia can appear on plain films as an irregular joint space, osteophytes, or loose bodies. Plain radiographs are of little help to the physician when diagnosing entrapment syndromes. Plain films may be of some help in excluding the differential diagnosis in patients who fail to respond to physical therapy (see Differentials and Other Problems to Be Considered). Triple-phase bone scans can be very useful in helping clinicians to diagnose olecranon stress fractures. Bone scans can show increased radionuclide uptake at the capitellum and/or radial head when an osteochondral lesion that is associated with chondromalacia of the radiocapitellar joint is present. Magnetic resonance imaging (MRI) is very good at delineating soft-tissue injuries. This imaging modality is also very helpful to the physician in the evaluation of chondral defects and loose bodies about the elbow.[5] Many times, the site of nerve entrapment—with the resultant edema around the nerve—can be visualized on MRI, which can be very helpful for planning the surgical release of the nerve compression. Often, MRI can be used to evaluate stress fractures and the resultant bone edema at the fracture site. With MRI, the extent of tendon degeneration in a tendinosis can also be evaluated, as well as emedicine.medscape.com/article/96638-‐‑workup#showall 1/4 9/3/12 Elbow and Forearm Overuse Injuries Workup ligamentous injuries, which can help in the treatment of a posterolateral rotatory instability. MRI is very good at delineating the extent of the articular erosion that is present in cases of radiocapitellar chondromalacia. Angiograms can be performed to rule out vascular causes for nerve pain in recalcitrant cases of nerve entrapment.

What are the imaging studies for shoulder dislocation?

Radiography: Conventional radiography should be performed in all patients with suspected shoulder dislocations to confirm the diagnosis and also to exclude associated fractures prior to any attempted reduction.[12] Routine radiographs should include at least an anteroposterior (AP) view and an axillary view.[13] The AP view can be obtained in neutral, internal, or external rotation. In internal rotation, one can easily see a Hill-Sachs lesion of the posterolateral humeral head. The axillary view nicely shows glenohumeral subluxation or dislocation, as well as anterior or posterior glenoid rim fractures.[14] A standard axillary view may be difficult in the acute injury setting because it requires 90° of abduction. However, many modifications exist to avoid excessive movement of the painful extremity. For example, the transverse axillary lateral requires the patient to abduct the arm only 10-30°. Other views that may be useful include the scapular Y view, which is helpful for diagnosing dislocations and scapular fractures. This view, however, should not replace the axillary view because it does not show subtle subluxations of the glenohumeral joint or fractures of the glenoid rim. The true AP or Grashey view is helpful in assessing subtle joint incongruity, superior or inferior subluxation, degenerative changes, or glenoid hypoplasia. The Garth or West Point view is useful in detecting bony Bankart fractures of the anteroinferior glenoid rim as well as Hill-Sachs defects. This view is advantageous in the acute setting because it does not necessitate the patient moving the arm. The Stryker notch view can also be useful in detecting Hill-Sachs lesions. However, this view is of limited usefulness in detecting subluxations or glenoid fractures. Computed tomography (CT) arthrography, magnetic resonance imaging (MRI), and/or magnetic resonance arthrography may be helpful in assessing some shoulder dislocations.[15] CT arthrography was commonly used in the past to evaluate patients with glenohumeral instability either after the initial dislocation or with recurrent instability. However, today, it is used only when an MRI is contraindicated or if glenoid version abnormalities are suspected. MRI and magnetic resonance arthrography have been shown to be more sensitive than other methods in detecting labral and ligamentous pathology, rotator cuff and cartilage tears, capsular abnormalities, and biceps injuries. MR arthrography is more sensitive than MRI alone and is the study of choice after a shoulder dislocation, particularly in cases of recurrent instability, and it is superior to MRI for diagnosing the pathologic lesions mentioned above.

What is the background info for dislocation?

Shoulder dislocations account for almost 50% of all joint dislocations. Most commonly, these dislocations are anterior (90-98%) and occur because of trauma. Most anterior dislocations are subcoracoid in location. Subglenoid, subclavicular, and, very rarely, intrathoracic or retroperitoneal dislocations may occur. Recent studies According to Scheibel et al, immobilization of the shoulder in 30º of external rotation seems to allow a similar coaptation of the glenoid labrum regardless of whether immobilization is for 3 weeks or 5 weeks. The authors divided 22 patients into 2 groups: 11 patients immobilized for 3 weeks and 11 patients immobilized for 5 weeks in 30º of external rotation. No statistically significant differences were found after acute, 3-week, and 5-week magnetic resonance imaging examinations.[1] In a study by Owens et al, acute arthroscopic Bankart repair in young, active patients with first-time traumatic anterior glenohumeral dislocations resulted in excellent subjective function and return to athletics, with an acceptable rate of recurrence and reoperation. Of 39 patients followed (40 shoulders), 6 patients sustained recurrent dislocations, 9 patients had subluxation events, and 6 patients underwent revision stabilization surgery.[2] Another study, also by Owens et al, reviewed data from the American Board of Orthopaedic Surgery (ABOS) and noted that the use of open repair has declined in recent years, with a trend toward arthroscopic Bankart repair. The study found the most commonly reported complications were nerve palsy/injury and dislocation; rate of nerve injury was 2.2% in the open group, compared with 0.3% in the arthroscopic group. The dislocation rate was 1.2% with open stabilization, compared with 0.4% arthroscopically.[3] Maier et al compared the clinical benefit of operative stabilization in younger patients (49 patients < 40 y) and older patients (23 patients > 40 y) after anterior shoulder dislocation and found that there was significant reduction in recurrence in both groups. However, the clinical functional results measured by the Constant score, Rowe score, and disabilities of the arm, shoulder, and hand (DASH) score revealed significantly better outcomes in the younger group.[4] In a study of Cordischi et al of skeletally immature patients (14 patients aged 10.9-13.1 y) who sustained a primary traumatic unidirectional anterior shoulder dislocation, those patients who were treated nonoperatively fared better than those treated by surgery (average Western Ontario Shoulder Instability index [WOSI] score of 9.1 vs 151.7, respectively). According to the authors, in the pediatric skeletally immature patient, nonoperative treatment results in low shoulder instability recurrence risk and sound functional outcome

What is the background for proximal humeral fracture?

Successful treatment of fractures of the proximal humerus (ie, that portion involving the glenohumeral articulation) presents a challenge for physicians. Many factors must be considered when developing a treatment plan. Accurate assessment of the fracture, patient compliance, medical comorbidities, and time from injury to treatment are critical factors affecting outcome. Additionally, technical factors in the reconstruction of these fractures require surgical experience that few surgeons have the opportunity to develop. Recent studies Kontakis et al studied the outcomes of 28 patients in whom the Aequalis fracture prosthesis was used for acute fracture of the proximal humerus and found that anatomic reconstruction was associated with a higher mean Constant score, as well as higher mean values of anterior forward elevation, abduction, and external rotation. In 18 patients, active anterior elevation was 150º or greater, and mean active abduction and external rotation were 163.6º and 31.3º, respectively. In 7 of the 28 patients, mean active anterior elevation, abduction, and external rotation were 130.7º, 129.2º, and 22.8º, respectively. In all, 12 patients were very satisfied with the results, 12 were satisfied, 2 were dissatisfied, and 2 were disappointed. In addition, 26 reported no or only mild pain, and 2 had moderate pain. Proximal migration of the humeral head occurred in 5 patients; there was no evidence of loosening in any of the patients.[1] Brunner et al evaluated the incidence of complications and the functional outcome one year after open reduction and internal fixation with a proximal humeral locking plate in 157 patients from 2002 to 2005. The incidence of implant-related complications was 9%; incidence was 35% for non-implant-related complications. Primary screw perforation was the most frequent problem, at 14%, followed by secondary screw perforation (8%) and avascular necrosis (8%). After 1 year, the mean Constant score was 72 points; the mean Neer score was 76 points; and the mean Disabilities of the Arm, Shoulder, and Hand score (DASH) was 16 points. The authors suggested that more accurate length measurement and shorter screw selection should prevent primary screw perforation and that awareness of anatomic reduction of the tubercles and restoring the medial support should reduce the incidence of secondary screw perforations, even in osteopenic bone.[2] Bahrs et al assessed the Constant score and radiographic outcome in 66 patients with minimally displaced and/or impacted fractures of the proximal humerus treated with early immobilization. All of the fractures healed well, without nonunion. In 80% of patients, radiologic assessment showed fracture-displacement of less than 15º angulation and/or less than 5-mm displacement of the greater tuberosity. There was a significant association between the final Constant score and age, ASA classification, AO classification, and initial fracture displacement. The authors concluded that early physiotherapy with a short period of immobilization is sufficient management for minimally displaced and/or impacted fractures of the proximal humerus.[3]

What is the problem of a SLAP lesion?

Superior labral (labrum) lesions can cause painful mechanical symptoms and difficulty with overhead activities whether they be athletic or those of daily living. SLAP (superior labrum, anterior and posterior) lesions (see image below), as opposed to occult anterior instability, are likely the underlying cause of the so-called dead arm syndrome in throwing athletes. Regardless of whether injuries to the superior labrum biceps complex are secondary to a throwing or nonthrowing etiology, they can be a source of considerable disability for the patient. These injuries are difficult to diagnose on physical examination because the findings are often nonspecific and demonstrate considerable overlap with those of other etiologies of shoulder pain. Similarly, both nonenhanced MRI and magnetic resonance arthrography have variable accuracy in determining if a SLAP lesion is present. At times, the only definitive way to diagnose a SLAP lesion is with a diagnostic arthroscopy. Even so, a thorough understanding of the normal anatomy and biomechanics of the superior labral biceps complex, as well as commonly observed normal anatomic variants, is mandatory to ensure appropriate surgical stabilization and to avoid unnecessary repair of the superior labrum.[4, 5] Snyder classified superior labral tears into 4 types.[3] A type I lesion is characterized by significant fraying of the labrum, but the biceps anchor is intact. A type II lesion is a tear of the superior labrum that results in instability of the biceps anchor. Significant fraying of the labrum occurs as is observed in type I tears; in addition, the superior labrum and associated biceps anchor is stripped away from the superior glenoid. A type III lesion describes a bucket-handle tear of the superior labrum. The central portion of the superior labrum is torn and usually displaced into the joint. The peripheral attachment of the labrum is intact, and the biceps anchor is usually stable. A bucket-handle tear of the superior labrum also characterizes a type IV lesion; however, the tear also propagates to a variable degree into the biceps tendon. Modifications have been made to the original classification. Morgan and Burkhart subdivided the type II lesions into anterior type II, posterior type II, and combined type II, referring to the tear involving the labrum both anterior and posterior to the biceps anchor Maffet et al expanded the classification to include types V, VI, and VII.[7] A type V SLAP refers to a Bankart lesion that extends superiorly to include the superior labrum and biceps anchor. A type VI lesion has a flap tear of either the anterior or posterior superior labrum with an associated type II tear. A type VII lesion describes a tear of the superior labrum that includes the middle glenohumeral ligament. Type II lesions are by far the most common and are also the source of the greatest diagnostic difficulty.

What is the surgical treatment for shoulder dislocation?

Surgery may be indicated if patients are unable or unwilling to change their occupation or avoid participating high-risk sports and they have recurrent dislocations or subluxations. The question of timing of surgery remains unclear. Several studies have advocated arthroscopic or open stabilization procedures after the initial dislocation in lieu of the traditional method of surgical intervention after a trial of nonoperative treatment in patients with a history of multiple dislocations or subluxations.[30] In military recruits and in young athletes aged 17-27 years, studies have shown far superior results with surgery after the initial dislocation in these patients, as opposed to the results after a trial of nonoperative treatment.[31, 32, 33] In a prospective trial, the repeat dislocation rate was 4% after arthroscopic stabilization of acute dislocations and 94.5% after nonoperative treatment.[34] In another study, patients were randomly assigned to immobilization and early surgical intervention. In these patients, the repeat dislocation rate was 15.9% at 2 years, and the recurrence rate was 47% in patients treated nonoperatively. Studies by Arciero et al,[35] Hintermann et al,[36] and Patel and Leith[32] should also be reviewed. Historically, open stabilization procedures have had a rate of repeat dislocation rate slightly lower than that of arthroscopic procedures, but the discrepancy is significantly less today, as technical skills and anchoring devices have improved. In a study in Sweden, the arthroscopic failure rate was 15%, compared with an open stabilization rate of 10%.[37] External rotation was better maintained in the arthroscopic group, in whom it was 90°, compared with the group that underwent open procedures, in whom it was 80°. Another study of arthroscopic and open reconstruction revealed failure rates of 33% and 8%, respectively.[28] However, many authors believe that their arthroscopic results are the same, if not better, than their results with open procedures for both athletes in contact sports and athletes in noncontact sports. The trend is toward minimally invasive surgery, and the results of arthroscopic instability repairs will continue to improve. A key element in a successful instability procedure is addressing any capsular laxity, whether by means of an open capsular shift, an arthroscopic capsular plication, thermal capsulorrhaphy, or rotator interval closure.

What ist he surgical consideration for medial collateral ligament reconstruction?

Surgical Considerations: For medial collateral ligament injuries, reconstruction occasionally is required to restore use of the arm in overhead activities and in sports. Repair of the MCL is performed secondary to an avulsion from the humeral attachment. The anterior bundle of the MCL is usually the focus of the reconstruction. Most commonly, the ipsilateral palmaris longus is used for the graft. The gracilis, semitendinosus or the plantaris tendon are other options for the graft.

What is the surgical consideration for ulnar nerve transposition?

Surgical Considerations: Prior to surgery, one or more medical examinations may be performed. An NVC study may be performed to assess status of the nerve. The loss of evoked sensory potential is a sensitive indicator of altered conduction. d Nerve conduction velocity less than 41 m/s or conduction latency across the elbow greater than 9 or 10 millseconds is consistent with denervation potentials of the ulnar-innervated muscles. Examination of concomitant elbow pathology such as UCL laxity should be performed and treated accordingly. Roentgenographic studies may be done to determine the degree of cubitus valgus or evaluate bony lesions compromising the cubital tunnel. http://www.cumc.columbia.edu/dept/peripheral- nerve/problems/un.html

what are the surgical management of medial epicondylitis?

Surgical treatment of medial epicondylitis When conservative management fails and there is persistent pain after 6 to 12 months and all other pathologies are considered, surgical treatment must be considered. Surgery for failure of conservative treatment relieves pain, restored strength and allows a return to the previous level of daily living and sports activity (level of quality C)[9]. Mini-open muscle resection procedure under local anesthesia For medial epicondylitis the degenerative tissue at the origine of the flexor carpi radialis brevis is removed during a mini-open muscle resection procedure. This procedure produces low levels of postoperative pain, a short hospital stay and rehabilitation period and early return to daily activities. The limitations of and open flexor carpi radialis brevis release include late return to work and sporting activities due to a prolongation of the postoperative recovery time, a risk of posterolateral instability, and the formation of nueroma after surgery (level of quality B)[10]. Steroïd injections The indication for injection therapy for epicondylitis is usually chronic pain and disability not relieved by more conservative means, or severe acute pain with functional impairment that calls for a more rapid intervention. These injections seem to have a short term effect ( 2-6 weeks) and effective in providing early symptom relief (level of quality A1)[7]. Autologous blood injection The combined treatment of dry needling and ultrasound guided autologous blood injection is discribed as a effective way to treat patients with refractory lateral and medial epicondylitis. The hypothesis of the mechanism is that the transforming growth factor-β and basic fibroblast growth factor carried in the blood act as humoral mediators to induce the healing cascade (level of quality C)[11].

What are the complication for olecranon fracture?

Symptomatic hardware requiring removal is the most frequent complication following internal fixation. Hardware problems have occurred in up to 80% of patients with Kirschner tension band wires. Wire migration with soft tissue irritation, wire breakage, or fracture displacement may occur with tension-band wiring. Counsel patients about the possibility of symptomatic hardware when internal fixation is offered. Hardware complications generally occur less frequently with intramedullary screw fixation. Plate and screw fixation carries a moderate risk of subsequent need for hardware removal. Loss of motion is a common problem following fractures of the elbow but is usually not a significant issue for olecranon fractures. Generally, patients lose 15° of extension and, occasionally, a small amount of supination. Motion tends to improve progressively with time for up to 2 years. Heterotopic ossification occurs in 13-14% of patients. The range of reported rates of infection following operative treatment is 0-6%. Reflex sympathetic dystrophy occurs on rare occasions. Generally, nonunion occurs in fewer than 5% of patients. When nonunions are treated by internal fixation and bone grafting, good to excellent results occur in approximately two thirds of cases.

What is the pathology/MOI of TOS?

TOS is characterized by pain and weakness from compression of nerves. Pressure on the nerves or blood vessel bundle creates symptoms. Pressure may be caused by an extra rib in the lower back, over developed neck muscles. muscle weakness, and drooping in the shoulder. It can also be caused by an injury that results overextending the arm.

What is the problem with lateral humeral condyle fracture?

The distal humerus is primarily cartilage at the age when these injuries typically occur, and knowledge of the secondary centers of ossification is necessary to understand the possible fracture patterns. Due to incomplete ossification, the fracture may appear subtle on radiographs as it courses through the cartilage anlage, as depicted in the images below. The physis of the lateral condyle extends into the trochlear notch of the distal humerus (see image below). Therefore, in some fractures, the lateral crista of the trochlea may be part of the fracture fragment, leading to an unstable humeral ulnar articulation. The difficulties related to treatment of this fracture are both biologic and technical. Biologic problems are a result of the healing process and may occur with appropriate treatment and anatomic reduction. These problems include lateral spur formation with pseudo cubitus varus and true cubitus varus. Technical difficulties are the result of errors in management and may result in nonunion, malunion, valgus angulation, avascular necrosis, or a combination of these conditions.

What is the problem with Monteggia fracture?

The first challenge is correctly assessing the extent and nature of the injury. The ulna fracture is usually noted, commonly in the proximal third of the ulna. The olecranon, midshaft, and distal shaft may be involved. In his classic 1943 text, Watson-Jones stated that "no fracture presents so many problems; no injury is beset with greater difficulty; no treatment is characterized by more general failure."[5] Some injuries associated with radiocapitellar dislocation (such as the transolecranon fracture-dislocation of the elbow) are mislabeled as Monteggia lesions, when in fact the proximal radioulnar joint remains intact. The Monteggia lesion is most precisely characterized as a forearm fracture in association with dislocation of the proximal radioulnar joint. The radial head dislocation may not be apparent and will possibly be missed if the elbow is not included in the radiograph. Whenever a fracture of a long bone is noted, the joints above and below should be evaluated using radiographs in orthogonal planes (planes at 90° angles to each other). If one of the forearm bones is injured, injury should be looked for in the other bone and in associated joints of the forearm, elbow, and wrist. This principle also applies to a Galeazzi fracture, which is a fracture of the distal radius with concomitant dislocation of the distal radioulnar joint. Separate radiographs should be taken of the elbow. The radial head should point towards the capitellum on all radiographs of the elbow. Unrecognized dislocations may result from reduction of the dislocated radius prior to presentation. This may occur in the field spontaneously or as a result of manipulation by emergency responders. The treating physician may reduce an unrecognized dislocation while reducing or immobilizing the ulna fracture. Problems relating to treatment are discussed in Complications.

What is the etiology of medial humeral fracture?

The following 3 possible mechanisms for medial condyle fracture[1, 2, 3, 5, 6, 7, 8, 9, 11, 13] have been described: A fall on the palm of an outstretched arm, with the elbow forced into valgus (see image below) A fall on the point of the elbow (apex of the flexed elbow), with the olecranon driving the medial condyle proximally and medially (see image below) An avulsion fracture due to violent contraction of the flexor and pronator muscles that attach to the medial epicondyle, such as that which occurs in arm wrestling (see image below) Three mechanisms of injury for medial epicondyle fractures[22, 23] have been proposed for an acute injury. All 3 mechanisms result in a partial or complete separation of the apophyseal fragment from the rest of the humerus. The 3 mechanisms are as follows: A direct blow on the posterior medial aspect of the epicondyle that may be associated with fragmentation of the avulsed bone Pure avulsion injury produced by the flexor muscles of the forearm (see image below) This avulsion may occur in combination with a valgus stress that locks the elbow in extension. The classic example is the child that falls on the extended arm and hyperextends the wrist and fingers, placing more stress on the forearm flexors. The normal valgus angulation or carrying angle of the elbow tends to accentuate the forces responsible for the avulsion injury. This mechanism can also explain associated injuries, including a radial neck fracture with valgus angulation and greenstick fractures of the olecranon. The second type of avulsion injury may be a pure muscular avulsion secondary to contraction of the forearm flexor musculature with an elbow flexed. This mechanism may be responsible for medial epicondyle fractures associated with arm wrestling and throwing a baseball. The final mechanism proposed for medial epicondyle fracture is associated with a dislocation of the elbow, as depicted in the image below. In this mechanism, the ulnar collateral ligament provides an avulsion force that causes the medial epicondyle to fail.

What is the background info for SLAP lesion?

The glenoid labrum is a triangular fibrocartilaginous structure that serves to deepen the glenoid. While tears of the anteroinferior labrum have long been known to be associated with significant shoulder pathology, injuries of the superior labrum have really only been appreciated as a potential pathologic lesion since the advent of shoulder arthroscopy. The superior labrum often has a more meniscoid attachment to the glenoid rim compared with the remainder of the labrum and therefore may be more susceptible to both degenerative and traumatic lesions. It also serves as part of the origin of the long head of the biceps. Injuries to the superior labral biceps complex can compromise the biceps anchor. Furthermore, the repetitive tensile force exerted by the biceps on the superior labrum likely contributes to poor healing of superior labral tears. Superior labrum tears were first described by Andrews.[2] In a study that reviewed their experience in 73 throwing athletes, the authors identified tears of the labrum involving the anterosuperior aspect near the origin of the biceps tendon. They attributed this lesion to the biceps tendon being pulled off the labrum as a result of force generated during the throwing motion. Snyder et al coined the term SLAP to denote a superior labrum, anterior and posterior lesion to describe a more extensive injury.[3] A SLAP lesion as described by Snyder involves a tear of the superior labrum, which starts posteriorly and extends anteriorly to include the anchor of the biceps tendon to the superior labrum. The injuries were subdivided into 4 types (I-IV). Similar to Andrew's findings, a subset of patients who reported a traction injury were identified. However, the majority of patients related a history of a compressive injury to the shoulder secondary to a fall on an outstretched arm in a flexed and abducted position.

What is the postoperative Details for SLAP?

The patient is kept in a sling for 3 weeks. Codman is initiated at week 2. Passive range-of-motion exercises, including elevation and external rotation by the side, are also initiated at week 2. No external rotation in abduction is allowed for the first 3 weeks because of the peel-back mechanism. Range-of-motion exercises, including passive posterior capsule and internal rotation stretching, are progressed during weeks 3-6. At 6 weeks, progressive strengthening of the rotator cuff, scapula stabilizers, biceps, and the deltoid is initiated. Throwing athletes are allowed to begin an interval throwing program at 4 months. At 6 months, throwing from a mound is allowed. Pitchers may resume throwing at full velocity at 7 months postoperatively. Throughout the rehabilitation, as well as after, the patient should continue stretching the posteroinferior capsule daily. A recurrence of the capsular contracture can once again put the shoulder at risk for developing a SLAP (superior labrum, anterior and posterior) lesion. For nonathletes, the initial 4 months of rehabilitation is identical to that of throwers. At the 4-month mark, they can usually resume full activities.

What is the presentation for medial humeral fracture?

The patient usually presents with a recent history of a significant fall on an outstretched hand or directly on the apex of the flexed elbow. The elbow may be severely painful following this injury. Swelling, deformity, and loss of function of the elbow may be present. Palpable crepitus may be present over the medial condyle. Elbow motion may be decreased due to swelling and pain. The patient often holds the elbow fixed at approximately 90° of flexion. The patient may present with medial dislocation of the forearm, referred to as a fracture dislocation. Distal neurovascular changes may occur, especially in the ulnar nerve distribution. Other injuries may be present that are easier to detect, such as elbow dislocation or fracture of the radial head or olecranon, which may distract the physician from making the diagnosis of medial condyle fracture. A high index of suspicion for this type of injury concurrent with other elbow injuries can ensure timely diagnosis and treatment.[1, 2, 3, 5, 6, 7, 8, 9, 11, 13] Medial epicondyle fracture The presentation of a patient with a medial epicondyle fracture does not differ significantly from that of a patient with a medial condyle fracture, as described above. A through physical examination should include a valgus stress test to assess for instability of the anterior oblique band of the ulnar collateral ligament (see images below). The test is performed with the patient supine and the arm abducted 90º. The shoulder and arm are externally rotated 90º, with the elbow flexed at least 15º to unlock the olecranon. Valgus stress is then placed through the elbow to assess for ligamentous instability

What is the pathophysiology for proximal humeral fracture?

The regional differences in the proximal humerus must be taken into account when attempting to reduce tuberosity fragments. The cortex of the proximal humerus near the greater tuberosity becomes progressively thicker distally. The exact location of the fracture line depends on the mechanism of, and energy from, the injury. In fractures in the thinnest cortical bone, the fracture lines can be difficult to appose. These fractures are produced by low-energy forces, occur in porotic bone, and typically are comminuted. Conversely, the denser cortical bone near the biceps groove, and more distally on the shaft, provides an easier surface to approximate fracture lines. Fractures in this area are produced by high-energy forces; the fracture pattern depends on the applied force. Indirect forces cause most shoulder fractures. The predominant force can cause predictable fracture patterns. Such injury forces are tension, axial compression, torsion, bending, and axial compression with bending. The primary fracture patterns from these forces are transverse, oblique, and spiral. For each fracture pattern, a preferred method of fixation has been developed to resist displacement forces. Unfortunately, these patterns have not been well described in the shoulder. The orientation of the fracture pattern as a result of tension depends on the muscle-tendon unit that produced most of the displacement force. Treatment recommendations for these fractures are based on factors such as patient motivation, medical history, coexisting medical morbidities, and the most influential factor, the fracture type. Fracture classification has recently been reconsidered. Neer's 4-part classification, with modifications of the 4-part valgus impacted type being separated from 4-part fractures in which the humeral head has been extruded laterally, is used primarily to separate these fractures into treatment groups. The majority of fractures are nondisplaced, and nonoperative treatment usually is appropriate. With fracture displacement, operative intervention typically is necessary. Operative treatment includes closed reduction with percutaneous fixation, open reduction and internal fixation, and humeral head replacement.[7] Fracture patterns best suited for arthroplasty are 4-part fractures, fracture dislocations, head-splitting fractures, impaction fractures, humeral head fractures with involvement of more than 50% of the articular surface, and 3-part fractures in elderly patients with osteoporotic bone. However, heterogeneity of fracture patterns exists within these groups.

What is the background of unlar collateral ligament?

The ulnar collateral ligament (UCL) of the elbow is critical for valgus stability of the elbow and is the primary elbow stabilizer.[1, 2] As such, the UCL plays an important role in most throwing sports, including baseball and javelin, as well as racquet sports and ice hockey. Elbow injuries in young athletes are generally chronic, with persistent pain and instability from repetitive overhead activities. In the athlete, ligamentous injury can also be heralded by an acute traumatic event such as an elbow dislocation. This can then lead to chronic pain and valgus instability. An understanding of the anatomy and biomechanics of the elbow in throwing sports is essential to the correct diagnosis and treatment of this potentially disabling injury.[3] Recent study Bushnell et al examined maximum pitch velocity and elbow injury in 23 professional baseball pitchers over a period of 4 seasons and found a statistically significant association (P = .0354).[4] Of 9 pitchers who had elbow injuries, 4 had an elbow muscle strain and/or joint inflammation, and 5 had an ulnar collateral ligament sprain or tear. Surgery was required for 3 of the 5 players with ulnar collateral ligament injuries;; these pitchers also had the highest maximum ball velocity.[4]

What are the causes of little league elbow syndrome?

Training errors, such as abrupt changes in intensity, duration, or frequency of throwing activity are frequently associated with sports injuries. Poor coaching and lack of preseason conditioning can also contribute to an increased risk of injury. Strength and flexibility imbalances can indirectly cause elbow injuries, as can injuries to other areas of the body (current and/or previous injuries). Anatomic malalignment of the lower extremities can cause little league elbow syndrome. Improper footwear or playing surface can result in an insecure platform for stability in throwing activities. Associated disease states or preexistent injury can be causative. Growth patterns are sometimes implicated as the cause of little league elbow syndrome. Physeal or growth cartilage at the epiphysis or apophysis is less resistant to repetitive trauma than fused adult bone at ligamentous and tendinous insertions;; thus, skeletally immature athletes more commonly develop growth plate (apophyseal) injuries. In addition, rapid growth (growth spurts) causes increased muscle and tendon tightness around a joint, resulting in loss of flexibility, biomechanical imbalance, and an increased risk of injury.

What is the intervention of radial head fracture?

Type I Non-operative managment is indicated with a splint or sling Motion is recommended early on, with care to ensure that it is not progressed to aggressively which may cause displacement of the fracture Type II The large pieces will be fixated surgically with pins or screws If the pieces are too small, they will be removed surgically If the pieces cannot be fit together well, they will also be removed - at times this includes the entire radial head Other soft tissue injuries to the surrounding ligaments will also be repaired Type III Surgery is required to remove the multiple pieces that are irrepairable Soft tissue injury in cases this severe is expected - with repairs to ligaments and tendons required Joint damage needs to be assessed and repaired as well Stiffness is a compon complication post-operatively and motion is indicated to minimize this Radial head prosthesis is an option to decrease deformity at the elbow

What are the different types of SLAP lesion?

Type I - fraying/ degeneration of labrum with firm labral attachment maintained Type II - detachment of superior labrum and biceps anchor from glenoid 1) anterosuperior lesion 2) posterosuperior lesion 3) combined anterior and superior lesions Type III - bucket-handle tear of labrum with biceps anchor intact Type IV - bucket-handle tear of labrum and biceps tendon Type V - includes a Bankart lesion Type VI -disruption of biceps anchor with anterior or posterior sup. labral flap Type VII - SLAP lesion including area inferior to middle glenohumeral ligament

What are the common surgical options for SLAP lesion?

Type I lesions are generally treated with debridement back to a stable labral rim. Type III lesions should be excised and debrided back to a stable rim. For treating type II lesions, the labrum should be reattached to the glenoid and the biceps anchor stabilized. Treatment of type IV lesions depends on the severity and varies between resection and superior labrum reattachment, a side-to-side repair and labrum reattachment, or biceps tenodesis.

When do you let the patient with SICK shoulder start throwing again?

When the affected scapula is 50% or more improved in position from its initial pathologic position, the thrower is begun on an interval throwing program, if asymptomatic, and continues the scapular program until the scapula is symmetric with the other side (Fig 8). At that time, return to sport and unrestricted throw- ing is allowed and the thrower is strongly encouraged to maintain an every-other-day scapular muscle- strengthening program to prevent recurrence of the syndrome.

A 195 pound 17 year old male returns to football daily doubles after summer break. After a particularly intense morning workout (including eccentric weight training), and practice in the 90 degree weather in the early evening, he complains of nausea, as well as bilateral pain of his gastrocnemius and quadriceps. He is markedly point tender to the bilateral gastrocnemius and quadriceps musculature as well. You compared his pre-practice weight to his post-practice weight, and find a difference of 0.5 pounds. When causes his underlying pathology?

a. Damage of myofibrils and the sarcolemma Incorrect: Hypocalcemia (decreased calcium) is not consistent with AER.

Humeral retroversion in the baseball pitcher is often correlated with which of the following?

a. Glenohumeral internal rotation deficit Correct: GIRD (Glenohumeral internal rotation deficit) is indeed correlated with humeral retroversion. See Reagan et al. (2002).

An 18 year old female soccer player receives a blow to the medial knee as she plants her left leg. She immediately drops to the ground, complaining of 10/10 knee pain. Her knee looks like the following. Assuming your state allows you as a sports physical therapist to reduce a patellar dislocation, how should this athlete be initially managed?

a. Have the athlete contract the quad to promote extension. If it does not reduce, provide manual extension, coupled with medial pressure to the laterally displaced patella. incorrect: If it is within your scope of practice (as the question states), you should reduce the dislocation. It will be easiest if it is done immediately, and no time is wasted.

The patient is an 18-year-old volleyball player who complains of generalized right shoulder pain. The patient does not recall a specific mechanism of injury but states that the pain has been recurrent over the past year. The patient complains of pain when reaching for books in the school locker and during overhead movements while playing volleyball. The patient is right-hand dominant with no significant past medical history. Examination: Posture: Slight forward head and shoulders Integumentary: No right shoulder effusion noted; no apparent decrease in right shoulder muscle tone compared to the left; right shoulder girdle slightly lower than left ROM: WNL - upper extremities and cervical region Strength: Left upper extremity 5/5 throughout Right upper extremity 5/5 except 4/5 noted to the following muscles: shoulder external rotators; middle and lower traps Sensation: Intact and bilaterally symmetrical to light touch and sharp/dull to both upper extremities Palpation: Tenderness noted over the right infraspinatus/teres minor Special Testing: Speed's test (+) Empty can (-) Drop arm (-) Hawkins-Kennedy (+) Sulcus sign (+) Apprehension (-) but creates a painful response Relocation test (+) for pain reduction Lateral scapula slide test (from inferior angle of scapula) Neutral upper extremity position: 5 cm on L/ 6.5 cm on R Hands on hips: 7 cm on L/ 9.5 cm on R What is the best description of this patient's primary diagnosis? Choose one answer.

a. Secondary Shoulder Impingement Correct! Secondary shoulder impingement describes a relative impingement occurring in the presence of rotator cuff weakness or instability of the capsule. This patient is a good fit for this condition.

The patient is an 18-year-old volleyball player who complains of generalized right shoulder pain. The patient does not recall a specific mechanism of injury but states that the pain has been recurrent over the past year. The patient complains of pain when reaching for books in the school locker and during overhead movements while playing volleyball. The patient is right-hand dominant with no significant past medical history. Examination: Posture: Slight forward head and shoulders Integumentary: No right shoulder effusion noted; no apparent decrease in right shoulder muscle tone compared to the left; right shoulder girdle slightly lower than left ROM: WNL - upper extremities and cervical region Strength: Left upper extremity 5/5 throughout Right upper extremity 5/5 except 4/5 noted to the following muscles: shoulder external rotators; middle and lower traps Sensation: Intact and bilaterally symmetrical to light touch and sharp/dull to both upper extremities Palpation: Tenderness noted over the right infraspinatus/teres minor Special Testing: Speed's test (+) Empty can (-) Drop arm (-) Hawkins-Kennedy (+) Sulcus sign (+) Apprehension (-) but creates a painful response Relocation test (+) for pain reduction Lateral scapula slide test (from inferior angle of scapula) Neutral upper extremity position: 5 cm on L/ 6.5 cm on R Hands on hips: 7 cm on L/ 9.5 cm on R The positive Speed's test for this patient can be explained by what?

a. Upward migration of the humerus due to instability during arm flexion Correct: This is a mechanism for causing irritation to the biceps tendon in type two shoulder impingement!

The patient is an 18-year-old volleyball player who complains of generalized right shoulder pain. The patient does not recall a specific mechanism of injury but states that the pain has been recurrent over the past year. The patient complains of pain when reaching for books in the school locker and during overhead movements while playing volleyball. The patient is right-hand dominant with no significant past medical history. Examination: Posture: Slight forward head and shoulders Integumentary: No right shoulder effusion noted; no apparent decrease in right shoulder muscle tone compared to the left; right shoulder girdle slightly lower than left ROM: WNL - upper extremities and cervical region Strength: Left upper extremity 5/5 throughout Right upper extremity 5/5 except 4/5 noted to the following muscles: shoulder external rotators; middle and lower traps Sensation: Intact and bilaterally symmetrical to light touch and sharp/dull to both upper extremities Palpation: Tenderness noted over the right infraspinatus/teres minor Special Testing: Speed's test (+) Empty can (-) Drop arm (-) Hawkins-Kennedy (+) Sulcus sign (+) Apprehension (-) but creates a painful response Relocation test (+) for pain reduction Lateral scapula slide test (from inferior angle of scapula) Neutral upper extremity position: 5 cm on L/ 6.5 cm on R Hands on hips: 7 cm on L/ 9.5 cm on R The lateral scapular slide test indicates:

b. Weakness of the scapular stabilizers Correct! "The Lateral Scapular Slide Test (LSST) is used to determine scapular position with the arm abducted 0, 45, and 90 degrees in the coronal plane. Assessment of scapular position is based on the derived difference measurement of bilateral scapular distances." Patients with suspected scapular weakness have increased measurement values on the involved side. (Odom et al., 2001). Now, that being said, it isn't a stellar test. Even the authors note: "Our results suggest that measurements of scapular positioning based on the difference in side-to-side scapular distance measures are not reliable. Furthermore, the results suggest that sensitivity and specificity of the LSST measurements are poor and that the LSST should not be used to identify people with and without shoulder dysfunction." However, you may still see it on your exam.

An 18 y/o 220# lineman c/o nausea, dizziness, fatigue and headache while practicing aggressively at daily doubles in hot, humid conditions. His skin appears cold, clammy and ashen. Appropriate treatment would be:

c. Escort athlete to shade and remove gear. Begin cooling with fanning and ice bags. Initiate oral hydration and monitor core temperature. Initiate EMS Correct: This individual is showing s/s consistent with heat exhaustion. This is the proper care for an individual with heat exhaustion. If his core body temperature continues to increase (especially with a core temp of >104), then rapid cooling via ice-water immersion is indicated as well as an immediate transport to ED. See chart below from DeFranco et al. (2008).

You are the assigned sports medicine personnel for a local high-school baseball team. You have been watching the weather all day and thunderstorms are predicted but no watch or warning has been completed on the local weather station. You report to the field 30 minutes prior to the scheduled start of the game with intermittent drizzling occurring and you discuss the situation with the designated officials. There has been no delay or cancellation announced at this point and the game begins as planned. In the second inning, you hear some thunder rumbles and then notice in the distance a lightening flash. The rain begins, and you note a flash-to-bang count of 20 seconds. Your next step is:

c. Evacuate all of the players, personnel and spectators to the high school gym, the designated safe haven Correct: The flash-to-bang ratio indicates that the lightening is within striking distance. See NCAA Sports Medicine Handbook section 1D

A 18 year old cross country runner, with a resting respiratory rate of 12 breaths per minute, finishes his race with a respiratory rate of 60 breaths per minute. 30 minutes after the conclusion of his race, his respiratory rate is 16 breaths per minute. What explains this individuals increased rate of breathing 30 minutes following his race?

c. Excess postexercise oxygen consumption Correct: EPOC is the oxygen uptake above resting values used to restore the body to the preexercise condition. With aerobic exercise, the EPOC effect is greatest soon after the exercise is completed and decays to a lower level over time. However, it has been shown to last for several hours.

A 21 year old collegiate lacrosse player comes into your clinic with a significant quadriceps contusion after being hit with a lacrosse stick. After 4 weeks of modalities, manual therapy, and exercise, he still complains of mild-moderate pain. Evaluation on an isokinetic machine reveals a 30% deficit in peak force, as well as a decease in eccentric strength. What do you suspect?

d. Heterotrophic ossification Correct: HO is defined as abnormal formation of true bone within extra-skeletal soft tissues. This was formerly called myositis ossificans; however, this has fallen out of favor because the condition is not always inflammatory and ossification occurs in soft tissues other than muscle. There is a 9-20% chance of developing HO after significant trauma to the quadriceps. Risk factors include: 1) limited knee range of motion, 2) previous quad contusion, 3) treatment for a quad contusion that was delayed more than 72 hours, 4) knee effusion (swollen knee), and 5) injury sustained by playing football

An 18 year old female soccer player receives a blow to the medial knee as she plants her left leg. She suffers a lateral patellar dislocation. This is her first patellar dislocation. She is presumed to be reduced, and is transferred to the emergency room. Which imaging will be necessary to ensure reduction?

d. Standard plain film radiographic views with merchant view Incorrect: The sunrise view is very similar to the Merchant view, but is taken in 115 degrees flexion. Because this degree of flexion would be very pain after dislocation, and because dislocations tend to occur closer to full knee extension, it is not ideally suited for instability. The sunrise view is best at identifying fractures and DJD. The merchant view is a radiographic projection for evaluating the patella, in which the patient is supine and the knees are flexed 45 degrees over the end of the table. The x-ray tube is at a 30 angle with the floor. It is the ideal visual for patellar instability.

A college level pitcher is referred to you for shoulder pain localized to the right posterior shoulder. He is right hand dominant. This pain increases with pitching and has started becoming painful at rest as well. He reports that the posterior pain is worst at the end range of the wind-up phase of his pitch. Upon examination he has full range of motion of his right shoulder except for external rotation, which is increased on the right, and internal rotation, which is decreased on the right. His posterior shoulder pain is reproduced at end range abduction and external rotation, and diminished with pressure to the anterior glenoid. With manual muscle testing he demonstrates 4-/5 strength in external rotation that is painful, and is otherwise strong and without pain with muscle testing. Neer impingement test is positive, Hawkins-Kennedy impingement test is negative. O'Brien's (Active Compression) test is negative as is the internal rotation resisted stress test. A load-shift test demonstrates excessive anterior translation. In this individual, which muscles of the shoulder dynamically resist excessive anterior translation of the glenohumeral joint?

d. The infraspinatus and teres minor muscles Incorrect: While the subscapularis portion of the rotator cuff and the long head of the biceps' attachment to the anterior labrum may passively constrain anterior translation of the glenohumeral joint, they do not actively resist anterior translation.

What x-rays do you take for medial instability?

f medial instability is suspected, stress AP radiographs can be performed with the use of a valgus stress radiography machine (Telos, Weiterstadt, Germany). We typically obtain AP views with 0, 5, 10, and 15 dN of valgus stress applied to each elbow at 25° of flexion.

What are the two predisposing factors that increase the risk of a SLAP lesion?

increase the risk of developing a SLAP lesion are throwing-acquired glenohumeral internal rotation deficit (GIRD), a malpositioned SICK scapula, or both

What is the MOI of anterior shoulder dislocation?

mechanism of injury is extreme abduction, external rotation, extension, and a posterior directed force against the humerus. Forceful abduction or external rotation alone can also lead to dislocation (about 30% of cases), as can a direct blow to the posterior humerus (29%), forced elevation and external rotation (24%), and a fall onto an outstretched hand (17%).

What are the mechanics for the bicep load test?

n 1999, Kim et al.9 described this test for the evaluation of SLAP lesions in patients with recurrent anterior dislo- cations. "This test is performed with the patient in the supine position. The examiner sits adjacent to the patient on the same side as the affected shoulder and gently grasps the patient's wrist and elbow. The arm to be exam- ined is abducted at 90°, with the forearm in the supinated position. The patient is allowed to relax and an anterior apprehension test is performed. When the patient be- comes apprehensive during the external rotation of the shoulder, external rotation is stopped. The patient is then asked to flex the elbow while the examiner resists the flexion with one hand and asks how the apprehension has changed, if at all. If the apprehension is lessened, or if the patient feels more comfortable than before the test, the test is negative for a SLAP lesion. If the apprehension has not changed, or if the shoulder becomes more painful, the test is positive. The test is repeated and the patient is instructed not to pull the whole upper extremity but to bend the elbow against the examiner's resistance. The examiner should be sitting adjacent to the shoulder to be examined at the same height as the patient, and he or she should also face the patient at a right angle. The direction of the examiner's resistance should be on the same plane as the patient's arm so as not to change the degree of abduction and rotation of the shoulder. The forearm should be kept in the supinated position during the test." This test is depicted in Figure 8. This test was assessed in 75 patients with a reported sensitivity of 91% and a specificity of 97%. The positive predictive value was 83% and the negative predictive value was 98%.

What is the rehab for little league elbow syndrome?

n general, overuse injuries are treated using protection, rest, ice, compression, elevation, medications, and modalities (PRICEMM).[12] PRICEMM is an accepted treatment regimen for overuse syndromes of the elbow and forearm, whether these conditions are nerve entrapments, tendinoses, or instability syndromes. Protection means the athlete needs to modify his or her activity and equipment to allow proper healing and to prevent further injury. Rest does not mean cessation of activity, which can lead to deconditioning, but rather modified activity, or relative rest, which does not aggravate the injury. Ice is used to alleviate patient's pain and help control swelling. Compression is used to prevent swelling, but this should be used with caution when dealing with nerve entrapment syndromes. Elevation is used to prevent venous stasis around the injury, which can lead to increased inflammation and pain. Medications that can be used include nonsteroidal anti-inflammatory drugs (NSAIDs) and, very rarely, corticosteroids. Modalities can include ultrasound, electrical stimulation, and friction massage. Myofascial release techniques are often very helpful in reducing the soft-tissue restrictions of motion.

What are the 5 stages of MCL sprains?

n stage 1 (debonding), evaluation of the elbow with valgus stress would not necessarily demonstrate laxity (medial ulnohumeral joint opening). n stage 2 (partial tear), some of the debonded collagen fibers tear. Healing with a scar then usually occurs with the ligament in a lengthened, attenuated posi- tion and would be evident on gross examination. Arthroscopic and radiographic stress tests would also be positive in this phase In stage 3 (complete tear), the entire MCL ruptures, either as a result of having progressed through stages 1 and 2 (with or without symptoms) or from a sudden load that exceeded the strength of the liga- ment (Figure 5).

What is the sign and symptoms are pronator teres syndrome?

o Compression of the median nerve in the region of the elbow or proximal part of the forearm can cause pain and/or numbness in the distribution of the distal median nerve, and weakness of the muscles innervated by the anterior interosseous nerve: the flexor pollicis longus ("FPL"), the flexor digitorum profundus of the index finger ("FDP IF"), and the pronator quadratus ("PQ"). o The characteristic physical finding is tenderness over the proximal median nerve, which is aggravated by resisted pronation of the forearm. o The flexor pollicis longus and FDP of the index finger are weak, leading to impairment of the pincer movement. This reflects involvement of the anterior interosseous nerve. o Sensory changes may be found in the first three fingers as well as in the palm, indicating impairment of the median nerve proximal to the flexor retinaculum.

What is the pathology and MOI of medial epicondylitis?

o Medial epicondylitis is the inflammation of the medial epicondyle of the humerus. It is mostly an overload injury and it's most sensitive region is located near the origins of the wrist flexor muscles. Histologically it is shown that medial epicondylitis is the result of microtearing in the tendon that isn't fully relapsed. It is also referred to as "golfer's elbow" but in 95% of the cases those involved are not sportsmen. It is caused due to sudden violence to the tendons by repeated usage or by chronic strain.

What is the pathology and MOI of elbow dislocation?

o Occurs when the radius and ulna are forcefully driven posterior to the humerus. Elbow dislocations are the second most common dislocation in an adult behind the shoulder dislocation. The injury is often caused by a traumatic fall on an outstretched arm causing a hyperextension at the elbow.

What is the signs and symptoms for volkmann's contracture?

o Paleness, pallor o The wrist is in palmar flexion o Pain occurs with passive stretching of the flexor o Palpation of the affected region creates persistent pain o It is possible that the pulsations cannot be felt in the swollen arm, mainly in the distal part (pulse less ness) o There are also neurological limitations noticeable from the muscles that pinch the neural pathways, there is a decreased sensation, andthere is an observable motor deficit

What is the pathology and MOI of pronator teres syndrome?

o Pronator teres syndrome is a compression neuropathy of the median nerve at the elbow. It is rare compared to compression at the wrist (carpal tunnel syndrome) or isolated injury of the anterior interosseous branch of the median nerve (anterior interosseous syndrome). o The most common cause is entrapment of the median nerve between the two heads of the pronator teres muscle. Other causes are compression of the nerve from the fibrous arch of the flexor superficialis, or the thickening of the bicipital aponeurosis.

What is the S&S symptoms with medial epicondylitis?

o The patient usually complains of pain about the elbow distal to the most medial epicondyle of the humerus with radiation up and down the arm, most common on the ulnar side of the forearm, wrist and occasionally the fingers. o Pain is evoked by resisted flexion of the wrist and pronation o There is often a noticeable weakness of the hand grip

What are the S&S for elbow dislocation?

o The person would feel immediately unstable o There might be a popping sensation or noise during the event of the dislocation o Upon palpation and observation the olecranon is prominent creating a divot over the distal triceps o After reduction radial sided elbow pain may be persistent in addition to "snapping" during supination o Swelling, joint line tenderness and a decreased ROM would be present

What is the pathology and MOI of UCL tear?

o Ulnar collateral ligamentous tears are often caused by overuse. Some mechanisms of injuries include a forced twisting of the arm, falling on an outstretched arm, and repeated overhead movement, such as pitching, volleyball or tennis. Ligamentous injuries in theelbow are often rare

What are the S&S of UCL tear?

o Valgus testing would show medial collateral ligament instability o Valgus overload or stress movement from pitching or throwing could cause pain o It is often seen in younger male pitchers, and can be found in athletes involving repetitive overhead activities such as tennis or volleyball

Where are the nerve endings in the labrum?

o mechanoreceptors have been identified within the glenoid labrum.53 However, free nerve endings have been isolated in the fibrocartilagenous tissue of the labrum, the biceps-labrum complex, and the connective tissue surrounding the labrum

What are the 3 stages of osteochondritis dessicans

tage one describes patients without evidence of sub- chondral displacement or fracture. Stage two refers to lesions showing evidence of subchondral detachment or articular cartilage fracture. Stage three lesions involve detached osteochondral fragments, potentially resulting in intra-articular loose bodies.

What x-ray view is used to see a osseus bankart lession

the west point view

What is the examination for lateral epicondylitis?

• To examine the sincerity of the tennis elbow, there is a dynamometer and a Patient-rated Tennis Elbow Evaluation Questionnaire (PrTEEQ).[10][11] The dynamometer measures grip strength.[12][13] The PrTEEQ is a 15-item questionnaire, it's designed to measure forearm pain and disability in patients with lateral epicondylitis. The patients have to rate their levels of tennis elbow pain and disability from 0 to 10, and consists of 2 subscales. There is the pain subscale (0 = no pain, 10 = worst imaginable) en the function subscale (0 = no difficulty, 10 = unable to do). • A positive sign is tenderness to palpation at the anterior epicondyle • Cozen's sign: The patient is positioned with the upper extremity relaxed. The examiner stabilizes the patient's elbow with one hand and the patient is instructed to make a fist, pronate the forearm, and radially deviate the wrist. At last, the patient is instructed to extend the wrist against resistance that is provided by the examiner. An altenative is resisted extension of the middle finger that can cause pain at the extensor carpi radialis brevis origin. The test is positive if the patient experiences a sharp, sudden, severe pain over the lateral epicondyle.[14][15] • Chair test: The patient grasps the back of the chair while standing behind it and attempts to raise it by putting their hands on the top of the chair back. Pain reproduction at the lateral epicondyle is a positive test. • Mill's Test: The patient is positioned in standing with the upper extremity relaxed at side and the elbow extended. The examiner passively stretches the wrist in flexion and pronation. Pain at the lateral epicondyle or proximal musculotendinous junction of wrist extensors is positive for lateral epicondylitis. • The coffee cup test (by Coonrad and Hooper) where picking up a full cup of coffee is painful

What is the pathophysiology of cubital tunnel syndrome?

As the elbow moves from extension to flexion, the distance between the medial epicondyle and the olecranon increases 5 mm for every 45° of elbow flexion. Elbow flexion places stress on the medial collateral ligament (MCL) and the overlying retinaculum. The shape of the cubital tunnel changes from a round to an oval tunnel, with a 2.5mm loss of height, because the cubital tunnel rises during elbow flexion and the retrocondylar groove on the inferior aspect of the medial epicondyle is not as deep as the groove is posteriorly. The cubital tunnel's loss in height with flexion results in a 55% volume decrease in the canal, which further results in the mean ulnar intraneural pressure increasing from 7 mm Hg to 14 mm Hg.[7, 8] A combination of shoulder abduction, elbow flexion, and wrist extension results in the greatest increase in cubital tunnel pressure, with ulnar intraneural pressure increasing to about 6 times normal.[9, 10, 11, 12, 13] Traction and excursion of the ulnar nerve also occur during elbow flexion, as the ulnar nerve passes behind the axis of rotation of the elbow. With full range of motion (ROM) of the elbow, the ulnar nerve undergoes 910 mm of longitudinal excursion proximal to the medial epicondyle and 36 mm of excursion distal to the epicondyle.[14] In addition, the ulnar nerve elongates 58 mm with elbow flexion. Within the cubital tunnel, the measured mean intraneural pressure is significantly greater than the mean extraneural pressure at elbow flexion of 90° or more.[15] With the elbow flexed 130°, the mean intraneural pressure is 45% higher than the mean extraneural pressure. At this amount of flexion, significant flattening of the ulnar nerve occurs; however, with full elbow flexion, no evidence exists of direct focal compression, suggesting that traction on the nerve in association with elbow flexion is responsible for the increased intraneural pressure. In addition, studies have shown that the intraneural and extraneural pressures within the cubital tunnel are lowest at 45° of flexion. As a result of these studies, 45° of flexion is considered to be the optimum position for immobilization of the elbow to decrease pressure on the ulnar nerve. Subluxation of the ulnar nerve is a common finding. Childress looked at 2000 asymptomatic elbows.[16] None of the patients were aware of ulnar nerve subluxation; however, 16.2% of these patients had subluxation of the ulnar nerve following flexion past 90°. Of the 325 patients with subluxation of the ulnar nerve, only 14 had unilateral subluxation. Although subluxation is a common finding and does not appear to cause cubital tunnel syndrome, the friction generated with repeated subluxation may cause inflammation within the nerve, and in the subluxed position, the nerve may be more susceptible to inadvertent trauma. Sunderland described the internal topography of the ulnar nerve at the medial epicondyle.[17] The sensory fibers and intrinsic muscle nerve fibers are located superficially. In contrast, the motor fibers to the flexor carpi ulnaris (FCU) and flexor digitorum profundus (FDP) are located deep within the nerve.[18, 19, 20] The central location protects the motor fibers and explains why weakness of the FCU and FDP is not typically seen in ulnar neuropathy.[21, 22, 23, 24] Proximal compression of a nerve trunk, such as that which occurs with cervical radiculopathy, may lead to increased vulnerability to nerve compression in a distal segment. This "double crush" condition can affect the ulnar nerve and results from disruption in normal axonal transport.[25] Histologically, severe demyelination of the nerve may occur in ulnar neuropathy. Demyelination may be located in the bulbous swelling just proximal to the entry of the nerve into the cubital tunnel. McGowan[26] established the following classification system: Grade I Mild lesions with paresthesias in the ulnar nerve distribution and a feeling of clumsiness in the affected hand; no wasting or weakness of the intrinsic muscles Grade II Intermediate lesions with weak interossei and muscle wasting Grade III Severe lesions with paralysis of the interossei and a marked weakness of the hand

What is the anatomy of cubital tunnel syndrome?

The ulnar nerve is the terminal branch of the medial cord of the brachial plexus and contains fibers from C8, T1, and, occasionally, C7.[44, 45] The ulnar nerve enters the arm with the axillary artery and passes posterior and medial to the brachial artery. The nerve travels between the brachial artery and vein. At the level of the insertion of the coracobrachialis muscle in the middle third of the arm, the ulnar nerve pierces the medial intermuscular septum (the first site of potential compression) to enter the posterior compartment of the arm.[46, 47] Here, the ulnar nerve lies on the anterior aspect of the medial head of the triceps, where it is joined by the superior ulnar emedicine.medscape.com/article/1231663‐overview#showall 5/12 9/3/12 Cubital Tunnel Syndrome collateral artery. The medial intermuscular septum extends from the coracobrachialis muscle proximally, where it is a thin and weak structure, to the medial humeral epicondyle, where it is a thick, distinct structure. The next potential site of compression is the arcade of Struthers. This structure is found in 70% of patients, 8 cm proximal to the medial epicondyle, and extends from the medial intermuscular septum to the medial head of the triceps. The arcade of Struthers is formed by the attachments of the internal brachial ligament (a fascial extension of the coracobrachialis tendon), the fascia and superficial muscular fibers of the medial head of the triceps, and the medial intermuscular septum. Next, the ulnar nerve passes through the cubital tunnel. The deep forearm investing fascia of the FCU and the arcuate ligament of Osborne, also know as the cubital tunnel retinaculum (CTR), form the roof of the cubital tunnel. The CTR is a 4 mm wide fibrous band that passes from the medial epicondyle to the tip of the olecranon. Its fibers are oriented perpendicularly to the fibers of the FCU aponeurosis, which blends with its distal margin. The elbow capsule and the posterior and transverse portions of the MCL form the floor of the cubital tunnel. The medial epicondyle and olecranon form the walls. O'Driscoll believes that the roof of the cubital tunnel, or Osborne ligament, is a remnant of the anconeus epitrochlears muscle.[48] He also identified a retinaculum at the proximal edge of the arcuate ligament in all but 4 of 25 cadaveric specimens. He classified this retinaculum as 1 of 4 types, as follows: An absent retinaculum A thin retinaculum that becomes tight with full flexion without compressing the nerve A thick retinaculum that compresses the nerve between 90° and full flexion An accessory anconeus epitrochlears muscle Upon entering the cubital tunnel, the ulnar nerve gives off an articular branch to the elbow. It then passes between the humeral and ulnar heads of the FCU, the next potential site of compression. The nerve then descends into the forearm between the FCU and the FDP muscles. About 5 cm distal to the medial epicondyle, the ulnar nerve pierces the flexor pronator aponeurosis, the fibrous common origin of the flexor and pronator muscles. The flexorpronator aponeurosis is another point of possible compression, with compression of the ulnar nerve beneath the muscle belly of the FCU. The ligament of Spinner is an additional aponeurosis between the flexor digitorum superficialis (FDS) of the ring finger and the humeral head of the FCU. This septum is independent of the other aponeuroses and attaches directly to the medial epicondyle and medial surface of the coronoid process of the ulna. This structure was found in 4 of 20 specimens in one study, and it is important to recognize and to release with anterior transposition of the ulnar nerve to prevent kinking. In the forearm, the ulnar nerve extends motor branches to the FCU and the FDP of the ring and small fingers. The ulnar nerve may extend as many as 4 branches to the FCU, ranging from 4 cm above to 10 cm below the medial epicondyle. Proximal dissection of the first motor branch to the FCU from the ulnar nerve may be performed up to 6.7 cm proximal to the medial epicondyle, facilitating anterior transposition of the nerve. An aberrant muscle, the anconeus epitrochlears, has been found in 328% of cadaver elbows and in as many as 9% of patients undergoing surgery for cubital tunnel syndrome. This muscle arises from the medial humeral condyle and inserts on the olecranon, crossing superficially to the ulnar nerve, where it may cause compression.[49] The arcade of Struthers must be differentiated from the ligament of Struthers, which is found in 1% of the population and extends from a supracondylar bony or cartilaginous spur to the medial epicondyle. This supracondylar spur can be found on the anteromedial aspect of the humerus, 5 cm proximal to the medial epicondyle, and it can often be seen on radiographs. The ligament of Struthers may occasionally cause neurovascular compression. This compression generally involves the median nerve or the brachial artery; however, the ulnar nerve can also be compressed by this structure. Posterior branches of the medial antebrachial cutaneous nerves cross the ulnar nerve anywhere from 6 cm proximal to 4 cm distal to the medial epicondyle. These branches are often cut when making the skin incision for a cubital tunnel release, creating an area of dysesthesia or resulting in potential neuroma formation. emedicine.medscape.com/article/1231663‐overview#showall 6/12 9/3/12 Cubital Tunnel Syndrome Extrinsic blood supply to the ulnar nerve is segmental and involves 3 vessels. These include the superior ulnar collateral artery, the inferior ulnar collateral artery, and the posterior ulnar recurrent artery. Typically, the inferior ulnar collateral artery (and often the posterior ulnar recurrent artery) is sacrificed with anterior transposition. At the level of the medial epicondyle, the inferior ulnar collateral artery is the sole blood supply to the ulnar nerve. In an anatomic study, no identifiable anastomosis was found between the superior ulnar collateral artery and the posterior ulnar recurrent arteries in 20 of 22 arms. Instead, communication between the 2 arteries occurred through proximal and distal extensions of the inferior ulnar collateral artery. Intrinsically, the blood supply is composed of an interconnecting network of vessels that run along the fascicular branches and along each fascicle of the ulnar nerve itself. The surface microcirculation of the ulnar nerve has been shown to have an anastomotic stepladder arrangement. The inferior ulnar collateral artery is consistently found 5 mm deep to the leading edge of the medial intermuscular septum on the surface of the triceps.[50] Finally, acute ulnar neuropathy may have a sex predilection. This perioperative condition is found 38 times more frequently in men than in women. Contreras et al revealed that the medial aspect of the elbow has 219 times more fat content in women than in men.[51] In men, the coronoid tubercle is approximately 1.5 times larger. He suggests that the coronoid process may be a potential site for ulnar nerve compression in men, and the increased subcutaneous fat around the ulnar nerve in women may provide a protective advantage against acute ulnar neuropat hy . The most common potential sites of compression of the ulnar nerve at the elbow are the medial intermuscular septum, the arcade of Struthers, the retrocondylar groove, the cubital tunnel, and the deep flexorpronator aponeurosis. The 2 most common sites of compression are the retrocondylar groove and the true cubital tunnel, where the ulnar nerve passes between the 2 heads of the FCU.

What is the nerve and muscle for the thoracodorsal nerve

1) C6-C7-C8.....2) lats.....3) extension/adduction of the arm

What is the nerve level and muscle for axillary nerve

1) C5-C6.... 2) teres minor/ deltoid.... 3) abductions of the arm/external rotation

What are the acronymes TUBS and AMBRI?

"TUBS: Traumatic, unidirectional, bankart, surgery. When considering the diagnosis of MDI, it is helpful to remember the mnemonic device AMBRII: A traumatic etiology M ultidirectional instability B ilateral involvement R ehabilitative initial management Rotator I nterval tightening with I nferior capsular shift

What are the signs and symptoms of posterior impingement?

pain weakness, loss of movement in the affected shoulder.. pain worsened with overhead movement... pain may be chronic or acute... limited ROM

What is the nerve and muscle for the ulnar nerve?

1) C6-T1.... 2) adductor pollicis/abductor digit minimi/lumbricals 3/4.....3)

What are the special tests for Lateral epicondylitis?

o Cozen's Sign o Mill's Test

What are the special tests or imaging

1) Step up deformity.. 2) compression test... 3) X-ray

What diagnostic test do you use for MDI association with MDI of the shoulder"

"Most often, plain radiographs are negative. Findings of an osseous glenoid rim fracture or a Hill-Sachs humeral head impression defect are usually not seen unless concomitant traumatic instability exists. Results of noncontrast MRI are the same as described for plain radiographs—that is, benign and negative, unless MRI is performed with contrast (gadolinium).[15, Typically, MRI-arthrography may demonstrate blunting of the labrum, diffuse capsular laxity, and increased capsular volume. Labral and capsular tears, such as those seen with traumatic instability, are unusual in classic multidirectional instability (MDI), and rotator cuff tears and SLAP lesions are only rarely seen in association with MDI of the shoulder"

What is the nerve and muscle for suprascapular nerve?

1) C5-C6 2) suprascapular/infraspinatus...3) abduction/external rotation

What is the nerve and muscle for the median nerve?

1) C5-C6-C7-C8-T1.... 2) abductor pollicis brevis/oppenens pollicis/ lumbricals 1 and 2..... 3) opposition of thumb/abduction of thumb/extension of IP joints/flexions of MP joints

What is the nerve and muscle for the radial nerve?

1) C5-C6-C7-C8...... 2) triceps/anconeus.......3) extension of forearm

What is the nerve and muscle for the musculocutaneous nerve?

1) C5-C6-C7......2) bicep brachii/coracobrachialis/brachialis...3) flexion/supination/adduction/forearm flexion

What are the mechanics for the synder bicep tension and compression test?

As stated by Snyder et al.,17 "On physical examination, the most useful diagnostic tests were the biceps tension test (resisted shoulder flexion with the elbow extended and forearm supinated) and joint compression-rotation test. The compression-rotation test is performed with the patient supine, the shoulder abducted 90° and the elbow flexed at 90°. A compression force is applied to the hu- merus, which is then rotated, in an attempt to trap the torn labrum. Labral tears may be felt to catch and snap during the test, as meniscal tears do with MacMurray's test." Figure 3 shows the compression-rotation test. No observation was made as to the accuracy of these tests; however, Field and Savoie2 observed that the biceps ten- sion test was positive in 20 consecutive patients with a diagnosis of SLAP lesion. Anterior Slide Test Kibler7 described this test in 1995. "The patient is exam- ined either standing or sitting, with their hands on the hips with thumbs pointing posteriorly. One of the exam- iner's hands is placed across the top of the shoulder from the posterior direction, with the last segment of the index finger extending over the anterior aspect of the acromion at the glenohumeral joint. The examiner's other hand is placed behind the elbow and a forward and slightly supe- riorly directed force is applied to the elbow and upper arm. The patient is asked to push back against this force. Pain localized to the front of the shoulder under the examiner's hand, and/or a pop or click in the same area, was consid- ered to be a positive test. This test is also positive if the athlete reports a subjective feeling that this testing ma- neuver reproduces the symptoms that occur during over- head activity." This test is depicted in Figure 4. The re- sults showed a sensitivity of 78.4% and a specificity of 91.5%. The authors commented that the test was useful as Figure 3. The compression-rotation test. American Journal of Sports Medicine Figure 4. The anterior slide test. an aid to diagnosis, but was not in itself sufficient to be relied on completely. The Crank Test Liu et al.11 described the crank test in 1996: "The crank test is performed with the patient in the upright position with the arm elevated to 160° in the scapular plane. Joint load is applied along the axis of the humerus with one hand while the other performs humeral rotation. A posi- tive test is determined either by 1) pain during the ma- neuver (usually during external rotation) with or without a click or 2) reproduction of the symptoms, usually pain or catching felt by the patient during athletic or work activ- ities. This test should be repeated in the supine position, where the patient is more relaxed. Frequently, a positive crank test in the upright position will also be positive in the supine position. "Tricks in performing this test are to make sure eleva- tion is kept as extreme as possible (not at 90° for the apprehension or relocation test), and axial load is applied followed by stress relocation." Figure 5 is a depiction of the crank test. This description was produced after a study of 62 pa- tients in whom the test was positive in 31 and the diag- nosis was confirmed at arthroscopy. The sensitivity was 91% and the specificity was 93%. The positive predictive value was 94% and the negative predictive value was 90%. O'Brien's Active Compression Test O'Brien et al.14 described this test in 1998 to distinguish between superior labral and acromioclavicular abnormal- ities. The test is depicted in Figure 6. "This test was

What is the presentation for posterior elbow dislocation?

After a PED, a person may feel immediate instability. Depending on the severity of the damage, they may report a 'popping' sensation or noise upon dislocation.[1] With palpation and observation, the olecranon is prominent creating a divot over the distal triceps.[9] After reduction, radial-sided elbow pain may persist in addition to 'snapping' with supination.[10] Swelling, joint line tenderness, and decreased range of motion (ROM) should be expected.[3][9][11] Various degrees of injury are also seen in muscles surrounding the elbow www.physio‐pedia.com/index.php?title=Posterior_Elbow_Dislocation 3/11 9/3/12 Posterior Elbow Dislocation ‐ Physiopedia, universal access to physiotherapy knowledge. that originate at the medial or lateral epicondyles. When ligaments are not compromised, recurrent dislocations are not common.

You are climbing Mount Rainier with friends and you are currently at an elevation of 9000 feet. One of your buddies stops secondary to, "not feeling well". Upon questioning, he states that he has a headache, feels nauseous, weak and a little dizzy. The group stops while he rests and drinks water. After 15 minutes, he states he is feeling better, but still has a little headache and nausea. You elect to drop 1000 feet. After 3 hours, your buddy has urinated twice and denies nausea but continues to have a headache. What is the best plan of action?

Call it a day and descend. Incorrect: If after the initial 1000 foot descent he still has a headache, it is necessary to pack up for the day and head home. It is not recommended to continue to climbing the same day at this point. See the snapshot below from Benner (2007) for more detailed info about altitude illness.

What is the prognosis for medial humeral fracture?

Children younger than 5 years tend to have nondisplaced fractures and good results. Older children tend to have more severely displaced fractures that involve the partially developed trochlear ossification center. Good results are obtained in patients who are seen early after injury and who have adequate reduction and fixation of their fractures. In patients seen late, proper reduction and immobilization are necessary to achieve satisfactory results; however, this is usually more difficult to achieve. Preserving full range of motion of the elbow depends on restoring the snug relationship between the humerus and ulna throughout the arc of motion. Most children with medial epicondyle fractures treated either nonsurgically or surgically end up with good long-term functional results. Children with fracture displacement greater than 5 mm treated nonsurgically had results that were comparable to those of children who underwent surgical stabilization. ORIF is required in children with medial epicondyle fractures and incarcerated fragments that block motion. Comminuted medial epicondyle fractures appear to do worse if the epicondyle is excised as opposed to nonsurgical fracture treatment.

What is the epidemiology for clavicle fracture?

Clavicle fractures are common injuries, representing about 4-10% of all adult fractures and 35-45% of all fractures that occur in the shoulder girdle area. If these fractures are classified into thirds, as proposed by Allman, the most frequent site of injury is at the middle third (group I fractures). These fractures account for approximately 72-80% of all fractures of the clavicle. Approximately 25-30% of clavicle fractures occur at the lateral clavicle (group II). Fractures of the medial clavicle are quite rare, accounting for 2% of all clavicle fractures in a recent epidemiological study by Nowak

What motion with anterior shoulder instability shoulder be avoid with the first 6 weeks?

Combined external rota- tion and abduction range of motion is avoided for the first 6 postoperative weeks

What is the etiology of cubital tunnel syndrome?

Cubital tunnel syndrome may be caused by constricting fascial bands, subluxation of the ulnar nerve over the medial epicondyle, cubitus valgus, bony spurs, hypertrophied synovium, tumors, ganglia, or direct compression. Occupational activities may aggravate cubital tunnel syndrome secondary to repetitive elbow flexion and extension. Certain occupations are associated with the development of cubital tunnel syndrome; however, a definite relationship with occupational activities is not well defined.[4, 5, 6]

what is the pathology/treatment/surgery for Valgus Extension Overload Syndrome with Posterior Olecranon Impingement

Description Impingement of posterior olecranon due to excessive valgus compressive forces, results in olecranon osteophytes that may fracture and form loose bodies Treatment options Nonoperative rehabilitation, open surgical treatment, arthroscopic surgical treatment Author's preference Arthroscopic decompression (debridement)

Intrafusal muscle fibers (muscle spindles) are largely responsible for what?

Detect the amount and rate of change of length in a muscle Correct: Both nuclear bag and nuclear chain fibers (both types of intrafusal muscle fibers) detect the amount and rate of change of length in a muscle

What is the definition for medial epidcondylitis

Epicondylitis is a painful inflammation or tenderness of the muscles and soft tissues around an epicondyle. Medial epicondylitis is mostly an overload injury. The most sensitive region is located near the origin of the wrist flexors on the medial epicondyle of the humerus. Sometimes the patient also experiences pain on the ulnar side of the forearm, the wrist and occasionally in the fingers. Although epicondylitis means there is an inflammation, there is some controversy with this pathology. Histologically it has been shown that medial epicondylitis is the result of microtearing in the tendon that isn't fully relapsed. Some physical therapists prefer the term tendonosis instead of epicondylitis.[1] These conditions are caused by a constant trauma and tension of the tendon attachment to the medial

What are the contraindication for Monteggia Fracture?

Few contraindications to surgery exist. Once the radial head is reduced in closed injuries, surgical treatment may be delayed until the patient is stable and the surgery may be performed in a more elective fashion. Open fractures require emergent operative attention. If the wound is open and heavily contaminated, serial debridement may be indicated prior to plate fixation. In medically unstable patients, emergent treatment of the open wound is still necessary. The procedure can be limited to irrigation and debridement of the open wound and closed reduction at the bedside. This can be performed under regional anesthesia or local anesthesia with sedation if absolutely necessary. If the patient is unable to tolerate operative treatment, the fracture dislocation may be treated with cast immobilization after reduction of the radius and irrigation and debridement if the fracture is open.

What are the mechanics of the anterior release test

Gross and Distefano4 described the anterior release test in 1997 to identify occult instability. "The anterior release test is performed with the patient in the supine position, with the affected shoulder over the edge of the examining table. The patient's arm is abducted 90° while the exam- iner places a posteriorly directed force on the patient's humeral head with his hand. The posterior force is main tained while the arm is brought into the extreme of exter- nal rotation. The humeral head is then released. The result is considered positive if the patient experiences a sudden pain, a distinct increase in pain, or when the patient states that his or her symptoms have been repro- duced." This test is depicted in Figure 2. In this series of 82 patients, the sensitivity was 92%, specificity was 89%, positive predictive value was 87%, and the negative pre- dictive value was 93%. In the article by Gross and Distefano4 the observation was made that to fulfil the criteria of Rowe and Zarins15 for a positive test result, the patient had to experience both apprehension and pain, and that the statement by Jobe and Jobe5 was that pain alone was suggestive of rotator cuff abnormalities, while pain and apprehension suggested instability.

What position produces the greatest bicep contraction?

Kim et al25 reported that maximal biceps activity occurred when the shoulder was abducted to 90° and externally rotated to 120° in patients with anterior instability. Because this position is remark- ably similar to the cocking position of the overhand throwing motion, the finding of instability may cause or facilitate the progression of internal impingement (impingement of the infraspinatus on the posterosuperior glenoid rim) in the overhead athlete

What is the etiology of lateral humeral condyle fracture?

Lateral condyle fractures, as depicted in the image below, account for 17% of all distal humerus fractures and 54% of distal humeral physeal fractures. The frequency of lateral condyle fractures peaks in children aged 6 years. Most fractures occur in children aged 5-10 years. Cases have been reported in patients as young as 2 years and as old as 14 years.

What is the frequency of lateral humeral condyle fracture?

Lateral condyle fractures, as depicted in the image below, account for 17% of all distal humerus fractures and 54% of distal humeral physeal fractures. The frequency of lateral condyle fractures peaks in children aged 6 years. Most fractures occur in children aged 5-10 years. Cases have been reported in patients as young as 2 years and as old as 14 years.

What is the pathology and MOI of Lateral Epicondylitis?

Lateral epicondylitis is the most common elbow injury, and is otherwise known as "tennis elbow". It is an injury to the extensor muscle of the forearm, causing pain to the origin of the muscle on the lateral epicondyle. Lateral epicondylitis is an overuse injury due to excessive gripping motions and stress upon the extensors in the forearm.

What are the absolute contraindication of brachial plexus injury,

More than two previous episodes of transient quadriparesis/quadriplegia... clinical history, physical examination finding, or imaging comfirmation of cervical myelopathy/melomalacia... continues cervical neck discomfort, decreased range of motion, or any evidence of a neurologic deficit from baseline after any cervical spine injury

What is the history of UCL injury?

Medial elbow pain is the most common symptom in athletes who throw. Pain may be especially prominent during the acceleration phase of the overhead throw. Pain is often chronic or recurrent, and it may lead to a slow erosion of the patient's throwing ability. Athletes may report having had similar pain in previous seasons of throwing. Rest generally helps to relieve the pain. Occasionally, during a single throw, athletes may experience acute pain over the medial elbow, sometimes associated with a popping sensation, that causes them to immediately stop throwi

What is the etiology of Monteggia Fracture?

Monteggia fractures are primarily associated with falls on an outstretched hand with forced pronation. If the elbow is flexed, the chance of a type II or III lesion is greater. In some cases, a direct blow to the forearm can produce similar injuries. Evans in 1949 and Penrose in 1951 studied the etiology of Monteggia fractures on cadavers by stabilizing the humerus in a vise and subjecting different forces to the forearm.[8, 9] Penrose considered type II lesions a variation of posterior elbow dislocation. Bado believed that the type III lesion, the result of a direct lateral force on the elbow, was primarily observed in children. In essence, high-energy trauma (eg, a motor vehicle collision) and low-energy trauma (eg, a fall from a standing position) can result in the described injuries. A high index of suspicion, therefore, should be maintained with any ulna fracture.

What are the different types of shoulder dislocation types

Most anterior dislocations are subcoracoid in location. Subglenoid, subclavicular, and, very rarely, intrathoracic or retroperitoneal dislocations may occur.

What are the mechanics of the O'brien active compression test

O'Brien et al.14 described this test in 1998 to distinguish between superior labral and acromioclavicular abnormal- ities. The test is depicted in Figure 6. "This test wasconducted with the physician standing behind the patient. The patient was asked to forward flex the affected arm 90° with the elbow in full extension. The patient then ad- ducted the arm 10° to 15° medial to the sagittal plane of the body. The arm was internally rotated so that the thumb pointed downward. The examiner then applied a uniform downward force to the arm. With the arm in the same position, the palm was then fully supinated and the maneuver was repeated. The test was considered positive if pain was elicited with the first maneuver and was re- duced or eliminated with the second maneuver. Pain lo- calized to the acromioclavicular joint or on top of the shoulder was diagnostic of acromioclavicular joint abnor- mality. Pain or painful clicking described as within the glenohumeral joint itself was indicative of labral abnor- mality." The results in their series of over 300 patients were a sensitivity of 100%, a specificity of 99%, a positive predictive value of 95%, and a negative predictive value of 100% for labral abnormalities. For the acromioclavicular joint, the sensitivity was 100%, the specificity was 97%, the positive predictive value was 89%, and the negative predictive value was 89%.

What is the management of Volkmann contracture?

Often times Volkmann's contractures are caused by supracondylary fracture. So before treatment occurs the fracture must be healed. Fasciotomy's are often performed to release the pressure within the muscles. Physical therapy is used to make certain that mobility is recovered. It must be noted that both the antagonistic and agonistic muscles are strengthened.

What is the outcome and prognosis for anterior shoulder instability?

Outcomes of anterior glenohumeral repair rely on identification of all of the patient's pathology and choosing the correct repair. With these key factors addressed, patients can expect a low incidence of complications, good range of motion, and stable repairs with a low incidence of recurrence.[28, 29] Potential morbidities associated with primary anterior glenohumeral dislocations are rotator cuff tear, greater tuberosity fracture, and/or neurological deficit. These injuries may occur alone or in combined patterns and are more common in patients aged 60 years or older.[3

What are the signs and symptoms of TOS?

Pain,numbness and tingling in the neck, shoulder, arm and hands... weakness in the arms and hands... poor circulation, characterized by coldness, swelling, blueness in the hand and fingers.

What is the definition of a posterior elbow dislocation?

Posterior elbow dislocation (PED) occurs when the radius and ulna are forcefully driven posterior to the humerus. Specifically, the olecranon process of the ulna moves into the olecranon fossa of the humerus and the trochlea of the humerus is displaced over the coronoid process of the ulna. PED is classified as simple or complex and staged according to severity

What therex has high EMG for mid trap

Prone rows

What are the images for lateral humeral condyle fracture?

Radiographs Obtain standard anteroposterior (AP), lateral, and oblique radiographs in patients with a suggested elbow fracture. Obtain a comparison view of the contralateral (ie, uninjured) elbow as a control or template. This is especially helpful when ossification is not yet complete. Varus stress views have been recommended in questionable cases. However, these are painful to the patient and may displace a previously undisplaced fracture. Reserve stress views for the operating room, where they can be performed under fluoroscopy and can assist in the decision of open versus percutaneous treatment. Arthrogram assesses the size of the cartilaginous fragment and the articular displacement and can help in decision making in difficult cases. However, this study is difficult to achieve without sedation and also should be reserved for the operating room. MRI may be used to determine the size and degree of displacement. MRI has taken the place of preoperative arthrograms in cases that are difficult to manage. Sedation may be required.

What is the diagnostic procedure for radial head fracture?

Radiographs would be the diagnostic imagery of choice to evaluate for radial head fracture. It is important to note the orientation of the radial head to the capitullum. Type I Involves cracks and/or chips in the radial head, but the fractures are not displaced The fracture is sometimes not visible until 2-3 weeks post injury once some of the healing has occured. Type II The fractures are slightly displaced involving larger pieces of bone. Type III www.physio‐pedia.com/index.php?title=Radial_Head_Fracture 3/5 9/3/12 Radial Head Fracture ‐ Physiopedia, universal access to physiotherapy knowledge. Multiple pieces of bone involved Displacement is so severe, that the head cannot be reconstructed Aspiration is used at times, to confirm suspicion of fracture. Globules of fat in the fluid are an indication that fracture may be present.[2]

What is the anatomy of the anterosuperiior labrum?

The anterosuperior labral fibers appear to be at- tached more to the middle and inferior glenohumeral ligaments than directly to the glenoid rim itself.

What type of blood flow is along the inferior labrum?

The anterosuperior labrum appears to generally have poor blood supply, whereas the inferior labrum exhibits significant blood flow.10 Vascularity of the labrum decreases with increasing age

What is the pathophysiology of Monteggia Fracture?

The forearm structures are intricately related, and any disruption to one of the bones affects the other. The ulna and radius are in direct contact with each other only at the proximal and distal radioulnar joints; however, they are unified along their entire length by the interosseous membrane. This allows the radius to rotate around the ulna. When the ulna is fractured, energy is transmitted along the interosseous membrane, displacing the proximal radius. The end result is a disrupted interosseous membrane proximal to the fracture, a dislocated proximal radioulnar joint, and a dislocated radiocapitellar joint. Radial head dislocation may lead to an injury of the radial nerve. The posterior interosseous branch of the radial nerve, which courses around the neck of the radius, is especially at risk, particularly in Bado type II injuries.[10] Injuries to the anterior interosseous branch of the median nerve and the ulnar nerve also have been reported. Most nerve injuries are neurapraxias and typically resolve over a period of 4-6 months. Splinting of the wrist in extension and finger range-of-motion exercises help to prevent contractures from developing while the patient awaits resolution of the nerve injury.

What are the 3 bundles of the UCL?

The ulnar collateral ligament is composed of three bun- dles: an anterior bundle, a posterior bundle, and a variable transverse oblique bundle (Fig. 1). The anterior bundle of the ulnar collateral ligament is the primary restraint to valgus force of the elbow from 30° to 120° of flexion and is subjected to near-failure tensile stresses during the acceleration phase of the throwing motion.

What is the management of medial epicondylitis?

There is a surgical treatment for medial epicondylitis, removing of the degenerative tissue at the origin of the flexor cari radialis brevis muscle. Steroid injections can be used to treat chronic pain, but only a few times. Physical therapy can be used to relieve pain and reduce inflammation in the area.

What therex is a good anterior serratus therex with GHJ compression?

Wall slides

The National Federation of State High School Associations recommends Pre-Participation Evaluations at regular intervals throughout an athlete's high-school athletic participation. What is the main purpose of a Preparticipation Screening process?

a. Disclosure of defects that may limit participation Incorrect: The PPE should not be seen as the "baseline" from which to build upon during the season.

The following question refers to an article attempting to identify a clinical prediction rule for identifying wrist fractures in pediatric patients with acute injuries. The authors prospectively examined 48 patients presenting to one hospital Emergency Department. The authors identified the following rule for predicting a fracture: tenderness over the distal radius and at least 20% loss of grip strength. The results of the study are given below: The sensitivity for the prediction rule was 0.79. The prevalence of fractures in this sample was 50% (24/48). If a follow-up study was performed in which the prevalence of fracture was 75%, the sensitivity would most likely:

a. Remain about the same Incorrect: The prevalence of the pathology should not affect the sensitivity of the test or prediction rule. For a refreshingly easy to understand article about sensitivity and specificity, refer to Loong (2003).

A 17 year old female who is a 3-sport athlete and currently participating in track comes to the high school clinic complaining of vague, right anterior knee pain. She notes that she had mild discomfort in basketball but now 3 weeks into track season the pain has increased to a level of a 6/10. She describes the pain as an aching with occasional sharp pain. She notes the pain worsens during and after track practice but denies any night or morning pain. What biomechanical observations would be consistent with the treatment plan for orthotic prescription for this athlete?

a. Tibial internal rotation Correct: The use of foot orthotics for PFPS is based on the assumption that excessive foot pronation during stance phase would cause abnormal tibial internal rotation that consequently would disrupt the normal tracking of the patella inside the trochlear groove. For a good review of patellofemoral interventions, read the systematic review by Bizzini et al. (2003)

17 year old pitcher presents to you complaining of shoulder pain and instability. He can not recall a specific traumatic incident. Pain and apprehension are most severe in abduction and maximal shoulder extension and external rotation. You suspect microinstability, and compromise of the rotator interval. What are the boundaries of the rotator interval?

a. a triangular region between the anterior border of the supraspinatus tendon and the superior border of the subscapularis tendon. Correct: Over the past two decades, it has become accepted that the rotator interval is a distinct anatomic entity that plays an important role in affecting the proper function of the glenohumeral joint. The rotator interval is an anatomic region in the anterosuperior aspect of the glenohumeral joint that represents a complex interaction of the fibers of the coracohumeral ligament, the superior glenohumeral ligament, the glenohumeral joint capsule, and the supraspinatus and subscapularis tendons. Lesions of the rotator interval may result in glenohumeral joint contractures, shoulder instability, or in lesions to the long head of the biceps tendon.

When performing a sideline assessment of a conscious football player who was believed to suffer a concussion, identify which following action(s) are inappropriate?

b. Cervical PROM to assess for bony block or instability Incorrect: following any head trauma (mild to severe) primary & secondary survey's should be repeated after any change in position or status to insure no rapid decline in mental or physical function. For a good review of concussion management, review the 2008 Zurich Guidelines.

What type of surgery option do you use if the external rotation is greater than 130 degrees

f there is greater than 130° of external rotation with the scapula stabilized, one should con- sider electrothermal shrinkage versus arthroscopic capsular plication of the anterior band of the inferior glenohumeral ligament.

A physical therapist who wishes to improve glenohumeral external rotation using joint mobilizations should direct their force (on the humerus) in which direction?

c. Posterior glides Correct: The glenohumeral joint does not comply with strict concave-convex rules due to capsular constraint. The humeral head needs to have enough capsular mobility available to posteriorly translate on the glenoid during external rotation. This concept was supported in a recent RCT (see Johnson et al., 2007), which found greater improvements in ER with posterior gleno-humeral glides.

Snyder's biceps tension test, compression rotation test, anterior slide test, crank test are all designed to test for the presence of...

d. A SLAP lesion Incorrect: These tests are used to detect a SLAP lesion. This was described in your weekly readings. See Tennett et al., (2002).

4-year-old male gymnast complains of left-sided low back pain for 3 months. He had plain radiographs, which were negative. A SPECT bone scan (see A below) reveals increased uptake of the left pars and adjacent pedicle. Thin-cut CT scan confirms a stress fracture of the pars (see B below). How should this individual be managed?

d. Cessation of gymnastics, coupled with TLSO bracing for 6-8 weeks. Incorrect: Cessation from sport is essential, and physical therapy indicated. However, given the significant updake seen on the SPECT, chronicity of the pain, and his relative youth, TLSO bracing will be vital

When considering shoulder joint stability, the capsule, labrum, and capsular ligaments can all be considered...

d. Static stabilizers Correct: These structures are passive, or static, stabilizers, as described in your weekly readings. See Hayes et al., (2002).

What is the surgical care for ulnar neuropathy?

f nonsurgical methods fail, and in patients with severe and/or progressive weakness/atrophy, specific surgical techniques such as medial epicondylectomy, simple release of the flexor carpi ulnaris aponeurosis, and anterior transposition of the nerve are often beneficial in cases of ulnar neuropathy at the elbow.[69] Entrapments in Guyon canal are also amenable to surgical treatment.[2] A Cochrane review presented results of 2 meta-analyses of 5 randomized, controlled clinical trials of surgical treatments for idiopathic ulnar neuropathy at the elbow.[67] Four of the studies addressed simple decompression compared with decompression plus transposition.[70, 71, 72, 73] These studies found no significant difference between simple decompression of the nerve and decompression with either submuscular or subcutaneous transposition. This was true both for clinical outcomes and neurophysiological outcomes (ie, nerve conduction and EMG). One difference between the two approaches was that decompression with transposition produced more superficial and deep wound infections.[67] Two additional meta-analyses, using somewhat different meta-analytic methods, have also concluded that they can find no significant differences between the outcomes of simple decompression compared with decompression plus transposition.[74, 75] However, one of these studies, detected a trend in favor of decompression plus transposition, and the authors opined that perhaps a more highly powered study could detect a difference.[75] The Cochrane review also examined one study that compared epicondylectomy with anterior transposition and concluded that no significant differences could be found in either clinical or neurophysiological outcomes.[67] Interestingly, patient satisfaction was higher in patients treated with epicondylectomy.[76] Surgery is also valuable for correction or stabilization of traumatic injuries, resection of masses/cysts, and sectioning of fibrous bands.

What forces are being place on the elbow during throwing?

he elbow joint complex undergoes tremendous tension (medial elbow) and compression (lateral elbow) forces. Tension forces across the medial elbow are maximized during the acceleration phase of throwing, while tension, compression and torsional forces are all being applied to the posterior compartment during both the acceleration and deceleration phases. Repetitive application of these stresses/forces result in a variety of elbow pathologies. Following are general rehabilitation guidelines for elbow pathologies:

What surgery provides the most stability of the UCL?

he figure-of- eight and docking techniques for ulnar collateral ligament reconstruction provided the highest return to sport and functional activity.

What is the normal carrying angle?

he nor- mal carrying angle is 11° of valgus in men and 13° of valgus in women.

What are the three factors for multidirectional instability

include suboptimal muscle con- trol for shoulder function, a deficiency in the rotator cuff interval, and connective tissue abnormalities.

What is the management for TOS

initial treatment involves medication and avoiding activity that causes the problem. Physical therapy and exercises are usually prescribed to promote shoulder muscle function and improve any posture abnormalities. Avoid straining or heavy activites for 3 months.

What special tests should you perform with peel back injury?

pronator load resisted supination and external rotation bicep 1 and 2 test pain provocation crank test

What are the range of motion restriction for SLAP surgery?

lexion ROM is performed to toler- ance. ER and IR in the scapular plane are initiated at 45° of glenohumeral abduction and advanced to 90° of abduction usually by postoperative day 4 or 5. ROM exercises may be performed early because an anatomical repair has not been performed.

What are the outcome measures for lateral epicondylitis?

mASES QuickDASH (reliability= .90, MCID not reported) DASH_Outcome_Measure PSFS (reliability= .82-.92, MCID=2.0)

What is the range of SICK scores?

most throwers with symptomatic SICK scapulas present with scores between 10 and 14. In- terval throwing usually begins with scores in the 4 to 6 range, and return to sport at the thrower's previous level of performance is attained when the score drops between 0 and 2.

What is the management of Lateral epicondylitis?

o NSAID's are often used for the management of the pain for lateral epicondylitis. Patients are advised to relax and rest to help reduce the pain, along with wearing a counterforce brace and corticosteriod injections. Physical therapy is often more beneficial for the patients by performing elbow mobilizing exercises.

What is the management of pronator teres syndrome?

o Standard treatment of PRICE is recommended in young athletes prior to injections and/or surgical decompression. Injection of corticosteroids into the pronator teres muscle produces may relief of symptoms, but a strong response to a steroid injection would be more consistent with carpal tunnel syndrome. Massage therapy can also provide relief for individuals experiencing this condition. Surgical decompression can provide benefit in selected cases.

What are the positive finding with the moving valgus stress test?

t must have 2 key components. First, the pain generated by the maneu- ver must reproduce the medial elbow pain at the MCL that the patient has with activities. Second, although the patient may experience pain throughout a range, the pain should be maximal between the position of late cocking (120°) and early acceleration (70°) as the elbow is extended.

What are special testing/imaging for an anterior dislocation?

1) X-ray

What are the two mechanism of a SLAP lesion?

In theory, SLAP lesions most likely occur in over- head athletes from a combination of these 2 previ- ously described forces. The eccentric biceps activity during deceleration may serve to weaken the biceps- labrum complex, while the torsional peel-back force may result in the posterosuperior detachment of the labral anchor.

What is the management of elbow dislocation?

Reduction under local or general anesthesia is the primary treatment. Surgery is also another option for treatment of an elbow dislocation. Treatment can vary from aggressive immediate active range of motion to traditional plaster immobilization for several days.

What is the etiology for proximal humeral fracture

The most common mechanism for proximal humerus fractures is a fall on an outstretched hand from a standing height. In younger patients, high-energy trauma is a more frequent cause, and the resultant injury is more devastating. Additional mechanisms include violent muscle contractions from seizure activity, electrical shock, and athletic injuries. Finally, a direct blow to the proximal humerus may also lead to fracture.

What is the imaging for median neuropathy?

Wrist magnetic resonance imaging (MRI) has the greatest sensitivity and specificity of the available imaging modalities for the diagnosis of CTS but is usually not necessary.[30] Current MRI research is focusing on the use of diffusion tensor imaging (DTI) and tractography of the median nerve in the diagnosis of carpal tunnel syndrome.[31, 32] Tractography in patients with chronic compression of the median nerve typically shows that mean fractional anisotropy values in the median nerve are significantly lower within the carpal tunnel than in locations proximal to the carpal tunnel. Preliminary results suggest that quantitative evaluation of the median nerve with DTI is precise, and that in patients with unilateral involvement, the healthy contralateral nerve can be used as an internal control.[33] Computed tomography (CT) scan of the wrist may be helpful in patients with subtle bony trauma. Wrist ultrasonography may evaluate the soft tissues of the carpal tunnel including the tendons and the median nerve and allow cross-sectional measurement of the carpal tunnel. Wrist radiograph can be used if a fracture (old or recent) is suspected. Cervical spine radiograph and/or MRI can be used if cervical radiculopathy is suspected. Chest CT scan, chest radiograph, or brachial plexus MRI can be used in patients with suspected brachial plexopathy or thoracic outlet syndrome.

You are the assigned sports medicine personnel for a local high-school baseball team. You have been watching the weather all day and thunderstorms are predicted but no watch or warning has been completed on the local weather station. You report to the field 30 minutes prior to the scheduled start of the game with intermittent drizzling occurring and you discuss the situation with the designated officials. There has been no delay or cancellation announced at this point and the game begins as planned. In the second inning, you hear some thunder rumbles and then notice in the distance a lightening flash. Which of the following is not a sign of increased electrical field activity?

b. Crackling sounds on a phone line Correct: This is NOT a sign of increased electrical field activity. Regardless, land-line telephones should not be utilized during a thunder storm. Cell phones are considered safer. See NATA Position Statement on Lightening Safety.

A 195 pound 17 year old male returns to football daily doubles after summer break. After a particularly intense morning workout (including eccentric weight training), and practice in the 90 degree weather in the early evening, he complains of nausea, as well as bilateral pain of his gastrocnemius and quadriceps. He is markedly point tender to the bilateral gastrocnemius and quadriceps musculature as well. You compared his pre-practice weight to his post-practice weight, and find a difference of 0.5 pounds. What is the best management of this individual?

b. Refer to emergency department Incorrect: AER is a potentially life threatening emergency. Prompt transport to the emergency room is vital

An athlete who sustains a concussion with loss of consciousness followed by a lucid interval and then a declining mental status, should be closely monitored for what?

c. Epidural hematoma Incorrect: As described by Chele Weinstein in SPTS's Emergency Response Home study course, the symptoms of a subdural hematoma are related to the underlying brain injury as opposed to the mass effect from the clot. The athlete may progress from an awake, alert status with no focal neurological deficits to significant neurological involvement and death in a few short hours. The mortality rate is 30-40 percent, even with prompt neurosurgical intervention.

An 18 year old female soccer player receives a blow to the medial knee as she plants her left leg. She suffers a lateral patellar dislocation. This is her first patellar dislocation. She is successfully reduced, and receives an MRI. The MRI report notes a bone bruise along the midlateral femoral condyle, medial patellofemoral ligament tear (grade II), extraarticular swelling, and a grade III osteochontral defect of the medial facet of the patella. Now 1 week s/p injury, her pain is 2/10 at rest, and she has a slightly antalgic gait pattern, WBAT, with bilateral crutches. Which of the following management strategies would research support?

c. Operative management Incorrect: According to a systematic review of first time patellar dislocations (Stefancin & Parker, 2007), conservative management is indicated. Surgical intervention is appropriate with an osteochondral fracture, substantial disruption of the medial patellar stabilizers, a laterally subluxated patella with normal alignment of the contralateral knee, a second dislocation, or in patients not improving with appropriate rehabilitation.

An inferior subluxation of the humeral head produced by downward traction on the arm is a description of...

c. a positive sulcus sign Correct: A positive sulcus sign is described as the inferior subluxation produced by downward traction on the arm. It is a test for multidirectional instability. This was described in your readings. See Tennett et al., 2003.

A 195 pound 17 year old male returns to football daily doubles after summer break. After a particularly intense morning workout (including eccentric weight training), and practice in the 90 degree weather in the early evening, he complains of nausea, as well as bilateral pain of his gastrocnemius and quadriceps. He is markedly point tender to the bilateral gastrocnemius and quadriceps musculature as well. You compared his pre-practice weight to his post-practice weight, and find a difference of 0.5 pounds. You suspect which of the following diagnoses?

d. Acute extertional rhabdomyolysis Incorrect: Bilateral cramps in both the gastroc's and the quads suggests something more severe than heat cramps

You see a collegiate basketball player during the spring semester for pain and dysfunction s/p left bimalleolar ORIF. You sent her home for the summer with a HEP. She is now returning in the fall semester. Currently, she reports that she has experienced a persistent anterolateral pain with running, and no pain at rest. When analyzing her running gait you notice that she has a shortened stance phase on the involved side with early heel off, pain is reported at the end of the stance phase after running for several minutes. She denies any buckling or feelings of instability. Given the information above, which of the following is the most likely diagnosis?

d. Anterolateral Impingement Syndrome Correct: Anterolateral impingement syndrome is caused by scar tissue impingement in the lateral gutter. This syndrome is common in athletes after an inversion ankle sprain. It is usually absent at rest and increases with activities. These patients do not usually report signs of instability. If physical therapy is not effective steroid injection or arthroscopic debridement may be required.

A 14 year old elite gymnast complains of constant anterior shin pain of insidious onset which increases with activity and decreases with rest. Symptoms have been present for 3 weeks after joining her school track team in addition to gymnastics. Her past medical history is significant for a stress fracture of her 5th metatarsal. Which one of the following questions would best screen for a component of the female athlete triad?

d. Have you are any of your family members been diagnosed with osteopenia or osteoporosis? Incorrect: As a baseline screening, today's caloric intake in not pertinent

The ACL is stressed, or under load, all of the following conditions except what?

d. Tibial external rotation moment incorrect: The ACL indeed assists in controlling frontal plane stress such as varus and valgus moments.

During prolonged, submaximal, steady state exercise, there is a gradual shift of bioenergetics as described by the following:

d. carbohydrates shifting to proteins and fats Incorrect: during submaximal, steady state exercise, the initial fuel is the carbohydrate substrate

What rhomoid therex has high EMG reading

prone ext with ER

What structures are being involved during the Cocking phase?

"Places AGHL and anterior capsule on stress This repeated action will lead to posterior impingement infraspinatus doing 90% of the ER This is the true ER MMT"

Differential diagnosis of elbow pain in a pitcher (OCD lesions, UCL tears, apophysitis, flexor/pronator mass, etc.)

...

Itis" vs. "osis" with regards to tendinous injuries. Histologically, what is different, and how should our treatment differ for each?

...

What is the nerve and muscle for the medial pectoral nerve?

1) C5-C6-C7.....2) lower pectoralis major.....3) adduction/extension/depression of the arm

what is the nerve and muscle for the lateral pectoral nerve?

1) C5-C6-C7.....2) upper pectoralis major....3) adduction/flexion of the arm

sprains with regards to specific ligaments"

1) Type 1: is a partial injury to the capsule. Paired with an incomplete tear of the acromioclavicular ligament and joint capsule... 2) Type 2: is a moderate injury. A partial tear of the ligament coracoclavicular with a rupture of the capsule and the acrominclavicular ligament. Often described as a subluxation. 3) Type 3 injury: a severe injury. Complete ruture of the acromioclavicular ligament but also the coracoclavicular ligament. This can lead to displacement of the clavicle and therefore very unstable. This is often described as a dislocation. Type 4: posterior dislocation.. Type 5: dislocation with serve upward displacement of the clavicle into or through the traqezius... Type 6: This is a rare injury. Dislocation of the clavicle inferiorly. locked under the coracoids process.

What special testing/imaging for posterior dislocation?

1) X-ray

What are the special testing/imaging for anterior instability

1) fulcrum test.. crank apprehension test. jerk test, anterior/posteiror load and shift... sulcus sign.

What is the management of a Clavicular fracture

1) intital treatment consists of ice, compressive dressing, and medication to relieve pani and reduce swelling. 2) An arm sling, shoulder immobilizer or figure of eight brace is usually recommended. 3) Refrain from sporting activities until the fracture is healed, a mimimum in 6 weeks. 4) exercise to regain shoulder motion and strength are necessary before returning to sports.

What are the signs and symptoms of a ACJ sprain?

1) marked pain with movement of the shoulder. especially horizontal adduction...2) Noticable deformity with grade 2/3...3) point tenderness over the ACJ

What are the signs and symptoms of a clavicle fracture.

1) pain and tenderness and swelling at the fracture. 2) deformity/bump..3) bruising... 4) loss of strength... 5) occasionally numbness or coldness in the shoulder on the affected side

What are the special tests or imaging for a humeral fracture

1) x-ray

What is the management of posterior elbow dislocation?

Before surgery is considered, research indicates reduction under local or general anesthesia as the primary treatment for PED.[6][7][8][9][15] Patient presentation including elbow stiffness and pain are key factors when considering the need for surgery along with irreducible dislocation, gross instability, neurovascular injuries, and associated fractures.[9][14] The most common surgical options include an open procedure, with or without Speed's procedure, and excision or closed arthroplasty. An open procedure, more commonly seen in neglected PED < three months, involves ulnar nerve release, humeroulnar and humeroradial reduction, possible triceps lengthening using Speed's procedure, and wires and/or screws placed in the olecranon for stabilizing the joint.[9] In the Elzohairy study, within two weeks the wires were removed and active motion was initiated, while the screws were removed six months after surgery. Excision arthroplasty is also used when patients present with neglected PED, but studies suggest high reoccurrences of pain and instability.[9] In other studies, surgery was indicated only when concomitant fractures occurred with PED.[7][9] Ligaments injured with fractures or dislocations are repaired via sutures attaching them back to the bone. Once surgery is complete, the patient is immobilized with time frames varying based on the individual and the surgeon's protocol.[7][8][9] Hinged braces, fixators, plaster casts, and slings are utilized to keep the elbow in a position of approximately 70-80o of flexion and slight pronation. Active movement is usually initiated between three to fourteen days, with slow, gradual supination.[6][7][9] When treating a post-surgical PED patient, physical therapists should be cautious of pin site infection.[9] A patient is able to return to functional activities around twelve weeks and sports around six months.[7] While much of the research highlights general dislocation of the elbow with some positive outcomes following surgery, there is not enough evidence to support surgical interventions for PED.[6][8] Also a disadvantage in studies that report positive outcomes post-surgery is the lack of comparison against non- surgical interventions, such as physical therapy

Based on your examination findings of a patient with chronic shoulder pain, you suspect that a majority of their symptoms are a result of decreased muscular activation of the mid & lower trapezius, with increased upper trapezius activation. While incorporating current best evidence into your decision-making, select the list of exercises most effective in restoring this imbalance.

b. Side-lying shoulder flexion, side-lying ER, horizontal abduction with ER, prone extension Correct: These 4 exercises identified by Cools et al., 2007 were found to be effective in promoting lower & mid trapezius activity while minimizing upper trapezius recruitment.

You are covering a ski race on Mount Hood and the condition on the course is 25 degrees F with a 30 mph North wind. A skier finishes the race and you notice blanched, white skin on his left cheek. He states that he has another race in 20 minutes. Palpation to the affected area reveals no resistance from the tissue. The best plan of care would be:

c. Warm his cheek with the palm of your hand until the color returns. Cover the area with protective garb and instruct the athlete to resume racing. Incorrect: The correct diagnosis is frostnip. Frostnip does not require immediate medical evaluation, nor does it require prompt transport to warm/indoor area. See the charts below for common signs/symptoms, and treatment, of cold injuries. The charts are taken from SPTS's home study course on Emergency Response for the Physical Therapist (ed. Smith, 2004).

The following question refers to an article attempting to identify a clinical prediction rule for identifying wrist fractures in pediatric patients with acute injuries. The authors prospectively examined 48 patients presenting to one hospital Emergency Department. The authors identified the following rule for predicting a fracture: tenderness over the distal radius and at least 20% loss of grip strength. The results of the study are given below: Do the results support the authors' contention that the prediction rule is an effective screening procedure for pediatric wrist fractures?

c. No, the negative likelihood ratio is not sufficient to cause a significant shift in probability. Incorrect: there are several problems with this answer - first, the pre-test probability is 50% (probably not well described by 'very low') - 24/48 arriving at this ER had a fracture on x-ray. Second, while the pre-test probability affects clinical decision making it does not help when determining the validity of the test as a screening tool.

A physical therapist is testing to rule out inferior shoulder laxity. The therapist performed a load and shift test and obtained a negative result. Given the negative likelihood ratio of 0.92 reported for the load and shift test in this patient population, the therapist should do what?

c. Perform a hyper-abduction test to further examine for shoulder instability Correct! This test has a -LR of 0.16, which will be more helpful in allowing the examiner to rule out this condition. Review the EBP section to nail down likelihood ratios as needed.

You see a collegiate basketball player during the spring semester for pain and dysfunction s/p left bimalleolar ORIF. You sent her home for the summer with a HEP. She is now returning in the fall semester. Currently, she reports that she has experienced a persistent anterolateral pain with running, and no pain at rest. When analyzing her running gait you notice that she has a shortened stance phase on the involved side with early heel off, pain is reported at the end of the stance phase after running for several minutes. She denies any buckling or feelings of instability. Given the minimal information given in the above scenario, which of the following mobilization techniques would be the most appropriate?

c. Posterior Glide of the Talus Grade IV (Maitland) Correct. A posterior glide of the talus would be appropriate to improve closed chain dorsiflexion (the likely impairment given the patient has pain at end stance). The patient does not have pain at rest so a Grade IV mobilization would most likely be tolerated.

Which of the following interventions is the best choice for a pitcher who has unilateral scapular winging secondary to long thoracic neuropraxia?

c. Push-up position with end range scapular protraction, elevated as necessary to perform the exercise pain-free Incorrect: While this exercise is appropriate and even targets some important muscles for scapular control (primarily middle trapezius/rhomboids), there is another answer that more directly addresses this impairment.

A professional baseball pitcher presents to PT with a gradual onset of medial elbow pain, decreased velocity and accuracy with pitching, and a feeling of laxity in the elbow. He is diagnosed with a UCL tear, and surgery is recommended. Which of the following interventions is most indicated?

d. Docking procedure for the UCL Incorrect: Jobe et al. (1986) were the first to report the results of a reconstructive technique for injury to the ulnar collateral ligament of the elbow. The original procedure (known as the "Tommy John procedure") involved reconstruction of the anterior band of the ulnar collateral ligament by placing a free-tendon graft in a figure-eight fashion in bone tunnels in the ulna and medial epicondyle of the humerus and suturing the graft to itself. The flexor-pronator mass origin was detached from the medial epicondyle, and submuscular ulnar nerve transposition was performed. 63% returned to their previous level of participation in sports; however, the authors noted a high incidence of complications related to the ulnar nerve, including ulnar neuropathy in two patients who then required secondary operations. See the following pic of a typical "Tommy John"

A 19 year old collegiate wrestler presents to you with the following lesion. He reports that it spontaneously arose over the past several days. What is it, and what are his return to play restrictions (if any)?

d. Herpes gladiatorum; lesion must be surmounted by a FIRM ADHERENT CRUST at competition time, and have no evidence of secondary bacterial infection Incorrect: Impetigo is usually present on the trunk or the extremities with raised blisters that rupture easily, resulting in moist erosions surrounded by a scaly rim. Nonbullous impetigo presents with thinwalled vesicles that rupture into a honey-colored crust (most common). The return to play decision is correct for impetigo. See NATA Position Statement (2010) for diagnosis and intervention for commonly seen dermatologic conditions in athletics.

A 195 pound 17 year old male returns to football daily doubles after summer break. After a particularly intense morning workout (including eccentric weight training), and practice in the 90 degree weather in the early evening, he complains of nausea, as well as bilateral pain of his gastrocnemius and quadriceps. He is markedly point tender to the bilateral gastrocnemius and quadriceps musculature as well. You compared his pre-practice weight to his post-practice weight, and find a difference of 0.5 pounds. What is of the following is not a logical step in your examination?

d. Perform a screening for secondary compartment syndrome Incorrect: This would be appropriate. A loss of strength is most consistent with AER. With DOMS, there will be signficant soreness, but shouldn't be a signficant loss of strength.

A 50 year old runner complains of Achilles pain over the past 12-18 months. He presents to your clinic reporting that after performing intervals last week, his Achilles pain has increased substantially. His examination reveals 4-/5 PF MMT with marked pain, swelling, moderate hypertrophy of the Achilles tendon, and point tenderness along the distal 1/4 of the Achilles tendon. Which of the following interventions is not appropriate regarding treatment for this individual?

d. Ultrasound to the Achilles tendon, followed by stretching of the gastrocsoleus complex Incorrect: This has been shown to affective in treating Achilles tendonosis (Carcia et al, 2010)

What is a good supraspinatus therex to decrease middle deltoid activity.

prone full can

What therex increase middle/lower trap

prone horz adb at 90 with ER T

What the background information for a coronoid fracture?

tage one describes patients without evidence of sub- chondral displacement or fracture. Stage two refers to lesions showing evidence of subchondral detachment or articular cartilage fracture. Stage three lesions involve detached osteochondral fragments, potentially resulting in intra-articular loose bodies.

What is the MOI of posterior shoulder dislocation?

"Posterior dislocations are less common (2-10%) and are the result of an axial load applied to the adducted and internally rotated arm. Classic posterior dislocations also occur as a result of electrocution or seizures because of the strength imbalance between the internal rotators (subscapularis, latissimus dorsi, pectoralis major muscles), which overpower the external rotators (teres minor and infraspinatus muscles). "

what is the nerve and muscle for long thoracic nerve?

1) C5-C6-C7....2) serratus anterior.... 3) upward rotation of the scapula

What is the nerve level and muscle for lower subscapular nerve

1) C5-C6.... 2) subscapularis/teres major. 3) internal rotation/lateral

What exercises should be avoid with a compression SLAP injury?

For patients who sustained a SLAP lesion via a compressive injury, such as a fall on an outstretched hand, weight-bearing exercises should be avoided to minimize compression and sheer on the superior labrum.

What are the intraoperative for Monteggia fracture?

After adequate analgesia and sedation, a closed reduction of the radial head can be performed with distal traction and direct pressure over the radial head. This can be done in the emergency department or in the operating room. An open technique should be considered if the radius is fractured or irreducible. Once the radius has been reduced, the ulnar fracture is addressed with rigid internal fixation. In adult Monteggia fracture, fixation with a 3.5-mm dynamic compression (DC) plate or a limited contact - dynamic compression (LC-DC) plate is recommended. If the fracture is comminuted, purchase should be obtained, if possible, with 3-4 screws or 6-8 cortices proximal and distal to the fracture. Once the ulna is stabilized, the stability of the radial head is assessed using intraoperative fluoroscopy. Permanent radiographs should be taken, and a posterior long arm splint should be placed with the elbow immobilized in 90° of flexion and full supination for types I, III, and IV. Type II is best splinted in the same manner, but in 70° flexion at the elbow to prevent radial head subluxation. If the radial head is unable to reduce, the reduction and alignment of the ulna should be checked. If the radial head is unstable after ulnar fixation, then the elbow should be splinted in supination, which is the position of stability

What is the physical therapy for UCL?

After surgery, the elbow is immobilized in a posterior splint for 10 days in 90° of flexion. The wrist is maintained free, and a soft rubber ball is given to the patient for squeezing to maintain grip strength. Active ROM is initiated after removal of the posterior splint. A hinged brace that prevents valgus stress should be used. At approximately 1 month after surgery, isometric strengthening exercises should be initiated. Limited progressive strengthening can be initiated at 8 weeks and plyometrics at 12 weeks. Shoulder and elbow exercises should be instituted as soon as feasible, but valgus stress of the elbow should be avoided for at least 4-6 months until the graft has had sufficient time to incorporate.

What is the description of ulnar nerve transposition?

Description: Cubital tunnel syndrome consists of compression of the ulnar nerve as it passes through the cubital tunnel at the elbow. The nerve compression results in paresthesias along the ulnar nerve distribution, affecting the small finger and ulnar side of the ring. In addition, with long standing cubital tunnel syndrome, there may be residual motor weakness of the ulnar nerve innervated intrinsic muscles. Severe, prolonged ulnar nerve compression may result in clawing of the ring and small fingers.

The subsequent 2 questions are based on the following scenario: You are climbing Mount Rainier with friends and you are currently at an elevation of 9000 feet. One of your buddies stops secondary to, "not feeling well". Upon questioning, he states that he has a headache, feels nauseous, weak and a little dizzy. The group stops while he rests and drinks water. After 15 minutes, he states he is feeling better, but still has a little headache and nausea. What is the best plan of action?

Descend ~ 1000 feet and wait for 100% resolution of his symptoms then resume with the ascent. Correct: This individual is showing signs of acute mountain sickness. Typically this will resolve with an 800-1000 foot drop in elevation. It does not require termination of the ascent, but further ascent should be precluded by full resolution of symptoms. See the following snapshot taken from Benner (2007)

what is the pathology/treatment/surgery for Olecranon Stress Fractures

Description Microfractures in olecranon resulting from olecranon impingement or excessive triceps tensile stress. Treatment options Nonoperative rehabilitation, surgical internal fixation Author's preference Arthroscopically assisted olecranon fixation

what is the pathology/treatment/surgery for flexor-pronator muscle mass injury?

Description Mild muscle overuse, chronic tendonitis, or acute mm tears Treatment options Non-operative rehabilitation Author's preference Non-operative rehabilitation (active rest, anti-inflammatory meds, physical therapy, gradual return to throwing)

what is the pathology/treatment/surgery for Osteochondritis Dissecans of the Capitellum

Description Osteochondral injury and loose bodies from the capitellum or radial head Treatment options Nonoperative rehabilitation, open surgical treatment, arthroscopic treatment, humeral osteotomy Author's preference Nonoperative rehabilitation unless presence of loose bodies or mechanical symptoms, in which case arthroscopic debridement, loose body excision, and abrasion chondroplasty

What are the outcome for Monteggia fracture?

In 1991, Anderson and Meyer used criteria to evaluate forearm fractures and their prognosis, as follows[12] : Excellent - Union with less than 10° loss of elbow and wrist flexion/extension and less than 25% loss of forearm rotation Satisfactory - Union with less than 20° loss of elbow and wrist flexion/extension and less than 50% loss of forearm rotation Unsatisfactory - Union with greater than 30° loss of elbow and wrist flexion/extension and greater than 50% loss of forearm rotation Failure - Malunion, nonunion, or chronic osteomyelitis Pain, nerve dysfunction, and cosmetic deformity are other factors to consider when evaluating the outcome of treatment in Monteggia fracture-dislocations. Type II lesions that are associated with ulnohumeral dislocation have been noted to have outcome scores with greater disability than those without ulnohumeral dislocation.[4]

What is the definition of instability?

Instability is defined in this article as, "a clinical syndrome that occurs when shoulder laxity produces symptoms.

What is the definition for lateral epicondylitis?

Lateral epicondylitis is the most common overuse syndrome in the elbow. Lateral epicondylitis or tennis elbow is an injury involving the extensor muscles of the forearm. These muscles originate on the lateral epicondylar region of the distal of the humerus.[1]

What are some additional signs and symptoms of SICK scapula

Symptomatic patients with an isolated SICK scap- ula may complain of anterior shoulder pain, postero- superior scapular pain, superior shoulder pain, proxi- mal lateral arm pain, or any combination of the above. In addition, posterosuperior scapular pain may radiate into the ipsilateral paraspinous cervical region or the patient may complain of radicular/thoracic outlet type symptoms into the affected arm, forearm, and hand or any combination of the above

What is the anatomy of coronoid fracture?

The coronoid acts as the anterior buttress of the greater sigmoid notch of the ulna. It provides attachment to the anterior band of the medial collateral ligament (MCL) and the middle portion of the anterior capsule. The anterior colliculus of the MCL is the primary stabilizer of the elbow against valgus strain in the functional arc of 20-120° of flexion. This ligament is most likely to become injured with a low coronoid fracture with the elbow in full extension.[18] A fracture of the coronoid, therefore, results in the loss of all of these supports. The brachialis muscle is attached to the base of the coronoid process.[19] Dissection of the brachialis during fixation of these fractures contributes to the risk of heterotopic ossification in these cases.

What is the diagnostic procedure for lateral epicondylitis?

The diagnosis of lateral epicondylitis is substantiated by tenderness over the ECRB or common extensor origin. By the following methods, the therapist or physiotherapist should be able to reproduce the typical pain: 1. Digital palpation on the facet of the lateral epicondyle 2. Resisted wrist extension or resisted middle-finger extension with the elbow in extension 3. Having the patient grip an object

What band of the medial ligament is the main stabilizer for valgus movement?

The anterior band of the ulnar collateral ligament complex is the primary stabilizer to the valgus stress encountered at the elbow during the throwing motion.

What is the outcome for olecranon fracture?

The best outcomes are observed in patients who have nondisplaced or minimally displaced fractures treated nonoperatively. Evaluation criteria are degree of pain, range of motion, and radiographic findings. In patients treated operatively, excision with triceps repair has the lowest rate of complications. In controlled studies, pain, subjective function, isometric strength, isokinetic work, range of motion, stability, and incidence of degenerative change were similar for patients treated with internal fixation and patients treated with excision. The preferred treatment appears to be excision when possible. Patients treated with internal fixation using an intramedullary screw plus wire or cable yield the fewest complications and best results when internal fixation is necessary. Occasionally, fixation removal is required. Of patients with plate fixation, 70-80% have good to excellent results, as compared with more than 90% of patients with tension band wiring. This probably is because simple fractures are usually treated with tension band wiring, whereas less favorable fracture patterns are treated with plate fixation.

What is the relevant anatomy for Sprengel Deformity?

The dorsal scapular nerve: This nerve courses close to the superomedial border of the scapula in the plane between the rhomboid and erector spinae muscles. The dorsal scapular nerve remains anterior to the serratus anterior and the subscapular muscles. There exists a risk of injuring the nerve during dissection of the periscapular muscles at the superomedial angle of the scapula, and when the trapezius and rhomboid muscles are reflected off as a single unit from the spine in the Woodward procedure (see Treatment, Intraoperative details, below). Therefore, staying subperiosteal during the process of freeing the periscapular muscles is essential, especially at the superomedial angle of the scapula. The spinal accessory nerve: This nerve is located between the trapezius and rhomboid muscles and is, therefore, at risk theoretically; however, because the spinal accessory nerve is sandwiched between the 2 muscles, it is rarely ever injured when these muscles operate as a unit. The suprascapular nerve: This nerve runs in the suprascapular notch of the scapula and may be injured if the dissection is carried too far laterally when the superior portion of the scapula is resected. By staying at least 1 cm medial to the notch, injury to the nerve during the procedure can be avoided.

What are the intraoperative details for lateral humeral condyle fracture?

With operative treatment, an anatomic reduction should be achieved. An anterior approach is utilized to avoid the posterior vascular pedicle to the fracture fragment. This is important to avoid avascular necrosis of the fragment. If delayed (7-10 days), open reduction is performed. Irrigate and evacuate the interposed hematoma to assist in reapproximation. On closure, the periosteal flap must be sutured into place to avoid lateral spur formation and pseudocubitus varus.

What is the etiology of SLAP lesion?

When considering the etiology of superior labral (labrum) lesions, it is useful to divide them into the 2 broad categories of traction and compression injuries. A compression injury is usually secondary to a fall on an outstretched arm that is in an abducted and slightly flexed position. This can result in a compressive load to the superior labrum with a resultant tear. A direct blow to the shoulder has also been found to be a contributing factor for SLAP (superior labrum, anterior and posterior) lesions. Traction injuries can be secondary to a sudden pull in an inferior direction such as occurs when an individual loses hold of a heavy object. An overhead traction force, as when individuals attempt to catch themselves from falling from a height, can also result in a superior labral injury. Clearly, engaging in throwing sports can predispose one to developing a SLAP lesion. The exact mechanism of this is somewhat controversial. In Andrews' original 1985 study, traction force placed on the superior labrum by the biceps tendon in the follow-through phase of the throwing motion was thought to be responsible for creating a SLAP lesion.[2] The hypothesis was that the eccentric contraction of the biceps necessary to decelerate the elbow resulted in the biceps tendon detaching portions of the glenoid labrum. However, recent studies suggest that the forces generated during the late cocking phase are in fact the predominant factor. The peel-back phenomena as described by Burkhart and Morgan,[8] along with shear forces generated by a tight posteroinferior capsule, are thought to be major contributing factors to developing a type II SLAP or variants thereof. The presence of a peel-back sign can be demonstrated arthroscopically. The arm is placed into 70-90° of abduction and then progressively externally rotated. In this position, the biceps vector is now more posteriorly and vertically oriented. To accommodate this, the base of the biceps twists. A torsional load is transmitted to the superior labrum, and if the posterosuperior labrum and biceps anchor is incompetent, medial displacement of the superior labral biceps complex occurs. If more than 5 mm of the posterosuperior glenoid is uncovered or the biceps root at the level of the supraglenoid tubercle is uncovered, a posterior type II SLAP is present. As already noted, posterior capsule tightness is thought to play an important role in the development of SLAP lesions. Almost all high-demand throwers develop a posterior capsular contracture with limitation of internal rotation. This tight posteroinferior capsule is thought to result in obligatory superior translation of the humeral head when the arm is in abduction and external rotation and as a result exposes the superior labrum to large shear forces. This increased shear force is most pronounced at the same time the peel-back forces are at their maximum, increasing the likelihood of a SLAP lesion developing.

What are the indication for olecranon fracture?

he elbow is a complex hinge joint. The major stabilizers to valgus stress (ie, bending away from the body) are the medial (ulnar) collateral ligament and the radial head. The major stabilizer to varus stress (ie, toward the body) is the lateral collateral ligament complex. The coronoid process stabilizes the humerus against the distal ulna. The olecranon also prevents anterior translation of the ulna with respect to the distal humerus. The anterior surface of the ulna is covered with articular cartilage. Therefore, all fractures (except the rare tip fractures) are intra-articular fractures. The olecranon articulates with the trochlea of the humerus. The triceps inserts into the posterior third of the olecranon and proximal ulna. The periosteum of the olecranon blends with the triceps. The ulnar nerve lies on the posterior aspect of the elbow, posterior to the medial collateral ligament. The ulnar nerve sweeps anteriorly to join the ulnar artery. The ulnar neurovascular bundle may be at risk during Kirschner wire (K-wire) fixation. Fracture displacement is largely due to the pull of the triceps muscle, which tends to pull a separated fragment upward but is resisted by the strong fibrous covering on the olecranon, as is shown in the image below. The blending of fibers in the lateral ligaments, the elbow capsule, and some triceps fibers that blend with the periosteum form this fibrous covering. If the fracture force does not tear this fibrous sheath, little or no tendency toward displacement exists, even in the presence of comminution. Most olecranon fractures exhibit little or no displacement. Fragment displacement of more than 1.5 cm is uncommon, even with complete bony and soft tissue injury. Usually, wide separation of fragments indicates an old fracture with extensive tearing of the fibrous sheath in which the unopposed triceps is contracted gradually, drawing the separated fragment upward.

What is the management of UCL tears?

o NSAID's are often used to treat the pain and inflammation o It is recommended that athletes remain active to help maintain their range of motion o Surgery: Modified docking procedure is of choice

What therex is good at recuiting lower trap

prone horz adb at 90 with ER T

what rhombod therex has a good lower/middle trap ratio?

prone horz adb at 90 with ER T

What rhomboids that recruits all trap muscles

prone rows


Conjuntos de estudio relacionados

Marketing Principles 1-5 Test (Quiz Questions)

View Set

Music Chapter 54, CHAPTER 54 MUSIC QUIZ, Chapter 53: MUS 110, Chapter 52: MUS 110, Music Test 2

View Set

Microeconomics Chapter 4: Desmans, Supply, and Markets

View Set

Biology 256 - Iowa State University - Final Exam Questions & Answers (2)

View Set

Chapter 6 - PSYC 2301 - Learning

View Set

Google Analytics Certification Exam

View Set

Nature Spirituality and Pop Culture Midterm Study Set

View Set